bankruptcy outline - pottow 2011

157
Outlines for Bankruptcy Professor Pottow Chapter 1 Collection without Courts 1. Fair Debt Collection Practice Act, p.15-20 o Why do we care about the debt collection method? Efficient? If consumers aren't protected, they may be scared away from borrowing. But if more debts are collected by extreme practices, the savings will be passed on to the borrower, encouraging him to borrow. o HEINTZ V. JENKINS , 1995, p.20 Debt collector: definition: The term "debt collector" means any person who uses any instrumentality of interstate commerce or the mails in any business the principal purpose of which is the collection of any debts, or who regularly collects or attempts to collect, directly or indirectly, debts owed or due or asserted to be owed or due another. Is a lawyer a debt collector? Holding: the FDCPA applies to attorneys who “regularly” engage in consumer-debt- collection activity, even when that activity consists of litigation. 2. Leverage and credit info o Leverage between the debtor and creditor: whether the creditor can pose creditable threat o Legal system can give some weight on the leverage of creditor o Creditor reporting: creditor who can make credible threat to deny access to future credit may increase their collection leverage. FICO score: the rating score (Fair Isaac Company) Protection of personal credit info: giving the debtor access to the credit report info and prescribing procedures to assure the accuracy of the info in the file. o Fair Credit Reporting Act, p.9-11 3. Bankruptcy laws o The contents Chapters 3, 5. General Rules of Bankruptcy 1

Upload: lastdinosaur1

Post on 11-Mar-2015

1.441 views

Category:

Documents


1 download

TRANSCRIPT

Page 1: Bankruptcy Outline - Pottow 2011

Outlines for BankruptcyProfessor Pottow

Chapter 1 Collection without Courts

1. Fair Debt Collection Practice Act, p.15-20o Why do we care about the debt collection method?

Efficient? If consumers aren't protected, they may be scared away from

borrowing. But if more debts are collected by extreme practices, the savings will be passed on to the borrower, encouraging him to borrow.

…o HEINTZ V. JENKINS , 1995, p.20

Debt collector: definition: The term "debt collector" means any person who uses any instrumentality of interstate commerce or the mails in any business the principal purpose of which is the collection of any debts, or who regularly collects or attempts to collect, directly or indirectly, debts owed or due or asserted to be owed or due another.

Is a lawyer a debt collector? Holding: the FDCPA applies to attorneys who “regularly” engage in consumer-debt-collection activity, even when that activity consists of litigation.

2. Leverage and credit infoo Leverage between the debtor and creditor: whether the creditor can

pose creditable threato Legal system can give some weight on the leverage of creditoro Creditor reporting: creditor who can make credible threat to deny

access to future credit may increase their collection leverage. FICO score: the rating score (Fair Isaac Company) Protection of personal credit info: giving the debtor access to the

credit report info and prescribing procedures to assure the accuracy of the info in the file.

o Fair Credit Reporting Act, p.9-113. Bankruptcy laws

o The contents Chapters 3, 5. General Rules of Bankruptcy Chapter 7. liquidations Chapter 13. Personal reorganization Chapter 11. Reorganization Chapter 15, cross-border bankruptcy

o Why to have bankruptcy law? Balance the interests of debtors and creditors

so debtors borrow without fear of social expulsion and creditors and

lenders still lend with the reliance that they will get something back

Discharge can encourage the entrepreneurs to invest risky projects?

1

Page 2: Bankruptcy Outline - Pottow 2011

Efficient? Fairness to distribute the assets pro rata?

o Why is it a federal law, not a state law?o Are the federal bankruptcy law and state collection laws the same

things?o Relationship with secured transaction

Secured transaction: specific obligation default Bankruptcy: general obligation default

4. Usury law Basically state law Dead; Banks only have to meet the state laws where they are in

and can solicit business everywhere. Delaware and South Dakota have no usury laws. Marquette, p.14

The Supreme Court changed the interpretation of usury law. The bank can charge any interest rate that is allowable in its state but may exceed the cap of the customer’s state.

Chapter 2 State Law Debt Collection

A Collection Remedies1. Introduction to Judgment Collection

o Execution After the judgment, the debt is no different from other unsecured

debtor; the only difference is that the judgment debt is indisputable and liquidated.

Start from a writ, called an execution writ or a writ of attachment The writ orders the sheriff or marshal to look for non-exempt

property of the judgment debtor, to seize it, and to pay the proceeds to the judgment creditor, until the judgment is fully paid.

Usually the sheriff takes physical possession of any property found, but he may tag it with a notice of seizure if the property cannot be physically taken.

The entire process of seizure is called a “levy” and what the sheriff does is “to levy upon” the property.

The whole process, from writ issuance to seizure, is often called an “execution”. Once the sheriff has seized the property, actually or constructively, it is said to be in the custody of the court.

After levy, the sheriff reports to the court with a document called a “return”; if he does not find the property, the return may state “nulla bona”.

o Turnover orders The debtor has to turnover the property under oath otherwise

under contempt and jail. o Other writs

Each state has some different writs, like sequestrationo Judgment liens by recordation

Only judgment cannot give a debt any special right or priority. By recording it in the proper public record, it creates a judgment lien

2

Page 3: Bankruptcy Outline - Pottow 2011

on the property. The process is often called “abstracting the judgment”.

Usually recordation only applies to the real estate, but in many states it can also be applied to personal property.

CA and Florida permit a single filing to put a legally effective lien on all the debtor’s personal property available for collection anywhere in the state.

o Debt collection by the federal gov’t Federal Debt Collection Procedures Act (§§1692(a)-1692(o)) The procedures are similar to those in this chapter; the real point

is uniformity. The act does not apply to enforcement of all judgments in federal

courts, only to judgments in favor of the federal government.o Family debts

Alimony and child support are among the most difficult debts to collect.

Some special rules for this: retaining the imprisonment; exception to the exemption of garnishment of wages.

Child Support Act of 1992o Voluntary liens

A consensual lien must follow a fairly detailed statutory scheme of agreement and recordation.

The law of security interests are covered in Article 9 of UCC Consensual lien may be “Purchase Money liens”. Those liens are

used to furnish the credit necessary for the purchase of the collateral. (PMSI = purchase money security interest).

Under Article 9, a debtor may offer a security interest in “all my equipment, current and after acquired.”

A consensual lien often can seize the collateral more quickly and cheaply than a judgment lien.

The seizure for sale – called a “foreclosure” in the case of real estate and “repossession” in the case of personalty – is heavily governed by a myriad of state rules that differ in detail but usually cover similar issues.

The Article 9 creditor can seize the property without any help from the sheriff (self-help repossession) or can offer to keep the property in satisfaction of the debt without any sale (retain in satisfaction). Both options have some restrictions, including a requirement that self-help repossession not involve a breach of the peace.

Deficiency judgmento Statutory liens and trust funds

Statutory liens, like the landlord’s lien, artisan’s lien, and construction lien. Usually those are created by state laws.

The trust fund statutes: the debtor is the trustee and the creditor is the beneficiary for certain debts.

o Property exempt from the collection process Exemption: typically the family home and household goods

o Collection in other jurisdictions

3

Page 4: Bankruptcy Outline - Pottow 2011

The rendering jurisdiction (where the judgment is obtained) and the enforcing jurisdiction (where the property is).

Common law method. Uniform Enforcement of Foreign Judgments Act: a registration

system of enforcement.2. The struggle among creditors: Prioritya. Background

o A race of diligence: First in time, first in right.o Two unsecured creditors: the levy perfects the judgment lien on the

property. The first to levy will win as to the property levied on.o Complicated example: A’s writ was delivered to the sheriff on

Monday, B’s writ was delivered on Tuesday, and the sheriff levied under both writs on Wednesday. Who got what? It is not always true that it is a tie just because the two writs were levied at the same time. The date of levy is not always the controlling date for determining priority. In many states, A still wins because its perfection is backdated to the date the sheriff got the writ. Some states use different rules, like the date of judgment or date of the sheriff’s levy.

o Unsecured judgment creditor v. secured creditor: First to perfect wins. The secured creditor perfects when it records its consensual lien

according to the statutory prescription. PMSI often involves a 20-day grace period during which the

secured creditor may record its lien and beat any intervening unsecured creditors with involuntary liens.

b. Competing unsecured creditorso First in time, first in right. o If two creditors enjoy equal priority, a taxing authority usually turns

out to be more equal than the other creditor.o Inactive judgments may lose their vitality.

c. The aging judgmento Dormancy: not seeking the enforcement within one yearo Limitations: expiration of the statute of limitations to enforce a

judgment; terminal; usually ten yearsd. Execution

o What must the sheriff do to complete a priority-conferring levy?o Credit Bureau of Broken Bow v. Moninger (Neb. 1979), p.47

Facts: 10/27/77: Bureau got default judgment against Moninger. 5/16/78: Moninger renewed his prior note to the Bank, w/ the

renewed note to be secured by a security agreement on feeder pigs and Moninger's Ford pickup. No security agreement was entered into at that time.

6/27/1978: At Bureau's request, a writ of execution was issued on its judgment for the balance remaining due on the judgment.

7/7/78: Sheriff who received the writ examined the motor vehicle title records and found no lien on the pickup. The sheriff went to where Moninger worked to levy on the vehicle. When the Sheriff told Moninger he was executing on the pickup, Moninger said that the Bank had title to the vehicle. The sheriff walked over,

4

Page 5: Bankruptcy Outline - Pottow 2011

"grabbed ahold of the pickup," and stated: "I execute on the pickup for the County of Custer." The sheriff left w/o taking possession of the truck or asking for its keys.

7/10/78: Bank and Moninger executed a security agreement on the vehicle which was immediately filed and notated.

7/13/78 : Deputy sheriffs seized the vehicle, which was sold at the sheriff's sale on 8/14/78.

Issue: Was the Bureau a lien creditor on 7/7/78? If the bank is a secured creditor before the purported levy,

the bank will get the money. If the bank is not a secured creditor, then the facts that it became a secured creditor mere days after the sheriff levied upon the property makes no difference; the judgment creditor gets the money.

At best, late recordation makes the bank a second secured creditor, second in priority after the first lender has been paid in full.

Bank: "There wasn't a properly executed levy before we recorded our security agreement. The pickup had to be physically seized to make the levy valid."

Holding: A valid levy occurred before the Bank had perfected its security interest in the truck. The Bureau was a lien creditor on July 7.

A lien on personal property is acquired in NE at the time it is "seized in execution" - requires only "that the property is present and subject for the time to the control of the officer holding the wriit, and that he in express terms asserts his domination over it by virtue of such writ."

Notes: The requirement that the lien creditor have no knowledge of the security interest was eliminated in the 1972 UCC Amendments. This court reached the same result under the 1962 Code by holding that the sheriff's knowledge is not appropriately imputed to the judgment creditor.

3. Discoveryo Get info about the existence, location, and exemption status of

leviable property.

4. Garnishment WEBB V. ERICKSON (AZ 1982), p.55

o Judgment creditor is superior because secured creditor did not perfect before truck was levied upon.

Garnishing creditor’s temporal net: o The time between service of the garnishment writ and the

garnishee’s answer, during which the creditor can hope to catch obligations arising in favor of the creditor.

o The garnishment of a bank account will catch not only the amount on deposit on the service date but also funds deposited thereafter prior to the answer.

o If a garnishment is contested and a trial is to be held, the creditor’s net may even extend to the time of the garnishment trial.

5

Page 6: Bankruptcy Outline - Pottow 2011

o Service: the official delivery of legal documents ("service of process") such as a summons, subpoena, complaint, order to show cause (order to appear to show reasons why a judge should not make a particular order), writ (court order), or notice to quit the premises, as well as delivery by mail or in person of documents to opposing attorneys or parties, such as answers, motions, points and authorities, demands and responses.

What is the garnishment different from the execution?o Writ of Garnishment is different from Writ of Execution in that the

Execution is directed at the debtor; the Garnishment is directed at the debtor of the debtor. He is the creditor’s friend.

o Same: legally enforceable documentRestriction on wage garnishment

o Generally the federal restricts the garnishment of wages.Garnish the contractual rights

o The property in bankruptcy includes the valuable contractual rightso Some contractual rights can be garnished; some not. o NETWORK SOLUTIONS V. UMBRO INTERNATIONAL , 2000, p.69

Holding: A contractual right to use the internet domain name is not garnishable because of the privity of the contract. A K for services is not a liability and hence is not subject to a garnishment.

 Problem Set 3, p.77

o 3.1 A: Amos State Bank can offset the $10 firstly. The first writ was issued on Feb.1, the second on Feb.7, and the

sheriff served both on Feb. 9 There is a temporal net of 14 days for the garnishment. Is the net

for the benefit of the garnishee to get their stuff together? Or maybe for the benefit of the Judgment Creditor to get as much money out of the account as possible. Pottow: the latter.

14 day net starts on service. Can take all the money that comes into the account during those 14 days.

Can bank allow money to be withdrawn from the account? Is it on notice and has to freeze the account.

Can JD get notice of the garnishment? Which one has priority? JC 1 or JC 2? It depends on the state laws.

The date of levy is not always the controlling date for determining priority. In many states, JC1 still wins because its perfection is backdated to the date the sheriff got the writ. Some states use different rules, like the date of judgment or date of the sheriff’s levy.

o 3.1 B: The bank is responsible for the $500 check paid to the telephone

company on Feb.9, though another $500 (200+300) comes back to the account. Because the bank disobeyed the order of garnishment. From WEBB V. ERICKSON , the garnishee may be responsible for the entire judgment, because he totally ignored the order, not just a mistake.

$300 of wages is still available for the collection.

6

Page 7: Bankruptcy Outline - Pottow 2011

o 3.2: Fraudulent conveyance to avoid garnishing of assets. You can

garnish the lease payments and get the assets when it is done even if you can't get the actually property.

If it is a fraudulent lease then the lessor is just the owner and can be garnished directly.

What about the fraud? If it is a fraud, who will get the equipment? o 3.3:

 Can’t fire for writ of garnishment, but can after the 2nd time! 15 U.S.C.A. § 1674 the Consumer Credit Protection Act

§ 1674. Restriction on discharge from employment by reason of garnishment

(a) Termination of employmentNo employer may discharge any employee by reason of the fact that his earnings have been subjected to garnishment for any ONE indebtedness.(b) PenaltiesWhoever willfully violates subsection (a) of this section shall be fined not more than $1,000, or imprisoned not more than one year, or both.

5. Prejudgment remediesa. Requirements for the prejudgment attachment

o State law A showing of need A bond

o Procedure requirement by Supreme Court: due process An order issued by a judicial officer (not a clerk) upon a factual

showing of need The judgment debtor should be offered a hearing or a chance to

get the property back very quickly after the attachmentb. SCOTUS also restricts a preliminary injunction freezing the assets of a defendant pending trial.

B. Fraudulent Conveyances and Shielding Debtor Assets1. Origin of Fraudulent conveyance lawo TWYNE’S CASE :

The debtor cannot give up all his property to one creditor other than the rest creditors when he was insolvent, because of unfairness.

The creditors have the burden to prove the actual fraud with clear and convincing evidence. Actual fraud is usually found by the list of references in the §4 b.

2. UFTA1) §2 Insolvency o (a) A debtor is insolvent if the sum of the debtor’s debts is greater

than all of the debtor’s assets, at a fair valuation.o (b) A debtor who is generally not paying his debts as they become due

is presumed to be insolvent.

7

Page 8: Bankruptcy Outline - Pottow 2011

o (d) Assets under this section do not include property that has been transferred, concealed, or removed with intent to hinder, delay, or defraud creditors or that has been transferred in a manner making the transfer voidable under this act.

o (e) Debts under this section do not include an obligation to the extent it is secured by a valid lien on property of the debtor not included as an asset.

2). § 4 Transfers fraudulent as to present and future creditors: Hard fraud:

(a)(1) try to hide assets: actual intent to hinder, delay, or defraud any creditor(a)(2) without receiving a reasonably equivalent value and

Undercapitalization. §4(a)(2)(i). Intended to incur or should reasonably have expected he would

incur debts beyond his ability to pay them as they became due. §4(a)(2)(ii).

There are unreasonably small business assets, and you reasonably should have known you would incur serious debts.

(b) Badges of fraud.3). § 5 Transfers fraudulent as to only present creditors

o Soft fraud or constructive fraud(a) Debtor was insolvent at the time or the debtor became insolvent as a result of the transaction. You can't give away your property when you are insolvent.

Rationale: when the debtor is solvent, it causes no harm to creditors; but if he is insolvent, it does cause the harm.

Two factors: 1) asset transferred in exchange for less than reasonably equivalent value; 2) debtor was insolvent.

Strict liability: Can have fraud without scienter, malicious intent. Some have created the concept of "constructive fraud" in which someone unintentionally deceives the creditor.

This is a serious infringement on personal property rights. Your property is no longer freely alienable.

4). §7 Remedies of Creditors(a) Subject to §8, a creditor may obtain

(1) Avoidance of the transfer or obligation to the extent necessary to satisfy the creditor’s claim;(2) An attachment or other provisional remedy against the asset transferred or other property of the transferee(3) Subject to the equity and civil procedure,

(i) An injunction against further disposition by the debtor or a transferee, or both, of the asset transferred or of other property;(ii) Appointment of a receiver to take charge of the asset transferred or of other property of the transferee; or(iii) Any other relief the circumstances may require.

(b) If a creditor has obtained a judgment and the court so orders, the creditor may levy execution on the asset transferred or its proceeds.

5). §8 Defenses, liability, and protection of transferee

8

Page 9: Bankruptcy Outline - Pottow 2011

(a) A transfer or obligation is not voidable under §4(a)(1) against a person who took in good faith and for a reasonably equivalent value or against any subsequent transferee or obligee.(b) to the extent a transfer is voidable under §7(a)(1), the creditor may recover judgment for the value of the asset transferred, as adjusted under subsection (c), or the amount necessary to satisfy the claim, whichever is less. The judgment may be entered against:

(1) The first transferee of the asset or the person for whose benefit the transfer was made; or (2) Any subsequent transferee other than a good faith transferee or obligee who took for value or from any subsequent transferee or obligee

(c) The value of the asset transferred under (b) must be the value of the asset at the time of the transfer;(d) Notwithstanding voidability of a transfer or an obligation, a good-faith transferee or obligee is entitled, to the extent of the value they paid, to

(1) A lien on or a right to retain any interest in the asset transferred;

(2) Enforcement of any obligation incurred; or(3) A reduction in the amount of the liability on the judgment.

(e) A transfer is not voidable under §4(a)(2) or §5 if the transfer results from:

(1) Termination of a lease upon the debtor’s default(2) Enforcement of a security interest under Article 9 of UCC

(f) A transfer is not voidable under §5(b):(1) …

3. Problem Set 4, p.91o 4.1

May be fraudulent under §5(a). Okay under §4(a). Key question: is this a reasonable equivalent value? Under §8(d), the transferee can get a lien.

o 4.2 This would be fraud under §4(a)(2)(ii). Bonney knew that she

would incur a debt beyond her ability to pay when it is due. American Express as an after transaction creditor can make such a

claim under §4. Does the cousin have to know O’Hare’s situation to claim UFTA?

Not necessary from the statute. It can also claim §4(a)(1) combining the proofs of §4(b)

o 4.3 The debtor can give the home away. It was exempt. The creditor

cannot reach it. o 4.4

Creditor can avoid the transfer to the extent necessary to satisfy his claim under §7(a)(1). Here Lo-Cost’s claim is more than the property’s value, so it can avoid the entire transaction. It probably can get back the property itself.

9

Page 10: Bankruptcy Outline - Pottow 2011

But after the avoidance of the transaction, what kind of remedy can Lo-Cost recover? A monetary remedy or a property remedy? Lo-Cost can recover a monetary remedy under §8(b) or a property remedy under §7(a)(1). Is there any preference for the property remedy over the monetary remedy? If it is a monetary remedy, Lo-Cost can recover 6K from Sam. If it is a property remedy, Lo-Cost can keep the property that appreciated 20%. Generally the creditor has the right to choose the form of remedy and he will get the property back.

But §7 is subject to the limitation of §8. Under §8(a), good faith purchaser is protected and the transaction is not voidable under §4(a)(1). But under §8(b)(1), the good faith of the purchaser is not relevant.

Under §8(d), despite the voidability of a transaction, a good faith purchaser gets a lien for his interest in the amount given to the debtor. Here the lien is $30,000. But can Sam recover the 5K for the repair? Does this unjustly enrich the creditor? He probably can get the $5000 repair cost under §10.

o 4.7 Is this a contract? Looks like it. Did the buyer breach by not

selling it back? Is this contract voidable? No creditor has tried to void it. Might we let the conveyee keep the horse to discourage people

from these fraudulent conveyances? 4. LBO

IN RE BAY PLASTICS , p.87Facts: Bay Plastics borrowed $3.95 million in the security of all its assets from the Bank. BP loaned this money to BPI and BPI bought all the shares of BP from its shareholders. BPI (Milhous) puts in no money. Eventually the shareholders of BP got the money. The Bank, BPI, and the shareholders of BP are benefited by this transaction. Who is hurt? BP and its creditors! The entire transaction is a fraudulent transaction under UFTA §5. BP did not receive reasonably equivalent value for the loan obligation and security interest that it granted to BT (the bank) from the perspective of the whole transaction. Analysis: This transaction puts Bay Plastics into insolvency, to the detriment of its other creditors. Pottow says this is like an improper dividend case. The stockholders are getting paid for their shares and taking away funds from the company that the creditors had a claim to.

Chapter 4 Elements Common to Consumer Bankruptcies

A. Background

B. Outline1. Two proceedings: liquidation and payout plans2. Chapter 7 liquidation:

o Result: the debtor receives a discharge of preexisting debts.

10

Page 11: Bankruptcy Outline - Pottow 2011

o Two objectives: fair distribution to creditors and a “fresh start” for debtors.

C. Getting Started

1. Start with filing a petitiono A basic request for bankruptcy reliefo With some certificationso Schedules of financial infoo Filing fee: $220; the people with earnings lower than 150% of the

poverty line can request a fee waiver2. The debtor’s attorney or his clerk takes the filing to the bankruptcy

clerk’s office, where a clerk will take the filing fee and stamp the filing with time.o Upon stamp, a bankruptcy estate is created and an automatic stay is

in place3. The info needed for the filing could be substantial. 4. The debtor’s attorney has a duty to verify the accuracy of the debtor’s

records as required in the amendment 2005.

D. The Estate1. Property of the EstateThe nature of the estate

o The estate is similar to a trust. Hence trustee in bankruptcy. A trust is not a legal entity. It is just the trustee and the beneficiary. The bankruptcy estate, therefore, can only act through its trustee in bankruptcy.

o Beneficiaries of the bankruptcy trust: Creditors (does this include future creditors? We will come back

later.) Debtor

o US bankruptcy law does not require the insolvency for the bankruptcy filing.

o Format of the case citing: For the litigations: Caminker v. Commissioner (In re Pottow) For the petitions: In re Pottow

Code §541 Property of estate, p.116o Inclusion: All interests of value to the estate, wherever located and by

whom held.o Exceptions:

“Services performed by an individual debtor after the commencement of the case, including wages, commissions…” §514(6).

o Who are the beneficiaries for this estate? All creditors including secured and unsecured creditors, and the debtors. Question: Do the creditors include the future creditors?

SHARP V. DERY , (E.D. Mich. 2000), p.117

11

Page 12: Bankruptcy Outline - Pottow 2011

o Facts: Debtor filed a Chapter 7 petition on Dec 21, 1998. On Feb.22, 1999, he received a bonus check from his employer for $11,331.63.

o Issues: Was the post-petition bonus the property of estate? Did Debtor have an enforceable right to receive the bonus check when he filed his petition?

o Procedural History: The bankruptcy court held that it was the property of estate, because the employer had no discretion as to the amount and timing of any bonus, since the bonus was based on a percentage of the employer’s salary. This court thought that though the employer had no discretion on amount and timing, it could decide not to give any bonus.

o Holding: No, the Debtor had no enforceable right to the bonus when he filed a petition. The employer, as of the date the debtor filed for bankruptcy, could have decided not to pay any bonus at all under the terms of the bonus plan itself.

o How to argue the future bonus is the property of estate? It is for the service rendered. But the courts may think the debtor has not been vested these benefits.

o Principle: When post-petition income is dependent upon the continued services of the debtor subsequent to the petition, the amounts do not constitute property o the estate.

Disputes about the inclusion of certain expectancies in “property of the estate” under §541 fall into 3 main categories:

1. Legal interests that are not enforceable at the date of bankruptcy but may be enforceable at a future time. Question is whether they are sufficiently matured and certain to be included in the estate. SHARP V. DERY .

2. Entitlements, such as permits or licenses that are nontransferable, which may or may not be property.

3. Restrictions on transferability imposed by K or by law.§541(c)(1) makes most restrictions on alienability unenforceable. Exception found in §541(c)(2), the spendthrift trust exception. – debtors often able to keep retirement accounts out of their bankruptcy estates because such accounts are often set up as spendthrift trust.

IN RE ORKIN , (D. Mass. 1994), p.122o §541(c)(2): validates restrictions upon transfer of a debtor’s

beneficial interest in a rust that are enforceable under “applicable nonbankruptcy law: by excluding property from the estate.

o “Applicable non-bankruptcy law” includes both state spendthrift law and federal law including ERISA (PATTERSON V. SHUMATE , 1992).

o Court was disturbed that Orkin was his own employee. Therefore not ERISA qualified.

o How is this different from an IRA though? You can't withdrawal from an IRA whenever you want. There is a 10% penalty tax. IRA accounts deemed not part of the debtor's property, since they

can't take money out of it without a penalty.

12

Page 13: Bankruptcy Outline - Pottow 2011

What about Pensions? CDs?

2005 Amendment o §541(b)(7) Expanded the protection on the retirement and other

savings accountso The creditors shall not reach the children’s educational trust fundso More protection on retirement by the S. Ct. IRA is exempted even

though it is not in the laundry list of exempt retirement plans Under the S.Ct., IRA with anti-alienation is not property of the

estate. But saving account is the property of the estate. What does the Congress try to protect IRA? What is the policy behind this?

IN RE BURGESS , (Bankr. D. NV 1999), p.125o Under §362, whether the debtor was entitled to a reversal of the

revocation of his brothel license and damages depended upon whether the license was property of the debtor.

o Holding: This court reversed the trial court’s decision that the brothel license is not property, but rather a “personal privilege.” Brothel license is property.

o Reason: It does not matter for the state law to name a right as “privilege.”

The dispositive standard is it is treated as property for purpose of the federal bankruptcy law. It seems whether something is property is under de novo consideration for the bankruptcy courts, the state right seems irrelevant.

In this case, the brothel license has enormous value for the estate-otherwise no business left to reorganize under chapter 11.

If it is not the property, it will be against the goal of the Congress to encourage the reorganization.

o §541(c). If the state law says that it is a property and is not transferable. Does the state law preclude the transfer of the property from the debtor to the estate? NO.

o Underlying Sticks Defined by State Law Pottow's take: State law defines the outlines of property. Whether

they call it property or not is not determinative. However, if the state grants licenses for this, allows it to be transferable, etc., those are factors that the bankruptcy court considers. State law defines the bundle of sticks of the property contention.

2. The TrusteeTrustee in Bankruptcy: TIBTIB’s duties:

o Gather the propertyo Protect and maintain ito Sell the property for the highest possible priceo Distribute the proceedso Scrutinize claimso Challenge any improper exemption claims

13

Page 14: Bankruptcy Outline - Pottow 2011

o Investigate the debtor’s affairso Has a special obligation to general creditors

Proposition that equity is equality To enlarge the estate as big as possible

Appointment of TIBo Elected by the creditors; very fewo Appointed by US Trustee, UST

Compensation for TIBo Percentage of the estateo For non-asset cases, Congress pays $60-$220 per case: statutory fees

USTo A government official to serve various administrating and monitoring

functions in the bankruptcy courts of a particular regiono 21 USTs in allo Functions: p. 135 last Para

Problem Set 5, p.1365.1A. Yes; offspring is also property; §541(a)(6).B. Yes;

o Other than the human beings, all other things are property. We will exempt other things based on the public policy.

o What is the value of this car? Let’s suppose it is $4990, and is secured for $5K. It is still the property of estate, because it is property.

o Because the debtor has zero equity in it, there is no value to the estate.

C. No. o Snapshots don't have real value, only an extortionist value and may

degrade the people that are in the pictures.D. Yes.E. Yes, but are exempt under §522.F. Yes; dividends are proceeds under §541(a)(6).

o Does it depend on that his uncle left to him? Spendthrift trust is an exemption; it cannot alienate. If the stock is the spendthrift trust, how about the dividends?

o There is a controversy whether the dividends of the spendthrift trust are part of the estate. If they are, does this undermine the importance that spendthrift trust is not a part of the estate?

G. Yes.H. Yes.I. It depends.

o On the state property laws under which who owns the glove? If it is the debtor’s glove, no matter who possesses it, it is still the property of the estate.

J. No. Account for which he acts as the trustee is not part of the estate. §541(b)(1).K. Yes.

14

Page 15: Bankruptcy Outline - Pottow 2011

o Salary for month prior is part of the state regardless of when check was received

L. Retirement income not part of the estate. §541(b)(7).M. The wage after the filing is not the property and his employer’s contribution to his IRA is also not.

5.2o The debtor got the money. Post-petition earnings

5.3o March 1 sold; April 1 filed for bankruptcy. On March 1: $10; April 1:

$15; May 1: $20; $150,000 belongs to the estate, and the other $50K belongs to Frances???

o Farmer’s contract is part of the estate. The trust has an equitable and legal right to enforce the contract.

o But what if Frances refuses to harvest the wheat since she doesn't own the wheat? She was under contractual obligation. But the bankruptcy estate now owns the contract. Would the trustee hire Frances to harvest it?

5.4 — Spendthrift trust provision. Trust is protected from creditors.

5.5 — The professor got the award; it is earned after the filing. But the TIB can argue that this is product of the pre-filing work, so it should be the property of the estate.

5.6 — Yes. The license has enormous value to the estate, like the brothel license case.

E. The Automatic Stay

§362o §362(a) details the prohibitions of the stay; §362(b) provides

exceptions that permit certain types of actions against the debtor to continue, like the criminal proceedings; exceptions continue to be expanded.

o It also applies against the debtors as well as the creditors. ANDREWS UNIVERSITY V. MERCHANT , 1992, p.138

o Facts: The automatic stay prohibits all kinds of the debt collection behaviors. The court held that the University’s refusal to issue a transcript until a debtor pays a pre-petition debt is an act to collect the debt and violates of §362(a)(6).

o Reasoning: The transcript is not really property, but the act is aimed at coercing her into paying her debt to the university.

o The educational loan is not discharged after the consumer bankruptcy. §523(a)(8).

NISSAN MOTOR ACCEPTANCE CORP V. BAKER, 1999, p.140

15

Page 16: Bankruptcy Outline - Pottow 2011

o Issue: Is a secured creditor is required to turn over its collateral which is property of the estate without adequate protection? Yes.

o Nissan’s self-help possession and sale of the car are willful violation of the stay. So it paid punitive damages and actual damages to the debtors.

Another Pothole on the way to the automatic stayo The stay is automatic. The creditors rarely got in trouble for violation

of a stay that they did not know though they have to give back the property or proceeds.

o Notice of address §342(e)(g) effective upon the arrival to the specific person and division

o The risks associated with delays in notification fall squarely on the debtors.

Preliminary Procedureso Schedules include lists of debts, assets, income and expenseso It is crucial that the schedules are accurate and complete. Otherwise,

the debt may be not dischargeable. §523(a)(3) If they are false, it may result in a complete denial for the discharge as to all the debts and may open the debtor to a perjury prosecution. 727(a)(4)

o They also have to file copies of their pay stubs for the two months before the filing, a statement of their income and the possibility that the income will increase in the next 12 months; also file a tax return for the prior year and a copy must be sent to any creditors if they request it

o Random audito For the debtor’s counsel, reasonable investigation and sign the

debtor’s petitiono First meeting of creditors: section 341 meeting

Problem Set 6, p.145o 6.1

Joe is getting paid tomorrow. If he files for chapter 7 now the money still goes to the estate if he is paid in arrears, they are already earned.

Garnishment is lifted. The whole amount of money should go to the estate. §362(a)(6) — this is an act to collect. 

What about the over-drawn check? Criminal actions are not stayed. §362(b)(1). 

Can't turn off utilities basically, but under 366 utilities can ask for a deposit. 

§362(b)(2)(C): Domestic support obligation is not stayed.  So far as the last month's pay check goes, it doesn't matter if you

file before or after you receive it b/c the work is already done. But if you get the paycheck before you can spend it all now! Or buy exempt property.

See §§362(a), (b)(2), (b)(22), (b)(23); 366.o 6.2

16

Page 17: Bankruptcy Outline - Pottow 2011

o Puja arrives in your office in tears. She shows you a notice that her house has been posted for sale by foreclosure at non tomorrow. She explains that she is in the middle of a divorce, and that her ex-husband lied about making mortgage payments. She called him about earlier notices, but he said they were “mistakes: that he would straighten out. For the past month she hasn’t been able to find him, fears he has fled the state. She tried calling mortgage company, but her loan had been sold from one servicer to another and she could never tack down the noteholder. Now the sheriff’s dep’t says the sale is going forward.

o Puja has filed out all of the schedules in your office today, but she didn’t know to bring paychecks stubs, tax returns, or any other paper work w/ her, nor has she sought credit counseling. She lives an hour away and doesn’t know where all of the paper work is at home. What can you do for her today? Make a list of what you need and how you can get it. See 11 USC §§521(a), b), (i); 101(12A); 109(h); 526; 707(b)(4)(C); (D). When your paralegal points to the car price she listed ($250), does that raise an issue beyond asking her if she is sure of it?

Have to watch out for the $250 valuation of the car. Attorney has to watch this under §707(b) (4)(D).

§521(i) you have 45 days to file your case. But how to read this article: it is correctable in 45 days; or you cannot file the petition without such materials 

§109(h) — You can't be a debtor without getting credit counseling. Has to be within 180 days before filing. You can get a waiver of this §109(h)(3), but it is hard to meet three conditions and not likely to work here.

§521(b) — Submit the certificate from the credit counseling and the debt repayment plan; otherwise, ineligible to file for bankruptcy.

o 6.3 He took the car before he filed bankruptcy. He should still return

the car to the bankruptcy estate. §362(a)(3). This is an act to obtain control of property of the estate. Repossession does not make the creditor the owner of the property; it is still the debtor's property and part of the estate.

§362(a)(h); 521(a)(2).o 6.4

How can I know whom I should send to? Designated person. There should be a public record or posting in the website about the

key creditors. Even if you cannot figure out who is the designated person, you

can still argue the theory of agency, like you can assume the corporate lawyer or a higher official is the agency of the company.

Chapter 5 Liquidation Bankruptcy

17

Page 18: Bankruptcy Outline - Pottow 2011

A. Introductiono Distribution process §726.o Priority creditors, general creditors and subordinated creditors.o Exemptions will determine what property of the estate is exempt from

sale for the benefit of the creditors and reserved for the debtor’s fresh start.

B. Eligibility1. A change in philosophy

o A change of philosophy: Encourage people to repay the debts under Chapter 13. It works and the filing under Chapter 13 rises to 30% of all filings.

o But one third of Chapter 13 filings cannot complete their repayment plan and convert to Chapter 7 or drop out of the bankruptcy. The proportion of the Chapter 13 filing stabilized at about 30% but the total filings rose dramatically.

o In 1984, Congress gave the judges power to dismiss the Chapter 7 cases if the filing involved “substantial abuse”.

IN RE SHAW , 2003, p.153o Key issue: how the court reshapes the debtor’s budget and requires a

minimum three-year repayment plan if the debtors want any bankruptcy relief.

o The test for the substantial abuse is “totality of circumstances”: whether the debtor has the ability to repay the debt, including

consideration of the relation of the debtor’s future income and the future expense

whether the debtor files because of sudden illness, disability or unemployment

whether the debtor’s schedules and statements of current income and expenses reasonably and accurately reflect his true financial condition

whether the debtor incurred cash advances and made consumer purchases in excess of his ability to repay

whether the debtor’s proposed family budget is excessive or unreasonable

whether the petition is filed in good faitho Holding: it is a substantial abuseo IN RE SHAW - pre means test law

Shaws were advised to move out of their house. Isn't this exempt? Would be if they could get into bankruptcy!

2. The presumption of abuseo Substantial abuse is only for the consumer debtors. For someone

whose debts are mostly business debts there is no screening for abuse. §707(b)(1).

o §707(a) may still provide a creative court with some chance to review a business petition.

o 2005 Amendment new scheme:

18

Page 19: Bankruptcy Outline - Pottow 2011

§707(b)(1) — The court may convert it to a repayment plan in Chapter 13 or Chapter 11 if the Chapter 7 filing is an abuse.

§707(b)(2)(A) — Creates a presumption of abuse based on a formula of income minus expenses. The debtors must supply extensive documents to support the

income and expenses. No more balancing test like in IN RE SHAW .

§707(b)(2)(B) — Special circumstances such as serious medical conditions or service in the armed forces, may justify adjustments to the calculations to determine which debtors are presumed to have abused the bankruptcy system. Special circumstances must have an effect on income or

expenses that can be quantified and documented. §707(b)(3) — The court may recognize an abuse based on the bad

faith and totality of the circumstances.

3. The Means testo  Only applicable to people above the median state incomeo Then a complicated expenses testo A multiple to see if you pass the means test after your expenses

a. Income o A debtor’s current monthly income and a threshold test for Chapter 7

eligibilityo The threshold test is whether the debtor’s income exceeds the median

income for similar families in the state where he filed. If lower, no presumption of the abuse bars the way to Chapter 7; if higher, more calculations await. §707 (b)(2)(A). It is a median income, not a mean (average) income.

o Monthly income §101 (10A). The court calculates the average monthly income for six months preceding the bankruptcy filing. It includes wages, interest, stock dividends, unemployment

compensation, income tax refunds, business revenue and accounts receivable.

o Median income depends on the Census Bureau, but the Bankruptcy code and Census use the different definitions of income. The code uses the post-tax income; but Census uses the pre-tax Census does not include the capital gains, etc., but the code

includes all The code excludes the Social Security benefits while Census

includes it.o Census Bureau collects data on median incomes by state and by

family size. Census does not collect data annually, but it offers estimates.

Whether such estimates meet the statutory muster is an open question

Consumer price index but it is based on expense not on income

b. Expense

19

Page 20: Bankruptcy Outline - Pottow 2011

o IRS: National Standards, which use a sliding scale for various expenses based on income, based on their family size and income level The bankruptcy code permits debtors to deduct these amounts

without further proof. §707(b)(2)(A)(ii).o 2005 Amendment provides that the court may increase the allowance

for food and clothing by up to 5% if the debtor can demonstrate that such expenditures are reasonable and necessary. §707(b)(2)(A)(ii) for the excess over 5% and any amounts in categories other than food and clothing, the code seems to leave the courts with no discretion to consider how families actually spend their money.

o Other deductions, p.163 Health insurance, disability insurance and health savings accounts Expenses for caring for the elderly Private schools up to $1500 per child per year IRS actual expenses for childcare, legal fees, life insurance, union

dues, taxes, and several other items Any expense to pay arrearages on priority debts, like alimony,

child support and taxes

c. Secured Debtso Car loans can be deducted in full, no matter how large, along with

any payment arrearages. §707 (b)(2)(A)(iii). The gasoline, insurance, and maintenance follow the local

standards in the IRS tables. §707 (b)(2)(A)(ii)(I).o The debtors can deduct the payment to the mortgage lender,

whatever it may be. §707 (b)(2)(A)(iii). Not clear about the deduction of utilities and maintenance of the

house

d. Income after expenseso To do calculation, it is to know (a) the total size of the surplus of

income over expenses over 60 months; and (b) how much general unsecured non-priority debt

o Abuse is presumed:(1) If the debt is greater than $24K, and

(a) The surplus is at least $10K or 25% of the debt; or(2) If the debt is $24K or less, and the surplus is at least $6K.

o Another way to express: If the surplus is less than $100, the debtor passes If the surplus is between $100 and $166.66, he passes if the

surplus is less than 25% of his unsecured debt divided by 60. If the surplus is greater than $166.66, he flunks no matter how

much he owes.

4. Policing the new standardsa. Procedure

o §521 requires the debtors to calculate the means test

20

Page 21: Bankruptcy Outline - Pottow 2011

o §704(b)(1): UST’s office is required to look at every case to see if the presumption of abuse is triggered and to file a statement reporting its finding that is sent to every creditor. For the debtors above median income, the office must file a

motion to dismiss or convert it if presumptive abuse is present or file a statement explaining why it has not done so.

o There are limitations on the standing to raise the issue of abuse in 2005 Amendment. Abuse can be alleged against an above-median debtor by the

judge, UST’s office and any creditor. A creditor cannot raise the abuse issue against a median–or-below

debtor under either §707(b)(1) general abuse or §707(b)(2) means test. Only a judge or UST’s office can bring a claim of general abuse. §707(b)(6).

Those officials cannot raise abuse under the means test under §707(b)(2)(A)(i) if the debtor’s income is equal to or below the state median.

o A spouse’s income: if only one party files, the couple’s income will be used to determine whether the debtor is above or below the median income; if the debtor fails the test, only the debtor’s income, not including the spouse’s income will be used for the means test.

b. Many roles for debtors’ lawyerso 2005 Amendment requires the lawyers to do some investigations of

the accuracy of the data. §707(b)(4)(C)(D).o §526(a)(4) and §707(b)(4)(A) impose the duties on the attorney

regarding the advice to decide whether to file for Chapter 7.o Rule 9011

Problem Set 7, p.167

7.1

o See separate page

o §101 (10A): Average monthly income in the preceding 6 months.o §707 (b) (6) only the judge or UST may file a motion under §707(b),

if the current monthly income of the debtor is equal to or less than in the case a debtor in a household of 2, 3 or 4 individuals, the highest median family income of the applicable state for a family of the same number of fewer individuals. (General Abuse).

o §707(b)(7) — No judge, UST TIB or other parties in interest may file a motion to dismiss under §707(b)(2) if the current monthly income of the debtor is equal to or less than in the case a debtor in a household of 2, 3 or 4 individuals, the highest median family income of the applicable state for a family of the same number of fewer individuals. (The “Means” Test).

7.2

21

Page 22: Bankruptcy Outline - Pottow 2011

o See separate page

7.3o He is slightly above the median. So he has to go through the means

test. His surplus is $150, higher than $100 and lower than $166.66, so the test is whether the surplus is less than 25% of his unsecured debt divided by 60 about 30000/60*25%=$125. So his surplus is higher than $125, and he fails the means test. He is presumed to abuse the bankruptcy system.

o It seems like he would want to wait so his income goes below the median income. §526(a)(4).

o He could buy a car with financing expenses to get more valid expenses and make his disposable income so he meets the means test.

o §526(a)(4) — A debt relief agency should not advice the debtor to incur ore debt in contemplation of such person filing a case under this title. A lawyer should be a debt relief agency

o However, is it constitutional to limit the advice a lawyer can give? What if we advise him to give more to charity or get married? Use more money on food? Can the attorney advise the debtor to spend more expense?

o §707(b)(2)(B) — The presumption of abuse may only be rebutted by demonstrating special circumstances, such as a serious medical condition…; here is his leg injury a special circumstances?

o §101 (12A) — Definition of “debt relief agency”

7.4o He is eligible for Chapter 7.o Why does he pass the means test? It is not the consumer debt; it is

business debtMeans test only applies to the consumer debts. It seems hard to deny him Chapter 7 relief under §707(b).

o But the judge has the discretion to deny the filing based on a totality of circumstances test for general abuse even though the debtor can pass the means test or the means test does not apply under §707(a). Do we like a totality of the circumstances test? Do we trust judges to decide who should be denied? Do we need this objective test?

7.5o §707(a)(3) — File the info of paragraph (1) of §521 within 15 days

after the filing the petition.o §707(b)(4)(C)(D) — The attorney made the reasonable investigation

and signed the petition with good faith that the did not know any incorrectness.

o §521 (a)(1) — The list of finance info of the debtor that he should file.o You cannot skip this info or estimate it; otherwise the attorney is

liable for any error without reasonable investigation.

C. Property Exempt from Seizure 1. Introduction

22

Page 23: Bankruptcy Outline - Pottow 2011

o Exemptions for two reasons A fresh start Personal items have little resale value but crucial to the debtors.

o Consensual security agreements are not constrained.2. A state/federal system

o The federal bankruptcy law would establish uniform deferral exemptions, but states would be permitted to opt-out of those exemptions, denying their own citizens the benefits of the federal protection when they filed for bankruptcy.

o 35 states have opted out.o Texas/Delaware Exemption Statuteso The federal exemptions

Covers a wide variety of property, including payments from crime reparations laws and unmatured life insurance. §522 (d).

2005 Amendment expended to retirement funds regardless of whether the state have opted out §§522(b)(3)(C) , (d)(12).

The specific dollar amount limits are adjusted every three years.3. Classification of propertyIN RE JOHNSON , 1981, p.179

o The debtor may claim the exemptions that are creatures of state law, but the federal court will be called on to determine their meaning.

o Holding: A bus is a motor vehicle.IN RE PIZZI , 1993, p.180

o Facts: Lottery annuity. She used her future winnings as security for loans. Then became insolvent and wants to go bankrupt and exempt her unpaid annual lottery pay-outs.

o Holding: Not annuity; non-exempt.o Reasoning:

Precedent made annuities exempt. But she is not the annuity owner, the state of Connecticut is. There are not annuities payable to her. She only has a contract entitlement. Note: If she were listed as the beneficiary, she would be entitled to exempt the property because of precedent.

Equity and policy argument; comparing life insurance, retirement benefits and the lottery winnings.

Summary:o When a statute exempts property by classification, rather than by

total dollar value, there will be hard questions at the margin about what property falls within the classification.

4. Valuation of exempt propertyo Whether the property fits within the permitted valuations.

IN RE WALSH , 1980, p.182o Should the value of the property be the FMV or liquidation value?o Holding: FMV (or just value?) in §522 is equivalent to the liquidation

value. o Reasoning: The property will be subject to the bankruptcy sale.o Discussion: What is the FMV? Price at which a willing seller and a

willing buyer will trade.In re Mitchell, 1989, p.184

23

Page 24: Bankruptcy Outline - Pottow 2011

o The appropriate valuation standard for the jewelry is FMV.5. Proceeds and tracing

o General rule: the proceeds of the exempt property should be exempt. But there may be difficult and inconsistent resolution.

o The retirement account is exempt. A debtor draw all the money in his retirement account to buy a pick-up after his filing for Chapter 7 and alleged that the pick-up is exempt because it is in a little more tangible from of his retirement account. The court said no.

6. Partially exempt propertyo If there is a dollar limit on a piece of property, it is partially exempt.o In most cases, it can be levied on and sold.o The exemption attaches, up to its dollar limit, to the cash from the

sale.7. Security Interests in exempt property

o The secured party has a priority over the debtor and TIB on the proceeds of the sale of the exempt property.

8. Avoiding judicial liens and non-PMSI lienso Background: Exemptions can't exempt voluntary liens. §522(f) is

designed to prevent involuntary liens; exception for things that are exempt even when voluntary liens were taken out on them. Policy to prevent "hostage liens."

o §522 (f) permits the avoidance of two kinds of liens on exempt property. §522(f) means that the debtor cannot voluntarily waive certain property rights. Judicial liens Nonpossessory, nonpurchase money consensual security

interestso In order to avoid the liens, the collateral must be exempt property.o In order to void a consensual security interest, the property

must also be of a kind of “household good” that is specified in §522(f)(4).

o For non-purchase money, nonpossessory interests, avoid them to the extent that the exemption is prejudiced.

o In effect, the law here says that these creditors cannot push their claims ahead of the debtor’s exemption, as with regular security interests or liens can.

o In general, a debtor can avoid all judicial liens on exempt property. Only one exception is the judicial lien for a domestic support obligation.

Problem Set 8, p.196

8.1 o Get most things. Not stock or checking account. This discourages

people from having liquid assets. o Wheelchair not towards the $60,000 cap (TX) on personal items.  o What of the potential $50,000 settlement? Is a cause of action

included in his estate? This is a legal or equitable interest. What if it

24

Page 25: Bankruptcy Outline - Pottow 2011

were a business with a copyright suit entitlement? Seems like property that they should have to list. 

o What could they keep under Federal law? Lower cap. But we don't meet the whole cap under Texas law anyway. Wheelchair (big item) is uncapped under Federal law, too. 

o What could they keep under Delaware. Only a $5000 cap. Lame. No wheelchair even?

o Gets the whole value of the Bible under WY statute? No cap?

8.2 o Federal only exempts amount she needs to support herself. §522o TX §1108.0531 exempts all of the life insurance

8.3 o If you sell a house, so first you pay the commission of 6%; then pay

secured creditors who are not influenced by the exemption; third, exemption; fourth, how about the leftover? It is the estate

o Does this fall in §522 (f)? To avoid the nonpossessory, nonpurchase money security interest? No. here it is a purchase money security interest.

8.4?

D. Homesteads, trusts, and exemption planning

1. The homestead exemptionso Typically the homestead is exempt from execution by creditors up to a

given dollar amount. But seven states and the DC now offer homestead protection based on area or some other test.

o Florida and Texas: No value limit of homestead exemption.

2. Exemption planningo In the states where there is no value limit, the debtors carefully plan

their filing for bankruptcy to discharge their debts.IN RE REED , 1981, p.201

o Reed could keep his conversion from the non-exempted property to exempted property under Texas law. But the federal court found his intent to defraud his creditors, so he was denied a discharge in bankruptcy.

o It is exempt property; §727 intent to defraud.o He wants a fresh start, but he was denied the discharge. Basically he

lost.

2005 Amendmento A provision to reduce the dollar value of the homestead

protection by any amount that is attributable to otherwise non-exempt property that the debtor dispose of with intent to hinder, delay or defraud a creditor; the provision has a 10-year reach-back period.

25

Page 26: Bankruptcy Outline - Pottow 2011

o Another provision for an absolute cap on the homestead for people who were convicted of securities law violations, fraud in a fiduciary capacity, and a handful of other related bad acts. §522(q) if necessary, the discharge can be delayed to see if the debtor is subject to a proceeding that might give rise to a limitation of the homestead exemption. §727(a)(12) “Enron-ized.”

3. Unlimited Exemptions and asset trustso FORSBERG V. SECURITY STATE BANK , 1926, p207

It is not fraud that the debtors who are insolvent to transfer their non-exempt assets to exempt assets, placing them out of the reach of creditors.

o Self-settled trust: the trustee and beneficiary are the same person People try to calculate the appropriate level of legitimacy for a

trust. If it is too fraudy, then it might be pierced; it is too real, you can't

really control your own assets.o Asset protection trusts, permitted in 11 states.

It is also open to out-of-state residents.o Solution: IN RE REED , if the intent to defraud, deny the discharge.

But it is tricky to determine the intent.o Alternative possible solutions:

If the debtor has intent to defraud the creditor, the creditor may avoid the transfer of assets to the trust under UFCA.

It may be possible for us to introduce the theory of piercing the corporate veil to pierce the trust veil.

4. Moving to better exemptionso IN RE COPLAN , 1993, p.211

Issue: whether considering all the circumstances, the debtors’ relocation to Florida with the attendant purchase of a home was for the specific purpose of shielding their assets from creditors.

Holding: Bankruptcy jurisdiction shopping; the debtor can't run to good bankruptcy jurisdiction to prejudice creditors.

Court applies Wisconsin exemptions to the case even though they are now Florida citizens.

Codified under §522(b)(3) in 2005 amendments. In this case, the judge can't rely on this new amendment, but follows it out of intuitive equitable sensibilities.

o 2005 Amendment: a sort of choice of law provision for exemptions. §522(b)(3) the applicable exemption is wherever the debtor

resided for 730 days. If the debtor moved, go back to the 180 days preceding the 730

days and see where the debtor was for the majority of that time.

Problem set 9, p.2179.1

o §522(b)(3). Not in the same place for two years. So look at the 180 days before the two years.

26

Page 27: Bankruptcy Outline - Pottow 2011

o Does Texas law apply? Does he get Texas homestead law then? He lived in Texas for 91 days, so he applies Texas law. But Texas homestead law only protects homesteads in Texas. So he cannot use Texas law.

o If there is no protection under any one state, then Federal exemptions apply with a cap of $18,500. (b)(3)(c) (end).

9.3 o Are we okay with him making this asset-protection plan? Does he

need to have the chance of litigation to keep the pressure on him to not commit a tort?

o  §522(f).

E. Claims and Distributions1. The Claims Process

o For Chapters 7 and 13, the creditors must file a proof of claim form (Official Form No. 10) in order to receive a dividend, even if they are listed on the debtor’s schedules.

o For Chapter 11, it is not required for creditors who are scheduled by the debtor to file a proof of claim form.

o To file a proof, a claim based on a writing must have a copy of the writing attached. Rules 3001-3008.

o Objections to a claim under §502(a); resolved as a “contested matter” under Rule 9014, unless the objection also makes a demand for a type of relief that converts the matter into an adversary proceeding under Rule 7001, like a demand to void a preference.

o More complex claims: warranties, contingent obligations and future claims based on past wrongdoing.

2. Disputed Claimso The debtor has little incentive to fight with creditors and is inclined to

settle; so TIB has to review the claims.

3. Unsecured Claimsa. The Claim

o All pre-petition claims, secured and unsecured, must begin with a §502 calculation.

b. Interesto Unsecured creditors get no post-petition interest because it is

unmatured. Rationale: pro rata distribution among unsecured creditors. Reshape the equality of unsecured creditors about the different

interest rates.o For a lump sum of interest rather than interest calculated over time

would be reanalyzed by a bankruptcy court to determine the portion of interest that was mature and unmature as of the instant filing.

c. Accelerated claimso All pre-bankruptcy claims are accelerated no matter whether it is

matured or not.o It also involves many valuation questions to bankruptcy judges.

27

Page 28: Bankruptcy Outline - Pottow 2011

4. Secured Claimsa. The Claim

o §506 allowed secured claimo A secured claim equal to the value of the collateral and an unsecured

claim for the deficiency.b. Interest

o Oversecured creditors are entitled to the post-petition interest at the contract rate up to the remaining value of the collateral.

c. Attorneys’ feeso All creditors including unsecured creditors are entitled to attorneys’

fees if the state law or contract allows this.o Oversecured creditors are entitled to the post-petition attorneys’ fees

up to the remaining value of the collateral.o Other creditors are not entitled to the post-petition attorneys’ fees.

d. Exemptionso The debtors can only claim exemption of the value remaining after the

secured creditor has been paid in full.

5. Post-petition Claimso Post-petition claims will be treated as expenses of

administration under §503 and paid by the first priority under §507 (a)(1)-(2).

Problem Set 10, p.227-22810.1

o $3200; pre-petition interest is allowable. §502b (2) excludes the unmatured interest.

o $3000 principal + pre-petition interest $200 + $0 (No $100 post-petition interest despite state provision) = $3200.

10.2 o $9900, capped at the value of the collateral; $8k principal; $400 post-

due interest (§506b); $500 interest after the petition; $1k lawyers’ fee, reasonable (§506b).

10.3 o Bank only gets $5K secured interest; no post-due interest or post-

petition attorney’s fee any more; so the unsecured debts $3500; totally allowed claims of 8500; if the attorney fee is allowed under the state law or contract, it will be allowed as unsecured claim.

10.4 o First give $5K from the car to the Bank; second, the bank still has

unsecured debts of $3500 (or $4.5k); third, distribute $15K among the creditors of $20K + $3.5K (or $4.5K) + 3.2 Pluming co. in question 10.1

Debts to Sovereign States, p.271-277o When a creditor is a state or local government, the federal courts

have very limited powers of enforcement. This limitation comes from

28

Page 29: Bankruptcy Outline - Pottow 2011

SEMINOLE TRIBE OF FLORIDA V. FLORIDA , in which the state asserted the 11th Amendment the state sovereignty.

o The 11th Amendment: Can't sue a state (even though language actually only says people from outside the state can't sue a state). SEMINOLE INDIANS V. FLORIDA .

o §106 waiver of sovereign immunityTENNESSEE STUDENT ASSISTANCE CORPORATION V. HOOD , 2004, p.272

o Holding: The state is bound by the discharge order issued by the bankruptcy court. But it is not clear that the bankruptcy court can impose an injunction on the state’s holding of the driver license for the violation of the automatic stay. (expand to the orders that require the state to do something, like an order to turnover some property)

o Reasoning: Refuse to follow Seminole tribe. They say Seminole was in personam and this is in rem so it is different. Bankruptcy code can apply to the states.

o What about an action under Fraudulent Transfer in bankruptcy where the estate tries to recover from a state charity? O'Connor's last opinion says this is okay. Conservatives dissented.

Problem 13.5, p.278 6. Priority among unsecured creditors§507(a)(1)(C) and §507(a)(2)

o General rule: domestic support obligation has a higher priority than the administrative fees.

o But if the administrative fees are incurred to collect the assets for the domestic support obligation, the these administrative fees get paid first. §507(a)(1)(C)

Section 507 (a) Priority in the following orders:

(1) First, domestic support obligation;o Exception: Administrative expenses of the trustee allowed under

§503(b)(1)(A) actual, necessary costs and expenses of preserving the estate including wages, salaries, and commissions for services rendered after the commencement of the case; wages and benefits pursuant to a judicial proceeding or a proceeding of the National Labor Relations Board as back pay attributable to any period of time occurring after commencement.

§503(b)(2) compensation and reimbursement for the trustee awarded by the court

§503(b)(6): the fees and mileage payable under chapter 119 of title 28.

(2) second, administrative expenses under §503 (b) and any charges and fees assessed against the estate;

(3) third, unsecured claims under §502 (f) in an involuntary case, a claim arising in the ordinary course of the debtor’s business or financial affairs after the commencement of the case but before the earlier of the appointment of a trustee and the order for relief shall be determined as of the date such claim arises;

29

Page 30: Bankruptcy Outline - Pottow 2011

(4) fourth, the following claims to the extent of $10K earned within 180 days before the date of the filing or the date of the cessation of the debtor’s business, whichever occurs first:o wages, salaries, or commissions, including vacation, severance,

and sick leave pay earned by an individual; oro sales commissions earned by an individual or by a one-person

company, acting as an independent contractor in the sale of goods or services for the debtor in the ordinary business if and only if during the 12 months preceding that date, at least 75% of the amount that the individual or company earned as an independent contractor was earned from the debtor.

(5) Fifth, contribution to an employee benefit plan –o Arising from service rendered within 180 days before the date of

the filing or the date of the cessation of the debtor’s business, whichever occurs first; but only

o For each such plan, to the extent of – The number of employees covered by this plan multiplied by $10K less The aggregate amount paid to such employees under paragraph (4), plus the aggregate amount paid by the estate on behalf of such employees to any other employee benefit plan.

(6) Sixth, claims of persons-o Engaged in the production or raising of grain against a debtor who

owns or operates a grain storage facility for grain or proceeds of grain. Or

o Engaged as a fisherman against a debtor who has acquired fish or fish produce from a fisherman through a sale or conversion, and who is engaged in operating of a fish produce storage or processing facility-but only to the extent of $4925 for each such individual.

(7) Seventh, claims of individuals, to the extent of $2,225 for each such individual, arising from the deposit, before the commencement of the case, of money in connection with the purchase, lease, or rental of property, or the purchase of services, for the personal, family, or household use of such individuals that were not delivered or provided.

(8) Eighth, claims of governmental units, only to the extent that such claims are for

A. A tax on or measured by incomeB. A property tax without penaltyC. A tax required to be collected or withheld for whatever capacityD. An employment tax on wage, salary or commissionE. An exercise tax on …F. A customs dutyG. A penalty related to a claim of a kind specified in this paragraph

and in compensation for actual pecuniary loss.(9) Ninth, claims based on any commitment by the debtor to a Federal

depository institutions regulatory agency to maintain the capital of an insured depository institution.

(10) Tenth, claims for death or personal injury resulting from the operation of a motor vehicle, or vessel if such operation was

30

Page 31: Bankruptcy Outline - Pottow 2011

unlawful because the debtor was intoxicated from using alcohol, a drug or another substance.

(b) if the trustee under §§362, 363 or 364, provides adequate protection of the interest of a holder of a claim secured by a lien on property of the debtor and if the creditor has a claim allowable under (a) (2) arising from the stay of action under §362, from the use, sale, or lease under §363, or from the granting of a lien under §364(d), then such creditor’s claim shall have priority over every other claim.(c) for (a) a claim of a governmental unit arising from an erroneous refund or credit of a tax has the same priority as a claim for the tax to which such refund or credit relates.(d) an entity that is subrogated to the rights of a holder of a claim of a kind specified in subsection (a)(1), (a)(4), (a)(5), (a)(6), (a)(7), (a)(8), or (a)(9) is not subrogated to the right of the holder of such claim to priority.

§503(c)o The insider should not be paid a retention “bonus” except the court

finds that (A) Payments are essential to retention of the person because he

has a better job offer or compensation (B) The services provided by the person are essential to the

survival of the business, or (C) The dollar limit…

o no severance payment unlesso no other payments that are outside the ordinary course of business

and not justified by the facts and circumstances.

Problem Set 11, p.228

1. John Harry, §507(a)(4) the 4th priority; limited to 180 days before the filing of petition or the cessation of the business which occurs earlier

2. §507(a)(8)(C) social security is a governmental pension system, 16% of the payroll tax, of which the employer pays 8% and the employee pays 8%; for the self-employed business, pays 15%; here is the withhold by the employer for the employee, so use §507(a)(8)(C); §507(a)(8)(D) cover the employer’s part;

3. property tax: §507(a)(8)(B) limited to one year; without penalty; the earlier two years than one year have only general claimsWhat is the meaning of “last payable”? Like the deadline for the individual income return is April 15 in the following year; so today, February 16, so the 2006 income tax is not last payable if you file a bankruptcy petition today.

4. down payment: §507(a)(7) deposit in connection with the purchase of property

5. IRS §507(a)(8)(A) three years6. Utility following bankruptcy: this is a tricky question. The claims

following bankruptcy are not allowable to the property of the estateShould it be paid before the priority claims? No. the debtor can pay the utility by his wages after the filing, or by his exempt property.

31

Page 32: Bankruptcy Outline - Pottow 2011

But notably the after-filing debt is not dischargeable, so utility companies can negotiate with the debtor for a higher price or something.

7. Attorney fee?Under 330, the compensation of officers in the bankruptcy will get the priority in the distribution. But §330(a)(1) the list of the officers does not include the attorney, however, §330(a)(1)(A) does include the attorney. The Congress interprets it as a typo error, so the attorney cannot get the priority and can only be a general creditor. In response, the attorney usually requires to be paid in advance before rendering the service, because he cannot get the priority in the distribution.Here the attorney fee is divided into the will service and filing service. It is a pitfall. There is no difference between these two debts, and both cannot get the priority.

8. The negotiable note held by his ex-wife: is it a domestic support obligation under§507(a)(1)(A)? It seems that, but it is not. The negotiable note is a dollar bill and does not rely on any substantial claims. So it is only a general claim.§101(14A) definition of domestic support obligationAlthough this negotiable note has no priority under §507(a)(1)(A), is it exempted from the discharge under §523(a)(5) or (15)? By the same token, I think it is not.

9. Trustee and his counsel: §507(a)(2)10. insurance premium, §507(a)(2)11.cost of sale, $2800; §507(a) (2); It also could be the secured creditors.12.other claims: general claims; no priority.

F. Discharge1. Exception to discharge

o The trustee or creditors may object to the debtor’s discharge of particular debts under §523 (a rifle shot) or of all debts under §727 (a global denial).

IN RE ROBERT W. MCNAMARA, DEBTOR (Bankr. D. Conn 2004), p.230o A global denial caseo Facts: Lost the money in a poker game.o §727 discharge is allowed unless the debtor has failed to explain

satisfactorily any loss of assets or deficiency of assets to meet the debtor’s liability. The creditor/plaintiff has the burden of introducing evidence of the

disappearance of assets or of unusual transactions. The debtor/defendant has the burden to satisfactorily explain the

loss or deficiency of assets.IN RE DORSEY

o Very different from the global denial as a rifle shot: under §523 (a), the creditor must object to discharge in bankruptcy court or the debt will be discharged automatically. §523(c)

o Fact: A widow with two children depending on only $480 per month used her credit cards for the travel to Europe, allegedly in good faith and repay by the generosity of her mysterious boyfriend.

32

Page 33: Bankruptcy Outline - Pottow 2011

o Holding: The court found this involves actual fraud when she had no intention to pay these charges. Denial of discharge of this credit card loan

Student loans – IN RE GERHARDT o The Brunner test: undue hardship test under §523(a)(8): three

parts: The debtor cannot maintain, based on current income and

expenses, a “minimum” standard of living for himself and his dependents if forced to repay the loans.

Additional circumstances exist indicating that this state of affairs is likely to persist for a significant portion of the repayment period of the student loans

The debtor has made good faith efforts to repay the loans. o This case a professional cellist failed the second prong.

IN RE PATRICIA M. MILLER , debtor, p.242o Points: Review of the legal standards to determine dischargeability

but permit a remedy not found in §523.o Issue: Whether a bankruptcy court can rely on §105(a) to grant a

partial discharge of student loan indebtedness and whether, before a bankruptcy court grants such a discharge, it must first find that the portion being discharged satisfies the “undue hardship” requirement.

o Holding: The court did not apply the BRUNNER test but used its discretionary power to review many factors. For the later question, it is yes.

o Implication: Not all courts accepted that a partial discharge of student loan can be written into the bankruptcy code, but Congress did not attach the Miller line of cases in 2005 Amendments.

2. Tax Priorities and dischargeo §507(a)(8)(A)-(G) taxes are not only given priority in payment, but

any unpaid portion is exempted from discharge by §523(a)(1)(A).o Pre-petition interest on §507(a)(8) priority claims shares the priority

of the claims themselves and enjoys their nondischargeable status.o Post-petition interest does not accrue to unsecured tax claims against

the TIB and the property of the estate §502(b), but does accrue against the debtor as to any unpaid, undischarged tax debts that survive the bankruptcy.

o Penalties on nondischargeable taxes are also nondischargeable even though such penalties do not get priority in payment under §507(a)(8).

o IRS has the right to satisfy tax debts by seizing property that is otherwise exempt under state law.

Problem Set 12, p.25712.1 Yes, he would have difficulties. §727(a)(3) failed to keep or

preserve any recorded info and this failure could not be justified

33

Page 34: Bankruptcy Outline - Pottow 2011

under all of circumstances. §727(a)(3) does not say how long you should keep the records: forever.

12.2 §523(a)(2)(B) and §727(a)(2); If the statement of the debtor is oral, just as “I has a lot of money”, is it covered by §523(a)(2)(B)? Probably no. but it is covered by §727(a)(2), right? I think there is no requirement for writing under §727(a)(2); if the creditor has a choice, §523(a)(2)(B) as a rifle shot is more favorable. If he sues under §727(a)(2) as a globe denial, other creditors will have a free ride.

12.3 $2500, $1000 and $200 are alimony, covered by §523(a)(5) that exempts a domestic support obligation from discharge; for the $10K lump sum, it is more like a settlement, covered by §523(a)(15). But is it influenced by the state law or the debtor or creditor’s economic ability? It does not matter the relative incomes of the parties.

12.4 §523(a)(6) willful or malicious injury; Drunk driver is not discharge under §523(a)(9); tort claimants from the driving under §507(a)(10) has a priority.

12.5 §523(a)(2)(A) and (C); it could be a false pretense under §523(a)(2)(A). Under §523(a)(2)(C), the debt for the luxury goods is presumed to be a false pretense. So (C) shift of the proof burden to the debtor. The key discretionary issue is whether the plane ticket, wallpaper, etc. are the luxury goods, and are they necessary?

12.6 §523(a)(7). Not dischargeable. Does the ticket get the priority? No; it cannot fit into the §507(a)(8).

12.7 Student loans is not dischargeable as a general rule.o Moral hazard; why not other debtors? What is the policy? o Students have the high potential income; income-related moral

hazard.o Have not made the efforts to pay.o Why do we distinguish the education loan and other loan?

From the perspective of investment, the creditors cannot take away the education.

G. THE DEBTOR’S POST-BANKRUPTCY POSITION: REAFFIRMATIONo §524(c) opens a door for the creditors for the soon-to-be-discharged

debt.o §524(c) includes a requirement that the agreement be filed with the

court and that it contain an option for the debtor to rescind the agreement for 60 days.

o §524(c) requires the lawyer to certify that the reaffirmation is in the best interest of debtor and to file an affidavit to that effect. This puts the debtor’s attorney in the position of decision-maker.

o Leverages for the creditor to ask for reaffirmation. Secured creditor: Foreclosure. Unsecured: Future credit and the threat of an objection to

discharge.

1. Reaffirmation of secured debt

34

Page 35: Bankruptcy Outline - Pottow 2011

Three ways for the debtor to keep the property.o Redeem the collateral, §722.o Negotiate a reaffirmation; §524(c).o Ride-through: retention; requires to maintain the contractual

payments; case law Keep the collateral by continuing to make the pre-bankruptcy

payments, without redeeming or reaffirming.Notes: All these reaffirmation efforts cannot be paid by the property of the estate; it should be paid by the free property, like the post-petition wages.

IN RE PENDLEBURY , 1988, p.263o The question is that the creditor requires an attorney’s fee provision

in each reaffirmation agreement.o §524(c) is a voluntary undertaking. It is a free-market activity.

2. Reaffirmation of unsecured debto Reasons to protect unsecured debt

Gratitude Affection To protect a co-debtor

IN RE PAGLIA , 2003, p.267o The debtor defaulted his loan from the first national bank. His mother

was the co-signer and assigned her interest in pension annuity to the bank. Debtor issued a note to the bank and continued to pay the notes after his discharge of bankruptcy.

o Issue: Debtor asserted that the bank violated the discharge injunction under §524(a)(2) and automatic stay under §362(a).

o Holding: Debtor voluntarily executed the notes because he did not want the

bank to take legal action against her mother’s annuity. §524(f) does not prevent the debtor from voluntarily repaying any debt.

The bank’s acceptance of the payment is not an act to collect the debt.

The debtor asserted that the note was unenforceable because it was not filed with the court, and did not advise debtor that he could rescind it within a specified period of time. The court held that the note is not a reaffirmation of the debt.

IN RE LATANOWICH o Threat of repossession and the incentive of new credito Debtor got discharge of his debts to Sears. Sears threatened to

repossess the goods unless the debtor signed the reaffirmation agreement. And Sears would provide new credit for him to restore his credit record. Debtor signed.

o Holding: Sears violated §524(c) in both aspects, so it is void. The reaffirmation agreement was not filed with the court. If the debtor was not represented by counsel in the negotiating the

agreement, the court must approve the agreement as

35

Page 36: Bankruptcy Outline - Pottow 2011

Not imposing an undue hardship on the debtor or his dependent

In the best interest of the debtor.

Problem set 13, p.2771. The membership of the gym club is suspended

§524(a)(2) discharge of the debts means that Peter does not owe money to MM. If MM revoked his membership due to his debt, this is a violation of the injunction of the act to collect the debts.

Counterargument: we are just angry. We don’t intend to collect the previous debts.

Breaking point: What is the reason of the suspension? If Peter pays back two months payment, peter is allowed to return back. Then the club is in trouble.

What if this was before discharge? Do they have to let him keep coming? Seems like you would have to continue providing services

2. Impulsive reaffirmation First, under §524(c)(2), Kevin can assert that he did not receive the

disclosure under §524(k), so the agreement is void. Second, he was not represented by the lawyers in the negotiating

the agreement, so the agreement has to meet §524(c)(3): filing with the court and court approves it if it does not impose an undue burden on the debtor and is in the best interest of the debtor.

Third, even if the agreement is enforceable and filed with the court, he can still rescind the agreement within 60 days.

3. We gonna sign an undue hardship thing for Big Moon's redemption? He can redeem whatever he wants. Try to talk him out of it. Remember for §722 it has to be exempt. Can only redeem to the amount of the exemption cap? Looks like no cap for redemption… Why? The cap is that redemption cannot extend beyond the exemption; an implicit cap.

4. Is the car exempt under §522? §522(d)(2)? It depends on the state law. If the state law has no opposite stipulation, it is exempted to $2,950. But can she keep the car or she can just get the exempted money? If it is fully exempted, she can keep the car; if partially exempt, she can only get the money Receptive to the reaffirmation? How about the other two

unsecured signature loans? This is a bad deal for her. Don’t sign. If she could possible get

redemption on the car, that's ok.  Note: remember the Sears case. Is soliciting a reaffirmation a

violation of the proscription on an act to collect. Is it all a matter of who starts the reaffirmation negotiation? It does matter who initiate the reaffirmation. If the creditor initiates the reaffirmation, it may violate the stay.

Note: After CH 7 personal discharge, the lien on secured debt still attaches to the property.

5. Q.8 §525(b) no private may terminate the employment of an individual who was a debtor in the bankruptcy case.

36

Page 37: Bankruptcy Outline - Pottow 2011

CH 6: Ch 13 Bankruptcy: Secured Creditors

A. Elements of an acceptable plan1. An overview of CH 13

o Gets to keep all his property; taking 3-5 years to pay off all the debts.o If you finish the plan, you are discharged of other debts.o Adjustment of debt; wage earners’ plan.o The role of the trustee:

Recommend approval or denial of confirmation of a debtor’s plan. Is obligated to ensure that the payments are commenced within 30

days after the plan is filed and that the payments are properly distributed to creditors.

Ordinary, the debtors will make a lump-sum monthly payment to the trustee and the trustee will then distribute to the creditors.

Monitor the debtor’s compliance with the plan and file to dismiss the debtor’s case for nonpayment.

If the debtor fails to pay, the trustee often seeks wage attachment orders. State restrictions on wage garnishment are inapplicable.

Standing trustee, appointed by UST.

2. Payment to secured creditorsTwo issues:

o Protection of the secured party’s interest in the collateral: adequate protection, §362(d)

o Adequate payment to the secured partya. Adequate protection

ob. Modifying the secured creditor’s contract

o General rule for the payment to the secured creditors is contained in §1325(a)(5) must be paid in full to allowed secured claims must be paid interest on that claim

o A secured claim equals to the value of the collateral and an unsecured claim for the remainder. The remainder is also called as deficiency, or cramdown section.

o The treatment of an undersecured claim is often called cramdown, because it can be imposed over the secured creditor’s objection; For the cram down, the debtor should pay at least the

present value of the collateral. It does not say that creditors get the full payment of the claim

with a present value of the collateral. How to calculate the present value? See associates commercial

corp.o 2005 Amendment added language to 1325(a): exempt certain

security interests from the cramdown rule: they must be paid in full plus interest first, any security interest granted within the year before

bankruptcy

37

Page 38: Bankruptcy Outline - Pottow 2011

Second, for the holder of a purchase money security interest in a motor vehicle, the security interests granted within two and a half (910 days) prior to the bankruptcy petition. §1325(a) (9) §1325(a)(9) you can't bifurcate claim into allowed secured and

unsecured claim. In a general rule, the undersecured claims are divided into secured and unsecured portions; like the claim is 4K; the collateral is worth $2K; then the creditor has 2K secured claims and 2K unsecured claims. But under §1325(a)(9), if the claim satisfies the conditions, you cannot bifurcate it; you can get $4K secured claim, like the §1111(b) election.

ASSOCIATES COMMERCIAL CORP. V. RASH , 1997, p.285o How to determine the value of the collateral?

Code §506(a): Choices: foreclosure value; replacement value; the midpoint

Holding: replacement value 2005 amendment codified the Rash rule, see §506(a)(2):

replacement value

c. Computing the amount the secured creditor must be paido the calculation of the present value: how to determine the interest

rateTILL V. SCS CREDIT CORP , 2004 p.292

o four options for the interest rates: the formula rate the coerced loan rate the presumptive contract rate the cost of funds rates

o Holding: the formula rate = prime rate + risk premium

Problem Set 14, p.3061. First there is an automatic stay for any repossession, so IC cannot

take it at will.Second, IC should bring a petition to lift the stay under §362(d), and he has two choices. IC can rely on §362(d)(1) for the cause of lack of adequate

protection under §361. What can be the argument for lack of adequate protection?

Like market drop, depreciation, tear and wear, etc.; but IC cannot get the opportunity cost and therefore ask for post-petition interest,

What can the debtor do for the adequate protection? See §361: cash payment, replacement lien, additional insurance, and others.

IC can also rely on §362(d)(2): the debtor has no equity in the property and the property is not necessary for the effective reorganization. We calculate the equity following the petition valuation.

38

Page 39: Bankruptcy Outline - Pottow 2011

Under §1325, the secured creditors must be paid in full for the confirmation of plan; the debtors can pay the creditor lump-sum cash or installment with a market interest.

(Notes about the interest: Pre-bankruptcy, creditors get the interest; from bankruptcy to confirmation: no interest; from confirmation, post-confirmation over three years: you have interest)

2. First, what is LL’s allowed claim? $39980 (principal) + $12300 (pre-petition interest) = about $52K (Note: if it is oversecured claim, the creditor can be entitled to post-petition interest with the K rate)Second, we bifurcate the claims: 41K secured claims to the extent of the value of the collateral; $11K unsecured claimsThird, for the secured claim, George had to pay back in full with the interest. What kind of interest should be paid? Prime interest rate +

adjustment3. First, it should be the replacement value without deduction of the

costs of sale or marketing under §506(a)(2). Replacement value: the price a retail merchant would charge for

property of that kind considering the age and condition of the property at the time value is determined.

Second, how much should the debtor pay? Under §1325(a)(9), she should pay the purchase money loan for the car within 910 days before the petition in full. Can't bifurcate claim into allowed secured and unsecured claim. Disputing point: §1325(a)(9) requires the car for personal use; but

here it may be argued that it is for the use of businessNotes: No modification for houses: §1322(b) (2).

4. §1322(b) (2) you cannot modify the rights of holders of secured claims if the claim is only secured by the debtor’s principal residence. So the debtor has to pay the full mortgage of $182k and the market interest§1322(b)(5) the debtor has to reinstate the mortgage and cure the defaultsThis is pretty tough for the debtor, so why does the Congress do this? Good lobby from the mortgage capital.

5. How about the filing in bad faith? Does it still get the automatic stay? Yes, until it is dismissed. §109(h). §521(b). Credit counseling service? §521 certificate for

credit counseling and the repayment plan. What is this for? Modification of the automatic stay. When you file for bankruptcy, the clerk will require the certificates; if not, the filing still gets the automatic stay until it is dismissed. Without the credit counseling certificate, it will be dismissed. So you cannot use the bankruptcy filing as an instant strategic tool any more.

 Do they have a previous remedy under bankruptcy? §362(c)(3) limit the stay. The stay will terminate on the 30th day after the filing if the debtor has a pending case within the proceeding 1 year but was dismissed. (c)(3)(B) requires good faith.

39

Page 40: Bankruptcy Outline - Pottow 2011

§362(b)(22) stay on eviction between the lessor and lessee.

6. They can file for CH 13 and they can modify the claims in their repayment plan, including secured or unsecured; but they cannot modify the claims secured by their principal residence. But the question is whether you advise them to file for CH 13 or Ch 7? What is the downside of CH 13? They cannot modify the home mortgage. The repayment is 3-5 years; very long.For the CH 7, they can get rid of all the debts, but they cannot keep their equity. If they want to, they have to reaffirm the claims with the creditors, in which the creditors have more bargaining power.The baseline to take CH 7 or CH 13 is If the mortgage is high, CH 7 is more favorable to the debtors? If

the mortgage is low, CH 13 is better. File for Chapter 7 to get rid of unsecured debt. Try to get

reaffirmation on mortgage. Bank can repossess but will likely negotiate a deal.

4. Payments to unsecured creditorso Secured creditors and priority creditors are paid in full in CH

13. But unsecured creditors are pooled together for pro rata treatment.

o The unsecured creditors have three ways to protect their interests: The best interest test: each creditor, secured or unsecured,

receives at least as much as that creditor would have received under CH 7. §§1325(a)(4) and (a)(5)(B).

The debtor must devote all “disposable income” to plan payments during the life of the plan. §1325(b).

A plan must be proposed in good faith and not by any means forbidden in law. §1325(a)(3).

a. Disposable incomeo 2005 amendment alienating the disposal income test in CH 13

that parallels the means test in CH 7.o It also starts with a median income test. A married debtor who files

alone must include spousal income in determining whether the debtor’s income is above the median.

o If the debtor is below the median, he passes the threshold test and is governed by a slightly modified version of the “reasonably necessary” test for disposable income adopted in 1984.

o If the debtor is above the median, he suffers two adverse consequences: First, they must keep paying for 5 years, not 3 years; this is not

necessarily related to whether there is a surplus of income over the expense

Second, the CH 7 means test applies here, and he has to pay the surplus of income over the expense to unsecured nonpriority creditors in CH 13.

i. Below-median debtors

40

Page 41: Bankruptcy Outline - Pottow 2011

o Disposable income: income and necessary expenseIN RE CARTER , 1996, p.309

o The CH 13 may only be confirmed that repay the allowed unsecured claims in full or committing all his disposable income for 3 years. §1325(b)(1)(B)

o Disposal income is income received the debtor that is not reasonably necessary for the support of debtor or his dependent. §1325(b)(2)(A)

o The debtor’s spousal income should be disclosed, because the spousal income may be used to the basic needs of the debtor, potentially increasing the share of the debtor’s own income that is not reasonably necessary for support.

o Holding: the plan is not ready for confirmation, because it cannot demonstrate that the debtor’s all disposable income is committed to the plan.

IN THE MATTER OF WYANT , 1998, p.312o The increases in projected monthly expenses are attributable to an

unwarranted attempt to offset his increase in income, and that the increase in expenses is not reasonable.

o As between the debtor’s elderly horses and dogs and his creditors, I think that the creditors should be paid first. The proposed expenditures on these animals are excessive, unreasonable, and not necessary for the maintenance or support of the debtor or his dependents.

ii. Above-median debtorso Five years; a surplus of income calculated according to the means test

of §707(b)(2)o §1325(b)(3): The debtors have to pay the entire surplus according the

means test to the unsecured nonpriority creditors for 5 years. They write a check to the trustee every month equal to the surplus.

o The above-median debtor with a $75 surplus does not have to use the IRS guidelines for expenses, because the means test only applies to the debtors with a surplus over 100 per month. Instead, they have to pay whatever amount equals income minus reasonably necessary expenses. That excess could be considerably more or less than $75 per month. The above-median, no surplus debtor may have a powerful incentive to file in CH 7 to avoid having to complete a 5-year plan.

o The debtor with a surplus over the $100 pays the amount that is limited to the means test surplus. The court has no discretion.

o The courts’ discretion – making people count a non-filing spouse’s income or giving up a pricy car, for example – is reserved for below-median income debtors and for above-median income debtors who has no surplus debt.

o §1325(b)(2) allows additional deductions for domestic support payments and certain charitable contributions.

iii. §1325(b)(3) the means test

41

Page 42: Bankruptcy Outline - Pottow 2011

o How to understand the means test in Ch 13? There are two ways of understanding it:

(1) One is the means test §707(b)(2)(A)(B).(2) Second is the means test,

If flunks, the means test §707(b)(2)(A)(ii); you have to use the IRS budget

If passes, default rule of Ch 13: R+N

b. Competition among creditors

c. Family support, taxes, and other priority claims in CH 13i. Priority repayments in general

o The priority claims must be paid in full but NO interest, so they are no longer placed in the competitive position they sometimes suffer in CH 7.

ii. Tax claimso Taxes are not dischargeable under CH 7. But CH 13 offers two

advantages: Paying the tax over time with the automatic stay holding off the

IRS. §362(c)(2)(C). The denial of post-petition interest on unsecured claims will lock

the tax claim at its value as of the date of the bankruptcy filing. §502(b)(2).

o Secured claims and unsecured claims are paid in present value, which means the interest. §§1325(a)(4) and (a)(5).

o §1325(a)(2) for the priority claims, the debtor is required to pay the nominal amount of the claim without interest.

d. Good faitho In many no-asset cases, the best interest test cannot protect the

creditors. Some courts use the good faith test to force the debtors to pay a higher sum. After the disposable income added in 1984, whether this test is necessary is a question and the courts divide.

o After 2005, the code retains the totality test only as an alternative where the presumptive-abuse doctrine does not apply. §707(b)(2)(B)(3) So the standard is still alive, especially in the below-median cases where the presumptive-abuse doctrine is not applicable.

IN RE LEONE , 2003, p.324o Whether a plan is filed in good faith is a question of fact based on the

totality of the circumstances. Notes:

o 2005 Amendment removed secured debt entirely from the calculation of income and expense. 2005 amendment also contemplated that a zero-payment plan is acceptable.

e. Modification and dismissal of CH 13 planso CH 7 liquidation involves only the liquidation of already-acquired

assets. By contrast, CH 13 relies on projections of future income and living expenses to extrapolate the amount the debtor can pay. The

42

Page 43: Bankruptcy Outline - Pottow 2011

projections may not work out, so the code permits the debtors or creditors to modify the plan.

o 2005 amendment specially permit a modification to permit the debtor to purchase health insurance. §1329(a)(4)

o 2005 amendment require the debtor to file annual financial updates if the judge or any party in interest requests them. §521(f)(4)

Problem Set 15, p.3241. First, is CH 7 a better choice than CH 13? Does she have something to

keep? Second, for CH 13, 23K should be below the median, so we don’t care the means test here. We apply the “reasonably necessary” test for disposable income projections. Is the piano class or orthodontic threat the reasonably necessary

expense? What is the difference for the expense under the scrutiny under

CH 7 or 13?Third, we have a 3-year repayment plan. Fourth, the malpractice insurer encourages the lawyers to deduct more expenses. If the lawyer is too conservative to calculate the expense, the client may sue the lawyer for the malpractice.

2. First, Todd is over the median income, so he is subject to a five-year plan and pays the amounts equal to the surplus by the means test.Second, what is the surplus of the means test? We know that the means test allows the full deduction of the secured debts and here most of Todd’s debts are secured debts, so he likely passes the means test with a surplus below $100. For the below-$100 surplus debtors, we still judge the disposable income test: Whether he should send his children to the public schoolsThird, since he has a high gross income but only pays 3% to unsecured claims, is he an abuser of the CH 13? §1325(a)(7): CH 13 should be filed in good faith.Notes: It seems that the unsecured creditors will be screwed, so what is

the policy for this system? The Congress encourage the secured debts

§1325(a) (4) the best interest test is not very relevant here, because under CH 7 unsecured debtors will basically get nothing.

The means test can force some debtors from CH 7 to CH 13, because they are not qualified for the liquidation; but it can also induce some debtors from CH 13 to the CH 7, if they pass the means test, they don’t have to complete the 3 or 5 years plan and can get rid of the debts.

B. PMSI car can't be discharged at all. Unsecured creditors are

hurting. Chapter 13 supposed to be good for unsecured creditors, but this isn't always great

 There's a conflict in representing both

43

Page 44: Bankruptcy Outline - Pottow 2011

3. First, what is the applicable median income? One-person household or two-person household? His GF should not be counted in, though they live together. So we apply the one-person household median income. I think he will be over the median income.Second, which should he file Ch 7 or Ch 13? He will likely fail the means test with an income of $60K. So he has to file for Ch 13. Because his income is above the median, he has to file a 5-year plan. Third, for the means test surplus under Ch 13, is the medical expense a deductible expense? Yes, it is deductible. But here is different; here it is a debt, not the medical care expense.

4. First, what is her income for Ch 13? Her base salary or base salary plus the overtime payment? Her base salary is below the median; plus the overtime, she will above the median. So it is important to include the overtime or not.Second, CH 13’s earnings are projections of the future earnings under 1322(a)(1), very different the means test in CH 7 which counts the income from the preceding 6 months. For the future projections, why do we count the overtime in? If the debtor knows that the overtime payment will be distributed to the debtors, why does she work over time? If you represent her, you can argue that the overtime should not be included. So she is under the median income, and pays $400 disposable income to her debtors.Third, the debtor or creditors can modify the plan; the creditors can also request the debtor to file the financial updates to the court.

5. §1325(b)(2)(A)(ii): the charitable contribution should be less than 15%; here 425 per month multiplied 12 is less than 12% of 45K; but it may be made in bad faith; but the debtor can argue the contribution to the church is not made in bad faith, but just the timing problem.

6. First, her creditors may request to modify the planSecond, could it be bad faith if she keeps silent? She cannot keep silent, because she has to file the financial report every year. Finally, her best move is to tell the employer to postpone her income; but this may be regarded as fraud; so she can negotiate with the creditor to lower the payment, if not, she won’t take the job or postpone the salary.

CH 7 and CH 13 filing limito §727(a)(8): You cannot file CH 7 within 8 years from the former CH 7

discharge or CH 11 discharge; o §727(a)(9): You cannot file CH 7 within 6 years from the former CH

13, unless the repayment plan totaled at least 100% of the allowed unsecured claims 70% of such claims and the plan was proposed in good faith and

was the debtor’s best effort Note: Pottow thinks that good faith must be interpreted in the

context of the code

44

Page 45: Bankruptcy Outline - Pottow 2011

o It used to be that you can file CH 13 each year; but now the Congress impose the limit through §1328(f): The court cannot grant a discharge if the debtor has received a discharge. In a CH 7, 11 or 12 case during 4 years before this order of relief. In a CH 13 case during 2 years before such order.

B. Threshold Eligibility for Ch 13

Identifying types of claims

§109(e) the debtor has noncontingent, liquidated debts under the statutory maximums. IN RE HUELBIG , 2003, p.330

o Holding: The debtor’s debt is noncontingent, liquidated and exceeds the dollar limit, so the debtor is not eligible for CH 13.

o Facts: Huelbig operated a car body repair shop and defrauded the Allstate insurance claims.

o Contingent: If all events giving rise to liability occurred prior to the filing of the bankruptcy petition, the debt is not contingent.

o Liquidated: A claim is liquidated if it is subject to “ready determination and precision in computation of the amount due.” What is ready determination? Whether the process for

determining the claim is fixed, certain, or otherwise determined by a specific standard.

Choices between CH 7, 11 and 13o CH7: The means test.o CH 13: Disposable income test; noncontingent and liquidated debt

dollar limit.o CH 11: For the below the median income debtor, disposable

income test, 1129(a)(15); for the above the median income debtors, it could be applied to disposable income test under §1325(b)(2) or the IRS budget means test under §1325(b)(3). CH 11 needs at least 5 years payout; It needs the vote of credit and approval of the court

o The debtor may find themselves unable to access to any chapter.

Problem Set 16, p.33416.1

o §1328(a)(4) even if he can file a petition under CH 13, he cannot be discharged from the restitution for damages as a result of willful or malicious injury by the debtor that caused personal injury to an individual. So what is the value for filing Ch 13? CH 13 does have the value: Automatic stay. But is such filing in bad faith? No. He is entitled to the stay.

o For CH 13, there is a dollar limit of the noncontingent liquidated claims in §109(e): $307,675. Here is a judgment debt, so it is a noncontingent and liquidated claim. It seems that it falls squarely in

45

Page 46: Bankruptcy Outline - Pottow 2011

the limitation. So the way to solve is that he should first pay portion of the debt to lower it below the limitation, and then file for Ch 13.

o And CH 13 applies to the regular wage earners under §109(e). Is the firefighter a regular wage earner? Yes, we judge it from a long term.

o How about negotiating with the creditors? o What is the congress’ intent for the liquidated debt requirement? To

know how much debt he owes and whether he falls within the limitation.

o Question: appeal bond: to prevent the frivolous suits.

16.2o Under §109(e), the limit for noncontingent liquidated unsecured

claims is $307,675; the limit for noncontingent liquidated secured claims is $922,975.

o Is the guaranty a secured debt? Probably not. It is an unsecured debt.

o Is the guaranty a noncontingent liquidated debt? Still contingent; no definite debt yet. Unliquidated; waiting for the judgment.

o Since the guaranty debt is contingent and unliquidated, it should not be counted for the dollar ceiling for the filing under Ch 13, so she can file in Ch 13.

o Is the guaranty dischargeable in the bankruptcy case? Is the contingent debt dischargeable? See §101 the claims include the contingent debts; so it is dischargeable.

16.3 o File for Ch 13o Q: is he a regular wage earner?

16.4 o First, can he file for CH 7?

Eligibility: the means test §707(b). Would he fail the means test? Probably. He has consumer debt

and business debt. He may lose all his property though he can get the discharge He can retain some property by redemption, reaffirmation and

ride-througho Second, CH 11

Eligibility: below the median, the disposable income test §1129(a)(15); above the median, the means test or disposable income test

5 years repayment plan or longer need vote of creditor and approval of the court

o Third, CH 13 Eligibility: below the median, the disposable income test 1325(b)

(1); above the median with above $100 surplus, the means test; above the median but below $100 surplus, the disposable income test

3 or 5 years repayment plan can keep all his assets

o Finally, general advice

46

Page 47: Bankruptcy Outline - Pottow 2011

Whether the debtor is an assisted person in §101(3). As a lawyer, I am providing the bankruptcy assistance under

§101(4A). Abiding br the rules of §§526-528: Cannot advice the debtor to

take more debts; to advice the debtor to make untrue statement or disclosure, etc.

For Ch 11 and Ch 13: 1115(a) and §1306(a): The property of estate includes the property acquired and the earnings from services after the commencement of the case before the case is closed, dismissed or converted.

o If his income and debt are too high, so he should go to CH 11. CH11 has no 5-yr cap, and you can repay over 10 years.

Conversion from Chapter 7 to CH 13MARRAMA V. CITIZEN BANK OF MASSACHUSETTS , (2006)

o Facts: Marrama is now in the proceeding of Ch 7. It misrepresented the value of its property and the transfer several months before the filing. The trustee and the creditors wanted to take the property into the estate. M sought to convert to Ch 13. TIB and creditors contended that the request to convert was made in bad faith and would constitute an abuse of the bankruptcy process.

o Holding: 5 justices agree that the bad faith forfeiture the debtor’s right to convert.

o Dissenting: 4 justices. §706(a) gives the debtor an absolute right to convert from Ch 7 to Ch. 13.

o Even if the legislative history said it is an absolute right to convert. The court said no for the bad faith.

o Who is right? 5 or 4?

White’s paper: Personal Bankruptcy Law - Abuse Prevention vs. Debtor Protection

Earnings Y + mAssets XDebt discharge dCost cStigma s(Individual)

o Background: repeated filings and a stay problem; abuse of the bankruptcy law

o Opportunistic and non-opportunistic The direct cost is the same: attorney fee The indirect cost: stigma is different for both types The new bankruptcy law raises the direct cost for the debtors, so

the opportunists have more incentives to file for bankruptcy than the non-opportunists.

o Conclusion: the 2005 amendment led the bankruptcy law to a wrong direction.

C. CH 12 for family farmers and fishermen

47

Page 48: Bankruptcy Outline - Pottow 2011

IN RE MURPHY , 1998, p.329o Individual with regular income means individual whose income is

sufficiently stable and regular to enable such individual to make payments under a Ch 13 plan.

o It does not matter with the source of the income. So in this case, the payment from the unmarried partner is okay.

D. The consumer bankruptcy system1. Overview

o Tension between the fresh start and a pervasive fear of abuseo Two stages: the debt-incurrence stage and the post-incurrence stage

a. Theoryb. Policy debates

2. The evolution of consumer bankruptcy lawo 2005 favors the domestic support obligation; a significant number of

debtors were guilt of abuse and could pay.3. A comparative look

4. The consumer bankruptcy system before and after the 2005 amendments

o The discharge and fresh start remain the heart of the system but there are some important changes.

a. Abuse o One is beyond doubt an abusive, like repeatedly filing the bankruptcy

cases to use the automatic stay to block eviction by landlords, though the debtor has no intent of fallowing through the bankruptcy.

o Chapter 20 cases: first file for Ch 7 to discharge all the dischargeable debts and then enter into a plan under Ch 13 to repay undischargeable debts.

b. Recoveries from creditorso

c. Domestic support

d. Bankruptcy rateso The increased cost and delay to file bankruptcy may low the

bankruptcy rate. 2005 amendment requires vastly more paperwork, plus debt counseling before bankruptcy and post-filing financial counseling as a condition of discharge.

o The means teste. Chapter choice

o A few cuts on the attractiveness of the Ch. 13 than Ch. 7.f. Lawyers

o Require to use “debt relief agencies” in the advertisements.o Higher liability for the accuracy of the filing.

IN RE SAN MIGUEL , 1984, p.356o The lawyer helped the debtors to file a Ch. 13 plan to pay the

attorney’s fee over 16 months and only pay nominal $1 to each

48

Page 49: Bankruptcy Outline - Pottow 2011

unsecured creditor. (That’s why the lawyers help the debtors to file Ch. 13 rather than Ch. 7).

o The court held that it was not brought in good faith. g. Local legal culture

h. Conclusiono 2005 amendment is important but not fundamental.o U.S. has always had the most pro-debtor bankruptcy system in the

world and it remains so.

Problem Set 17 (not discussed in the class)17.4

o §109(e) the dollar ceiling of Ch 13.o §1325(a)(3) good faith and (b) disposable income test.o §1328(a): Not dischargeable for personal injury.o Whether the dividends of $275K from a spendthrift trust are the

property of the estate.o No matter whether the dividends are property of the estate or not, he

has a very high income, so if he files for bankruptcy in Ch. 7 or 13, the court may question whether he is in good faith.

o Even if he can file such a plan, the dividends are the disposable income, right? So he should commit the disposable income to the creditors.

o Notably, the debt for the personal injury is not dischargeable, so Frank cannot escape the liability.

o My advice is to file a petition as negotiation leverage and comprise with the creditors.

17.6o Loose the limitation on the access to the Ch. 13? o How about the disposal income?

Chapter 7 Liquidation

A. Introductiono Corporations are not dischargeable under Ch. 7, so they would

reluctantly file Ch. 7, while the individual persons usually file Ch. 7 rather than the repayment plan.

B. Business Liquidationso Reorganization bankruptcy is the central focus of modern business

bankruptcy law and practice.o Two conceptual points:

Ch. 7 is primal bankruptcy, the original creature. Many Ch. 11 cases end up in Ch. 7; liquidation analysis.

o Two practical points Ch. 7: collective nature and its national scope: not like the state

collection law, not a race of diligence for the creditors; TIB has the

49

Page 50: Bankruptcy Outline - Pottow 2011

flexibility to choose the best method to dispose of the property; avoidance of some transactions; orderly distribution.

Ch. 7 is most often the chapter in which an involuntary bankruptcy petition is filed.

C Initiationo Usually the debtors have a strong incentive to resist filing for

bankruptcy, and the creditors typically lack the necessary info to do so.

o Solution: the party with the best info, the debtor should open a bankruptcy proceeding while there is still substantial value in the company that can be distributed in liquidation or used to rescue the company in reorganization. The US law provides the debtors some incentive: DIP for business and discharge for personal bankruptcy cases.

D. Involuntary bankruptcyo Involuntary bankruptcy filing is very rareo Traditionally involuntary bankruptcy is always a Ch. 7 case but the

Code also permits a Ch. 11 case, §303(b); a creditor may put a debtor in Ch. 11 to obtain disclosure of important financial info and court supervision of the debtor’s activities.

o Protection on the debtors §303(i) grants attorney’s fees and costs (and sometimes punitive

damages) against an unsuccessful involuntary petitioner. §303(b)(1) three-creditor requirement

o two principal safeguards against an unjustified involuntary petition §303(b)(1) – three-creditor requirement Creditors allege and prove that the debtor had committed an “Act

of Bankruptcy” within a specified period prior to the filing of the petition.

o Act of Bankruptcy: two tests Equity insolvency test: Inability to pay: traditionally.

Disadvantage: It could force a debtor into bankruptcy merely because of the cash flow or liquidity problem.

No valuation problem for failure to pay. The harshness is ameliorated by the debtor’s absolute right t

convert to a reorganization plan. §706(a). Bankruptcy insolvency test: Liabilities greater than assets.

Advantage: it would have been protection of a debtor from involuntary bankruptcy so long as the debtor’s non-exempt property that could be attached under state law was sufficient to satisfy the creditors.

Difficult valuation problem. Generally not paying standard: the current law.

More creditor-oriented than the inability to pay. It also includes the debtor who refused to pay; cessation of

payment.

50

Page 51: Bankruptcy Outline - Pottow 2011

IN RE FABERGE RESTAURANT OF FLORIDA , 1997, p.377o A panel trustee is simply a TIB appointed by the UST from the

standing panel of persons who have been qualified to serve as trustees.

o Note that the restaurant would continue to operate in Ch. 7 until it could be sold as a going concern-a pretty surprising finding in the liquidation section of the bankruptcy code.

IN RE SILVERMAN , 1998, p.378o Filing an involuntary bankruptcy petition in bad faith.o The creditor has to pay the fees and costs of the debtor, and punitive

damages for bad faith.

Problem Set 18, p.38318.1

o As an attorney, how can you find that the debtor also missed other bills? How to make the debtor to tell you? UCC search for the personal credit; hire a private detective, credit

firms. Maintain the relationship with the clients: through the personal

approach. Lure the debtor: if you can prove it cannot pay the bill, we would

negotiate with this debt. Pre-Contract it.

o Pottow: Just sue it; discovery procedure.o The risk is that if the involuntary petition is dismissed, the creditor

may have to pay the fees and costs and the debtor, and also possible punitive damages if he is regarded as in bad faith.

18.2o I would like to ask

how many creditors he has; how much debt he owes; whether he can expect how many creditors would join in SSB’s

suit.o Alternative methods?

Grace period to pay: but it may be found by other creditors and force it to go bankrupt

Why not give SSB a lien silently? Fraudulent conveyance.o Equity insolvency; balance insolvency; I am not persuaded that he is

insolvent from the balance sheet §303(h). But §303(h) is a “generally not paying standard”, right?

18.5o Can Universal file the petition under §303(b)? How to calculate the

number of creditors? Insider? See §101(31). A case in the textbook: You cannot mechanically calculate the

number of the debtor, and we should care the policy behind the number.

51

Page 52: Bankruptcy Outline - Pottow 2011

Universal cannot count its subsidiaries; they may be regarded as one person.

o §303(h) generally not paying?o §303(i) - Shifting the expense; if you sue others wrongfully, you

shoulder the expense.o Is the debtor in bad faith? Laughing.

Fraudulent conveyance?18.6

o §303(a) – the debtor should be a person or an incorporated corporation. But the committee is not incorporated.

o Mon-commercial corporationo What is a moneyed corporation? o Pottow: I am afraid that they may not file against the committee?

18.7o §303(a) – the concern here is whether he is a farmer that is an

exception. But here he is not a farmero Is it bad faith for AFF to transfer his assets to his friends?

18.8o Good faith requirement o Bad faith – punitive damages?

Chapter 11 Reorganization

A. Introduction to business reorganizations1. Overview

o Haircut: the forgiveness of part debts.o Balance sheet reorganizations: no change of the business operation;

the unsecured creditor took the places of shareholders.a. The debtor in possession

o In corporate liquidation there are no exemptions and no discharge from debt.

o The incentives to do reorganizations: To retain the employees’ jobs. Save something for equity shareholders. Retain the management positions.

b. A brief history of business reorganizationo 2005 amendment treats the big and small companies differently. o The small business case as defined with a debt less than $2 million

will face: Increased reporting requirements Greater US trustee supervision Shorter deadline More drop-dead points at which they can be forced out of Ch. 11.

o Big business is also scathed in 2005: Suppliers have greater leverage. Reduce managements’ control of the reorganization plan process. Likely increase the cost of reorganization and make it more

difficult.

52

Page 53: Bankruptcy Outline - Pottow 2011

c. High theory and personal traumao

2. Traditional Ch. 11a. Mechanics of Ch. 11

o DIP runs the company, have most of the rights and duties of TIB and it acts on behalf of all of its creditors and itself. Ordinarily no TIB is appointed.

o Its use of the secured assets is limited and the creditors may seek to lift the automatic stay unless DIP provides adequate protection.

o Post-petition financingo DIP’s rights

§547 avoid the preferences- the payment or transfer to favored creditors within 90 days before bankruptcy

to avoid fraud conveyance §544(a) and §547 to set aside unperfected or late-perfected

security interests in the debtor’s property turnover power

o A creditors’ committee is appointed to scrutinize the debtor’s activities on behalf of all creditors and negotiate with the debtor. §1102.

o Plan for reorganization to pay each class of creditors a certain percentage over a period of time.

b. The logic of a traditional Ch. 11o A Ch. 11 petition is an invitation to a negotiation. Why is a court

proceeding necessary? Panic of the creditors. Not all creditors want to negotiate.

o Hope, sometime false hope, lies at the heart of the reorganization process.

o Added leverage for the debtor’s negotiation in Ch. 11. Automatic stay and its breathing room. A plan binding all creditors. Turnover and avoiding power.

c. The analysis of a Ch. 11 negotiationo 95% business cases with assets above $1 million go to Ch11 and more

than half the debtors with less than 100K went directly to Ch. 7. o Two implications following that many cases doomed to liquidation are

filed in Ch 11. The courts are the gatekeeper to dispose of such cases as quickly

as possible by dismissal or conversion into Ch 7. If the gatekeeping turns out to be efficient and timely, then it may

be plausible to measure the confirmation rate success of Ch 11.o Litigation trust

B. The automatic stay and adequate protection1. General consideration

o Automatic stay

53

Page 54: Bankruptcy Outline - Pottow 2011

o It is difficult to lift the stayo DIP has the burden to show adequate protection.

2. Scope of the stayo The stay must be nationwide and strictly enforced.

FARM CREDIT OF CENTRAL FLORIDA, ACA V. POLK , 1993, p.407o The court held that the pre-petition agreement to waive the automatic

stay to particular creditor was not enforceable.o Reasoning: the policy behind the stay is to protect the debtor’s estate

from being depleted by creditors’ lawsuits and seizures of property before the debtor has had a chance to marshal the estate’s assets and distribute them equitably among creditors.

o §362(b) provides a long list of exceptions to the automatic stay and the 2005 amendment substantially lengthened it. But the exceptions are often narrowly construed.

US V. SEITLES , 1989, p.409o Not allowed the governmental claims under the False Claims Act

escape from the automatic stay.

3. Lifting the stayo §362(d) three alternatives for lifting the stay

adequate protection

IN RE ROGERS DEVELOPMENT CORP. , 1980, p.415o Heritage has claimed two grounds to lift the stay

Lack of adequate protection The debtors have no equity in the property and the property is not

necessary to an effective reorganizationo Lack of adequate protection

§362 does not define the adequate protection §361 provides three alternatives for the adequate protection

periodic payment additional and replacement lien catch all provision

equity cushion could be one of the other means, however, valuation is need for determining the equity cushion fair market value is the proper method for the property

valuation conclusion: there is a cushion and the property may increase the

value over time, so there is adequate protectiono no equity and not necessary

4. Payments while the Ch 11 is pendingo How much the debtor must pay to provide adequate protection to the

secured creditors; only if the debtor cannot make the adequate protection payments does the litigation lead to lifting the stay.

54

Page 55: Bankruptcy Outline - Pottow 2011

o Whether an equity cushion or additional lien will suffice or whether the debtor must make cash payments to balance the decline in value is often a critical question to a cash-squeezed debtor.

o The creditor must be no worse off in a reorganization than it would have been if there were a liquidation. This means the creditor should get the present value; the creditors usually get the pre-filing and post-confirmation interest; the oversecured creditors also get the post-petition before the confirmation interest.

o Special adequate protection measure for the single asset real estate case SARE The debtors propose a workable plan promptly or to start paying

interest on the value of the collateral. Failure to do either would lead to lifting or modifying the stay. This applies to all SAREs.

SARE debtors can use rents to make adequate protection payments and mortgage lenders can get the interest rate set at the preexisting contract rate.

Payment in the bankruptcy proceeding:

o Post-confirmation, you got the interest; you got interest before the petition, but you did not get the interest between the petition and the confirmation, except

1) The oversecured creditors’ interest, collect the interest on confirmation or payment over the period.

2) Cash payment for the depreciation or replacement lien; not compensated for the time value, only for the depreciation.

3) Equity cushion: you cannot get the equity cushion.The valuation of the collateral is frozen, no depreciation. Can it be re-evaluated?

Problem Set 19, p.42719.1

o What is the value of the tools? Can we just use the tax depreciation? It is not relevant to the real value of the property, and it is only standard depreciation.

o Is the bank oversecured? Is there any equity cushion in the property? What is the possible depreciation during the reorganization?

o Under §§361 and 362, the bank can ask for the adequate protection or to lift the stay.

o What is the feasibility for the debtor to go through the reorganization? The bank may also want the money now and force the debtor into Ch 7.

19.2o The problem is that even if it won’t depreciate much, the debtor has

no equity on the jet. §362(d)(2). Is it necessary for the effective reorganization? I am not convinced

of this. So the creditor can ask to lift the stay under §362(d)(2).

o As the debtor, we can argue that the policy behind the adequate protection is that the value of the collateral may drop during the reorganization period.

55

Page 56: Bankruptcy Outline - Pottow 2011

We have the appraiser expert’s testimony that the jet won’t depreciate much if it can be maintained well. We can prove that we have a good maintenance on the jet.

o We would like to pay a small sum or buy additional insurance for the jet.

o The jet is dispensable for the reorganization

19.3o The filing for Ch. 11 will have an automatic stay on Sam’s litigation.

The debt to Sam is contingent and unliquidated, so he cannot ask for the adequate protection; and he cannot get the post-petition interest; it is also hard for Sam to ask to lift the stay. So how to fight?

o §1112(b) – San can ask the court to convert to Ch. 7, if Sam can establish that the conversion is not the best interest of the creditor and the estate.

o §1129(a)(3) – The reorganization should be proposed in good faith; here is not the business problem; zero problem about his business; it is not for his business, only for the litigation stay; so RRI filed in bad faith

o Who will determine the amount of the plaintiff’s claim? The jury or the court? No jury; the judge decides it. §105 – power of the court.

19.4o PMSI will be paid first. o 362(d) is the gatekeeper, screen for the Ch 11o §362(d)(1) HM can ask to lift the stay by lack of adequate protectiono §362(d)(2) HM can argue that the American Slide has no equity on

the collateral and it is not necessary for an effective reorganization. Slide rule: what is his reorganization plan? What is the feasibility

to be success? Is it filed in bad will? o Can the first creditor claim this property? Are the collateral

calculated together or separately? Yes, the first creditor can get the property. But if the debtor still has any equity, even if it is very small, the debtor can still keep the property, because the trustee has the control of the estate

o Pottow: the HM should petition to convert it to Ch 7. 19.5

o To lift the stay: §362(d)(3) against the background that too many debtors park in the bankruptcy as a free option, so file the plan or make the payment

o §362(d)(3) whether the debtor have timely filed a plan or have commenced to make a payment? The interest at the K rate

19.6o What is the nature of the payment? Interest? Compensation for the

depreciation or others? Or payment for the debt? o §506(b) oversecured claims.

C. Operating in Ch 111. Who is running the show?

56

Page 57: Bankruptcy Outline - Pottow 2011

o Debtor and creditors alike focus on the operation of the business as the central concern of the ongoing Ch 11.

o The debtor may have different interests with the creditors about the reorganization.

o If the dispute between the DIP and a group of creditors become really serious, the creditors may seek appointment of a trustee to run the business.

In re Sharon Steel Corporation, 1989, p.438o It is settled that appointment of a trustee should be the exception,

rather than the rule. o Facts: Here the DIP made some preference transfer, high interest

post-petition loans; low rental for the lease of the office building.o Holding: under §§1104(a)(1) and (a)(2) the totality of the

circumstances singled the need for a trustee. o Reasoning:

Gross mismanagement Conflict of interest, only an independent trustee can make a proper

investigation and determination of the best interest of Sharon/debtor.

Management in the reorganizationo 2005 amendment pushed the UST to get more involved. UST should

move for the appointment of a trustee if there are “reasonable grounds to suspect” the current management participating in “actual fraud, dishonesty or criminal conduct”. 1104(e) additional ground to the courts: if there is a reason to dismiss the

case, then the court may replace the DIP with a trustee if such a move is “in the best interests of creditors and the estate”

general ground §1104(a)(2)o it is not hard to understand to appoint a trustee in a case like Enron

but finding a trustee skilled in the particular business to be reorganized can be a daunting task. On the other hand, if the business will be sold, it is unclear how important to keep the old management.

o Middle option between DIP and TIB: examiner. §1104(c) The core function is to investigate the affairs of the debtor; to

provide a replacement for the investigation and recommendations of a trustee in an appropriate case

But recently the courts have ventured far outside the statutory guidelines in appointing examiners. The courts treat it as a sort of special counsel appointed to do some discrete job.

o For the public companies, the development of turnaround management (TM) specialists has further changed the dynamics regarding when and whether to seek either a trustee or an examiner. TM could serve as trustees, examiners and consensual replacement management

2. What happens to the Cash?o The Congress has placed few constraints on the DIP’s use of cash that

is not subject to a lien in the ordinary course of business; in contrast,

57

Page 58: Bankruptcy Outline - Pottow 2011

the debtor cannot use the cash that is subject to a lien without the permission of the bankruptcy court. §363

IN RE EARTH LITE, 1981, p.439o Sun bank has a secured debt of $350K by account receivable and

inventory. Sun Bank lent $75K more to the debtor Earth Lite for additional collateral, personal guarantees and periodic payments to interest and principal. EL defaulted the payments and the bank sought an immediate relief on the emergency basis and wanted to sequester all funds. EL Asked the court’s permission to use the cash.

o Holding: Due to the different nature of the proceeding and the different

collateral involved, the debtor cannot rest on the equity cushion but must offer more to the secured party before it is entitled to use cash collateral.

Personal guarantee is not legally sufficient to furnish adequate protection.

The bank is more than adequately protected if the debtor is required to resume the contractual payments and makes provisions for curing the default even without considering the value of the personal guarantee, so the bank is not entitled to a preliminary injunction and the debtor is entitled to use cash immediately.

Set offo §553: The availability of the cash is also subject to the bank’s setoff

with some qualifications and limitations. The exceptions: Congress worries that the creditors change its

status to secured creditors (preference); §553(a)(2)o A creditor with a right of setoff is treated as secured §506(a).

Technically the debtor with a right of setoff is not a secured creditor; it is an unsecured creditor.

It is a netting out rule; for efficiency.o The account subject to setoff is cash collateral and governed by

the rules set for the in §363(c).o The creditor is also subject to the automatic stay and may not

exercise a setoff without the permission of the court §362(a)(7).o It is very like a lien on the account; The bank can ask the court to lift

the stay;o CITIZENS BANK OF MARYLAND V. STRUMPF , 1995: The bank needs

the court’s approval to have the stay lifted before it set off; but the creditor could protect itself with an “administrative freeze” by holding that money in the checking account pending its application to the bankruptcy court for the lift-stay motion.

o The banks can only setoff the general account of the debtor, not the special accounts. There is substantial litigation over what constitutes a special account.

IN RE HAL, INC. 1996, p.445o HAL overpaid the exercise tax to the IRS. US sought to setoff the

overpayment against other debts of other agencies.

58

Page 59: Bankruptcy Outline - Pottow 2011

o To enforce a setoff right, a creditor must have a right of setoff under nonbankruptcy law and this right should be preserved under §553. The principal element of setoff is mutuality, which requires that

the debts are in the same right and between the same parties, standing in the same capacity and same kind or quality.

o Holding: US has a right of setoff under nonbankruptcy law, because “all federal agencies draw from or contribute to a common pool of money, the US treasury” (mutuality).

Lieno The proceeds of the collateral are still subject to the lien.o In bankruptcy, the creditors are nervous with the cash; commingling

problem-commingled with other cash; dissipate easily.o If the debtor wants to use the cash collateral, the debtor needs the

permission of the court; encumbered cash.o When the secured creditors start to exercise their rights, what do they

do? Take repossession of the collateral. But the repossession is not the ownership. What is the next stage? Foreclosure. Repossession of the car does not mean that you lose the car. Two different stages. Repossession is the collection behavior. It is limited by the

automatic stay; turnover power of the TIB or DIP;

Problem Set 20, p. 45020.1

o Yes, he can run the business in the ordinary course; but he cannot use the cash collateral as before.

o §363(c)(2) the debtor may not use, sell or lease cash collateral unless each entity that has security consents or the court, after the notice and a hearing, authorizes it. Definition of cash collateral §363(a).

o UCC 9-315(a) the security interest will continue in the collateral property and attach to any identifiable proceeds of collateral.

20.2o It is cash collateral, o Why does the bank want the compensating balance? Like a security

interest; at least setoff interest of 10%, better than nothing The compensating balance debt is squarely within §553(a)(3):

Specially created within 90 days before the filing for the setoff while the debtor is insolvent. So the $15K cannot be setoff.

o Pottow: The bank should be allowed to set off $25K ($40k-15k).

20.3o As a general rule, the bank can set off $50K and the debtor can use

other $11K. But under §553(a)(2) the creditor cannot setoff the debt that was transferred to the creditor after 90 days before the date of filing and while the debtor was insolvent. We can argue that Gretchen is insolvent.

59

Page 60: Bankruptcy Outline - Pottow 2011

o So the bank cannot set off at all. 20.4

o You should investigate the rumors before the negotiation financial decision market decision: sister company fiduciary duty to this company.

o The solution is to appoint a trustee; what is the trustee’s obligation: to investigate the debtor’s affairs it is like a fraud conveyance whether the current management committed fraud, dishonesty,

incompetence or mismanagement.o What is the difference between the trustee and examiner? Why does

the examiner need to investigate some debtor’s affairs? In case of trustee, the trustee take place of DIP, so no DIP anymore; in the case of examiner, the examiner does not displace the DIP. Someone committing the fraud, you appoint the trustee; If you bring the petition to appoint the trustee, can you get your

money back if you withdraw the petition? Maybe not.o §1104 (c)(2) mandatory appointed trustee or examiner.o §503(c)(2) the severance fee should not be paid to an insider of

the debtor unless the payment is part of program that is generally applicable to all full-time employees and the …

3. Post-petition financingo The source of new financing:

current lenders specialized high-risk business loans.

o §552(a) pre-petition security interests do not attach to property acquired by the DIP after bankruptcy, even the secured debt has an after-acquired property clause. E.g., the creditor has a security interest in the inventory of the

debtor. At the time of filing the creditor can lay first claim to this inventory. But inventory acquired after the filing will be property of the estate, no longer subject to the secured creditor’s after-acquired property clause.

In practice, the creditors often use UCC 9-315 to trace back to the proceeds of the old property used to buy the new property, so they can claim a continuing security interest. This is by way of proceeds, not by way of after-acquired property.

IN RE GARLAND CORP. , 1980, p.453o The court approved a post-petition borrowing secured by

unencumbered property. An unsecured creditor challenged. The evidence did not demonstrate a reasonable likelihood that the

debtor could be successfully rehabilitated. Adequate protection to the unsecured creditors for new borrowing

secured by the unencumbered property. It was an abuse of discretion for the court to approve the new

borrowing secured by the unencumbered property without first

60

Page 61: Bankruptcy Outline - Pottow 2011

determining that the debtor was unable to obtain unsecured credit, as required in §364(c).

The court’s argument: There is a reasonable likelihood that the debtor could be

successfully rehabilitated. No special code section giving unsecured creditor adequate

protection. The permission of the new borrowing was done under §364(c)(2).

IN RE HUBBARD POWER & LIGHT, INC. , 1996, p.455o Sometimes the post-petition financing pushes a pre-petition secured

creditor out of the way.o The creditor got a super priority administrative claim under §364(c)

and under §364(d) a priming lien on all of the debtor’s assets, as well as an assignment of and a first priority security interest in all revenues the debtor receives from the sale of electronic power. The pre-filing secured creditors objected to it.

o The court granted the super priority lien. The debtor has not been able to obtain credit without priming its

senior liens. Adequate protection: without the new financing to clean up the

plant, the property is presently unsalable or has a nominal value at best; the investment to clean-p will result in a benefit not only to the debtor and its estate, but to all secured creditors and parties in interest.

SHAPIRO V. SAYBROOK MANUFACTURING CO., INC. , 1992, p.462o Two most important and controversial issues:

Cross-collateralization: should a creditor’s pre-petition position (often unsecured) get promoted in exchange for post-petition financing?

Mootness: §364(e) encourage post-petition lenders by assuring them that their rights will not be upset by an appeal of the order that gives them security or a priority, leaving them with a very high-risk loan. The new creditor argued that the appeal is moot because of the §364 authorization of the cross-collateralization. The holding is not moot.

o Holding: cross-collateralization is inconsistent with bankruptcy law for two reasons: It is not authorized as a method of post-petition financing under

§364. It is beyond the scope of the bankruptcy court’s inherent equitable

power because it is directly contrary to the fundamental priority scheme of the bankruptcy code. §364(c) and (d) apply only to future, post-petition extensions of credit.

Appeal under §364(e)o If the court authorized to obtain credit or incur debt, or granted a

priority or lien, the appeal cannot affect the validity of any debt so

61

Page 62: Bankruptcy Outline - Pottow 2011

incurred, or any priority or lien so granted to an entity that extended such credit in good faith.

o 364(e) no jurisdiction for the appeals; they assume the underlying judgment is correct

Problem Set 21, p.46821.1 (not discussed)

o §364(a) Borrow money as an administrative expense.o §364(c) request the court to authorize TIB the obtaining of credit with

priority over any administrative expense lien a junior lien.

o §364(d) super priority security interest or equal lien. The trustee is unable to obtain such credit otherwise. There is adequate protection of the existing secured debts.

21.2o §726 (b) the administrative expense before the conversion has a

priority over the after-conversion administrative expense. According to §503(b)(4) the administrative expense includes the attorney’s fee. Under §507(a)(2) the administrative expense has the priority.

o The debts: Pre-petition creditor: FSB $500K secured by inventory worth

$250K + §362 account receivable worth $0; $250K remained. Post-petition creditor: Hanratty $250K secured by equipment

worth $150K; $100K remained. Ch. 11 counsel $150K Ch. 7 counsel $50K Open asset is $350K

o Priority: FSB has a super priority under §507(a)(2)(1/2) and get $250K right

away, see §507(b) Ch. 11 counsel and Hanratty have the same priority under §507(a)

(2), so they share pro rata the remaining $100K; Hanratty got the status of administrative expense under §503(b)(3)(D).

Ch. 7 counsel got nothing.o Note:

Here the attorney’s fee got the priority, because it is the administrative fee. §330(a)(1) only means the debtor’s attorney, while here is the trustee/DIP’s attorney.

§726 distributions: we pay the liquidation expense first. §364(c)(1): the judge can give the post-petition creditors the super

priority.

21.3 (not discussed)o Murphy proposed to make a lower interest loan to VEI conditioned

that all his loans are secured by all VEI’s assets. It is hard to get the approval of the court for this cross-collateralization.

o §364(c) if the debtor cannot get a loan by granting the administrative expense status, it can set up a new lien on unencumbered property the court is likely to approve it

62

Page 63: Bankruptcy Outline - Pottow 2011

If the YSB trying to leave, it will enhance the chance for the court’s approval.

21.4o §503(b)(9) the administrative expenses include the value of any

goods received by the debtor within 20 days before the date of commencement of a case under this title in which the goods have been sold to the debtor in the ordinary course of such debtor’s business.

o §546(c)(1) the trustee is subject to the right of a seller of goods that has sold goods to the debtor, in the ordinary course of such seller’s business, to reclaim such goods if the debtor has received such goods while insolvent, within 45 days before the date of the commencement of a case but such seller may not reclaim such goods unless such seller demands in writing reclamation of such goods not later than 45 days after the date receipt of such goods by the

debtor no later than 20 days after the date of commencement of the case,

if the 45-day period expires after the commencement of the caseo §546(c)(2) if a seller of goods fails to provide notice in the manner

prescribed in §546(c)(1), the seller still may assert the rights contained in §503(b)(9).

o When is the commencement of the case? Three weeks ago; here National did not provide the notice under §546(c)(1), so they have to use §503(b)(9); but here it was beyond the 20 days limit, so National cannot reclaim the parts.

o Reclamation is a right between Possession (holds) and Title (owns) defined in the UCC article 2.

o Reclamation is also subject to the automatic stay, right? So the sellers need to ask the court to lift the stay under §362(d)? both yes

o §546(c)(1) requires that the debtor is insolvent when he received the goods; two dates are important: 45 days look-back window to receive the goods 45 days or 20 days demanding window. It depends on when the

bankruptcy is filed. I think if the seller sold the goods to the debtor before 25 days before the filing, he has 20 days to demand reclamation after the filing; if within 25 days, he may have more than 20 days calculating the 45 days.

o If the debtor received the goods after the bankruptcy filing, then the sellers get the administrative expense priority under §503(b)(3)(D); the sellers have no reclamation rights.

o §503(b)(9) also stipulates the reclamation rights, and the sellers don’t have to go through all the requirements.

D. Reshaping the estateo Avoiding powero Pre-bankruptcy “debtor” and DIP after the petition: DIP has not only

the rights of the debtor, but also the collective rights of the creditors

63

Page 64: Bankruptcy Outline - Pottow 2011

to preserve the business’s assets. So DIP can avoid some transactions that the older debtor cannot do.

1. The Strong Arm Clause – §544(a)o §544(a) gives the TIB the rights and powers as of the date of the

bankruptcy filing of a judicial lien creditor, an execution creditor, or, for real estate, a bona fide purchaser. Strong arm to knock off unperfected interests.

o §544(a)(3) for the real estate, a bona fide purchaser, much stronger than a lien creditor

o Lien creditor, secured creditor, and TIB. the same legal status lien is usually the judicial created; involuntary; Lien creditors are

always perfected. Secured debts are usually from the voluntary contracts. It could be

unperfected – attached lien but not perfected. IRS: The tax debts automatically attach lien to the property; but

unperfected though attached. IRS has to file the lien to be perfected.

TIB is the perfected lien creditor without regard to the knowledge.

The secondary secured creditorso §544(a)(1) and (2): Is the strong arm clause good news for the

secondary secured creditor? What is the remedy for the strong arm clause? §551 automatic preservation of avoided transfer: any transfer

avoided under §§ 522, 544, 545, 547, 548, 549 or 724(a) or any lien avoided under §506(d) is preserved for the benefit of the estate but only with respect to property of the estate.

TIB replaces the position of the first secured creditor; so the secondary secured creditor remains the same position.

UCC 9-322: Priorities among conflicting security interests in and agricultural liens on same collateral.

IN RE BOWLING , 2004, p.472o TIB alleged that the mortgage on the real estate was defective

because the notary was not present when the mortgage was executed as alleged. So TIB used the strong arm as a BFP to avoid the mortgage.

Notes: o The strong arm clause is a test of the strength of the lock that the

secured creditor or lien creditor has put on the property.Other property

o Where the dispute is over personal property rather than real estate, the TIB has only the lien creditor status of §544(a)(1)-(2); much less powers than BFP.

Federal tax lienso Before the tax liens are filed, it is treated much like an unfilled

security interest; the strong arm provisions permit the TIB to exercise the rights of a judgment lien creditor or bona fide purchaser of real

64

Page 65: Bankruptcy Outline - Pottow 2011

estate on the date of filing, which gives the TIB priority over the unfiled tax liens.

o After the tax lien is filed, the TIB must recognize the lien and treat the government as perfected secured parties.

Problem Set 22, p.47622.1

o 9-317(e) PMSI security interest has a look-back window with a priority over the lien creditor arising between the time the security interest attaches and the time of filing. Here the problem is that the security interest is not timely perfected; so screwed

o §544(a) it is likely to be avoided by the TIB.

22.2o Perfecting the security interest after learning about the bankruptcy

filing would usually violate the automatic stay; but o According to §362(b)(3) the automatic stay does not prohibit

the perfection of the security interest under §546 (b) or the extent that such perfection is accomplished within the period in §547(e)(2)(A)

o §546(b): The rights and powers of a trustee under §544 (the strong arm clause), §545, and §549 are subject to the generally applicable law that permit perfection. Generally applicable law means 9-317(e) PMSI 20-day look-back

window. Perfection under 9-317(e) also gets the priority of PMSI.

o §547(e)(2)(A) has a 30-day grace period to perfect a security interest in the transfer of real property. It is superior to a BFP. We do not care about the 20 days any more.

22.3o BFP gets the land free of mortgage or not? Get the land free of

mortgage unless it is recorded; mortgage is a serious thing, so lack of the signature may not beat the BFP; so TIB or DIP may have a power to avoid the defective mortgage.

K-MARTo The debtor paid the pre-petition debts of all critical vendors. The

district court authorized it under §105(e), but the appeal court reversed it. The debtor appealed.

o Holding: Cannot pay.o Reasoning: the debtor has to prove two things: if it does not pay the

old debts, the vendors will stop supply; non-favored creditors are not worse off. There is no evidence that the vendors will stop supply. Vendors cannot walk away because of the automatic stay on the

long-term K, if the debtor pays the current debts.The vendors who have no long-term K can walk away.

There is no record that shows other creditors are benefited. o Question:

65

Page 66: Bankruptcy Outline - Pottow 2011

Even if the vendors have the long term K, Kmart breached the K, can vendors walk away? Have no confidence on Kmart; can the priority help the debtor to get the credit?

What is the problem of the first day order to pay the critical vendors? If allowed, you encourage the hostage value.

o Code basis to pay the critical vendors There is no code saying the critical vendors in the list of priority

under §507, so where is the code basis? Is it covered by §105? You cannot use §105 to create the super

priority. §363 operation in the non-ordinary course of business; but here it

is arguably the non-ordinary business. §364 post petition financing: cash demand; condition the credit.

2. Preferences: Baseline Ruleo Besides the strong arm clause, TIB or DIP also has some new powers

including the power to dismantle certain transactions between the debtor and creditors that took place within the 90 days immediately preceding the bankruptcy filing.

o §547(b) determines what constitutes a preference. Made a payment to the creditor. The debtor was insolvent while the transfer was made. The transfer was made.

On or within 90 days before the filing Between 90 days and one year before the filing, if the creditor

was an insider at that time. The transfer enables such creditor to receive more than such

creditor would receive if The case was under Ch. 7. The transfer had not been made, and Such creditor received payment of such debt to the extent

provided by the provisions of this title.o Pottow: 7½ requirements

Transfer What is the transfer? It includes creating or perfecting a lien. Of the interest of the debtor in property. To or for the benefit of a creditor Antecedent debt Insolvent Look-back: 90 days or one year Better than the treatment under Ch. 7.

o Exceptions, see §547(c).

GILBERT V. GEM CITY SAVINGS ASS’N , 1981, p.484o Holding: payments to a fully secured creditor during the 90 days

preceding the filing of bankruptcy will not be considered a preferential transfer.

IN RE CALVERT , 1998, p.486

66

Page 67: Bankruptcy Outline - Pottow 2011

o The debtor was insolvent and paid money to the tort claimer and gave the security interest to his parent.

o Issue: whether there is an exception to §547(b)? Yes, the earmarking doctrine.

o Holding: The earmarking doctrine applies here. o The earmarking doctrine means that a voidable preference must

involve a transfer of an interest of the debtor in property. If the earmarking doctrine applies, no avoidable transfer is made because the loaned funds never become part of the debtor’s property. There is no preference if the new creditor is merely substituted for the old creditor. For the doctrine, the transaction has to meet three requirements: the existence of an agreement between the new lender and the

debtor that the new funds will be used to pay a specified antecedent debt;

performance of that agreement according to its terms. The transaction viewed as a whole (including the transfer in of the

new funds and the transfer out to the old creditor) does not result in any diminution of the estate.

o The earmarking doctrine applies only if the new and old creditors enjoy the same priority. Secured or not. The trustee bears the burden of proving that the earmarking

doctrine does not apply and thus bears the burden of proving the security interest exception does apply.

Notes:o The voidable preference provision’s greatest importance may lie in its

effect on pre-bankruptcy behavior. For many debtors, it is a primary motivation to reorganize through

Ch. 11 rather than attempting a workout outside bankruptcy. It also encourages the creditors to cooperate in informal workouts.

IN RE DENOCHICK, 2003, p.487o Indirect preference: §547(b)(2) the transferee and others who

benefit indirectly from the transfer are liable for the preference transfer.

o The appellant is the guarantor for the debtor. The debtor made a preferential transfer to a bank. The trustee tried to avoid the transfer and got the money back from the appellant.

o Holding: yes, it can.o Reasoning:

Guarantor is also encompassed in the “creditor” She is indirectly benefited from the debtor’s transfer to the bank,

because she was reduced the exposure to the possible obligation. Notes:

o Indirect preference is a big controversy.o §550(c) and §547(i) to fix this problem: insulate the non-insider

creditor from the effects of an insider benefit for transfers made more than 90 days before bankruptcy.

67

Page 68: Bankruptcy Outline - Pottow 2011

Voidable preferences at state lawo UFTAo UFTA only restricts transfers to insiders. o UFTA has a 4-year statute of limitations; while the bankruptcy code

reaches back only one year to review transfers to insiders. o §544(b) incorporates the state fraudulent conveyance law into

bankruptcy proceedings.

Problem Set 23, p.49023.1

o Yes. They are paid 100%, way better than 10%.23.2

o Half a month; it is a preference.o §547(f): for the purpose of this section, the debtor is presumed to

have been insolvent on and during the 90 days immediately preceding the date of the filing of the petition.

23.3o Yes, within 90 days.o Is it a transfer? Create a lien; sure it is.

23.4o Made the loan on June 1; perfected on July 1; filing for bankruptcy on

Sep. 15.o At first sight it is not an antecedent debt; and it is also not within 90

days.When did the transfer occur? Perfection or attachment of the lien? Attachment! But §547(e)(2): the transfer takes effect when it is perfected; so it is an antecedent debt within 90 days, so it is a preference.

23.5o Magic chef may not keep the repayment, because it is a preferential

payment.o Commercial bank may keep, because it is fully secured.

Oversecured, it cannot satisfy the fifth elements of (b).23.6

o The transfer includes creation of a lien.o For the $150K on March 1, though there is creation of lien, it is

beyond 90 days, so no preference.o For the $200K on April 15, is there creation of lien? Yes, so we have a

transfer on April 15 under §547(e)(3). There is a preference. o So TIB can avoid $200K.

23.7o There is a transfer to pay 20K, and is it a preference? It meets the

four elements of §547(b), but how about the (5)? What is the time to compare and say whether it is better than its position in Ch 7?

o We think that it should be judged in the bankruptcy liquidation, so it is a preference.

23.8o If there is an after-acquired clause, and the creditor is oversecured

again, so is there any preference?

68

Page 69: Bankruptcy Outline - Pottow 2011

o There are two voidable preferences: one is the payment of 20K; the other is the attachment to the newly acquired property.

23.9 o There is a preference, no doubt. Is there an exception like the

judicially created the earmarking rule? o Does the earmarking rule apply? It depends on whether forklift got

the lien in different state laws. If it got the lien, then the debt of the NY bank is secured or partially secured, and the debts to the Oregon bank and Granny is unsecured, so the earmarking rule does not apply.

o Even if the earmarking rule does not apply, for the NY bank, it got the exception of (c)(1) contemporaneous exchange. But TIB can avoid the preference to Granny and Oregon bank under §547(b) as an indirect preference. They were benefited from the preference.

o UFTA 5(b) if the transfer was made to an insider for an antecedent debt, the debtor was insolvent at that time, and the insider had reasonable cause to believe that the debtor was insolvent. §1(7) definition of insider; in this case, are Granny and First Oregon insiders? The former one is the relative, so being an insider; the later as the golf buddy may not be an insider.

23.10 o It does matter to file the petition right now. Even if the transferee is

an insider, TIB can avoid the preference within one year, but §547(f) does not make a difference to insiders, so the debtor is still presumed to have been insolvent for only 90 days. If the 90 days lapse, TIB has to prove that the debtor was insolvent when transfer happened.

o (not discussed for this part) It does not matter to file the petition now or later for the purposes of UFTA 5(b), as long as you file within the four years of the limitation of statutes. But practically it may have an impact. The property may dissipate if you file too late.

3. The exceptionso There are eight exceptions to the preference under §547(c); the

following is the most commonly used five: Contemporaneous exchange Ordinary course payments Purchase money New value rule Floating lien

Contemporaneous exchangeo (c)(1)

Ordinary course paymentso (c)(2)

IN RE STEWART , 2002, p.493o The debtor bough a cattle from the creditor and paid by the check;

but check was bounced; then the debtor bought a cashier’s check and paid the creditor. Soon the debtor filed for Ch 13.

o The creditor claimed the contemporaneous exchange exception

69

Page 70: Bankruptcy Outline - Pottow 2011

Though the check is an extension of credit, the law treats it as cash for the purpose of the contemporaneous exchange exception

But if the check bounces, it will ruin the exception; the creditor becomes a creditor in the pool, at most a secured creditor.

o The creditor also claimed ordinary course payment exceptions, which has three conditions The debt incurred in the debtor’s ordinary business course The transfer was in the ordinary course of the debtor’s business The transfer was made according to ordinary course terms In this case, the creditor failed the second and third conditions.

IN RE SHREVES , 2001, p.499o An earmarking defenseo Grace period for delayed filings under 547 is 30 days after 2005

amendment.o Facts: Shreves bought a car financed by United National Bank; he

refinanced it subsequently with Valley. After more than 3 months, Valley perfected its lien which fell within the 90 days before the bankruptcy filing.

o Issue: Valley claimed the earmarking defense.o Here the transfer at issue is the perfection of the lien, not the transfer

of funds to the United National Bank; in the perfection, there is no such transfer of funds involved, so Valley cannot rely on the earmarking defense.

Notes:o Valley could have protected itself completely by getting United to

transfer its security interest to Valley at the time of payment.

Purchase money exceptiono Grace period for perfection is 30 days.o The problem is that UCC still keeps 20 days; so the PMSI lender who

may be able to beat a trustee in bankruptcy on a voidable preference claim may have considerably difficulty beating a previous creditor with an after-acquired property clause.

New value exceptiono (c)(4) only “shelters” preference payments that come before a

particular extension of new value; applies only on a payment-by-payment basis

o The mechanics to use (c)(4), p.507 bottom

The other exceptionso (c)(6) statutory lien, associated with 545o (c)(7) alimony and support paymentso (c)(8) and (c)(9) permit creditors to keep value transferred to them

preferentially if the amount in question is less than $600 in consumer cases and less than $5K in business cases.

Problem Set 24, p.509

70

Page 71: Bankruptcy Outline - Pottow 2011

24.1o Contemporaneous exchange: three days interval between the delivery

of goods and payments; ruin the exception of (c)(1); counterargument: Is it substantially contemporaneous exchange? Only three-day intervals.

o It may be preference under (b).24.2

o July 1 signed the K and paid; July 12 identified the goods; July 28 delivered the equipment, and the debtor got the possession (c)(3); August 21 perfected the lien. Sep.30 filed for bankruptcy

o Under UCC art 4, when some common goods is identified for the contract, the buyers get the equitable title, which means the buyer will shoulder the risk. It is also the time that the debtor first acquire the right of property under §547(e)(3).

o As a rule of thumb, the transfer happens when it is perfected or possession; as an exception, under §547(e)(3), the transfer is made when the debtor has acquired the right in the property. In this case, it is within the rule of thumb.

o Though the perfection is later on August 21, there is a 30-day grace period from the possession on July 28 for the perfection of PMSI debt under (c)(3) and (e)(2), so there is no preference for Big Rig.

o If the bank has an after-acquired clause security interest, it may be different.

o Outside the bankruptcy, PMSI has only 20-day grace period to perfect its lien under 9-317(e); in this case, from July 28 to August 21, has exceeded 20 days, so Big Rig will lose to the bank as an inventory creditor. Inside the bankruptcy, PMSI has 30-day grace period, so now Big Rig can beat TIB; TIB can also beat out the inventory creditor, because it is a preference to attach the after-acquired property. However, the question arises that under §551, TIB takes the position of the inventory creditor, so can TIB now beat PMSI, since the inventory creditor can beat PMSI outside the bankruptcy? This is a circular priority problem.

o The answer lies where we look for the priority. Bankruptcy code does not give PMSI the first priority, so we have to look outside the code. So since the TIB steps into the shoes of the inventory creditor, TIB can beat PMSI creditor Big Rig. So finally the TIB gets the equipments.

o Note: Strong arm cannot beat the inventory creditor; preference can do it.

24.3o §547(b) preference; (c)(2) ordinary course payment; (c)(8) consumer

debts less than $600o UFTA 5(b) UFT to insiders within 90 days while insolvent; 8(f)

exception to 5(b): the new value exception; ordinary course; good faith rehabilitation

o Utilities bills: not ordinary course; see the three elements

71

Page 72: Bankruptcy Outline - Pottow 2011

o Mortgage: preference? Ordinary course? Mortgage usually is paid on time; so it is a preference.

o They just paid when the debt is due, so it is ordinary course???o Payment to the stockholder: UFTA 5(b)? To an insider, it may be

preference; but it may arguably be the ordinary course24.4

o Preferenceo (c)(4) new value exception o 1/3 beyond 90 days; no preferenceo 2/10 it is preference; but new value exceptiono 3/10 the same to 2/10o 3/20 and 4/1 preference but no exception

4. More on the exceptions: the floating lieno After-acquired clause conflicted with the preference clause. Grain

Merchants 1969 developed the “relation back” doctrine to avoid the preference. DuBay 1969 went further in protection of after-acquired clause. But these two cases may encourage abuse of the after-acquired clause.

o Comprise: §547(e)(3) and (c)(5) (e)(3) Rejects the “relation back” approach and recognizes

that the creation of the security interest in after-acquired property is a transfer within preference, so it is avoidable.

(c)(5) Grants an exception for inventory and receivables. It tempers protection with an “improvement in position” formula.

o Computation of (c)(5) On the 90th days before filing, the secured creditor was owed 13.4

million and its inventory worth 12.7 million, leaving a deficiency of 0.7 million. On the filing day, the secured creditor was owed 10.7 million, and the inventory was 10.8 million, so the creditor was oversecured 0.1 million. So application of the floating lien formula yields an improvement position of 0.7 million that is avoidable by TIB. And the creditor is also not entitled to the 0.1 million remaining equity. So TIB is entitled to get 0.8 million from the inventory.

Floating lien exceptiono Reduction in underwateredness is a preferenceo Fluctuation in overwateredness is not a preference

IN RE NIVENS , 1982, p.516o A mere increase in value of collateral, whether from a rising

market or a good rainfall, does not generate an improvement in position under 547(c)(5).

o Facts: the crops are inventories.

Setoff preferenceso §553(b): TIB can avoid the offsetting amount by which the creditor’s

setoff position improved during the 90 days before the bankruptcy.

72

Page 73: Bankruptcy Outline - Pottow 2011

o Avoidance power only applies if the creditor offsets before bankruptcy. The creditor who waits and exercises the setoff after the bankruptcy with the court’s permission under 362 is not required to surrender any improvement in setoff position obtained during the 90 days.

o There is a long line of exclusions from §553.IN RE WILD BILLS, INC . 1997, p.518

o The 90th days means the entire day.

Problem Set 25, p.51925.1

o May 14 file for bankruptcy; o On Feb 14, Chase is oversecured, no deficiency; on May 14, the debt

is 4.6 million, and the inventory is 5.2 million, still oversecured.o Whether there is a preference? If yes, is there an exception?o Pottow: it depends. If Chase pays the new inventory before the

inventory come in (no new value exception?), it may be a preference; but even if it is a preference, there is a floating lien exception which only looks at the 90th days and the date of filing for bankruptcy: whether there is an improvement in its position.

o Here there is no improvement in its position (No reduction in underwateredness)

25.2o Oversecured, no preference; floating lien rule

25.3o No. There is a preference but there is also a floating lien exception.

25.4o The gold price jumped 42%, so we assume that at the date of filing, it

is fully secured. It seems that the creditor was better off $200K. But the increase in value is not a preference.

o No transfer here, only appreciation; so no preference25.5

o Before bankruptcy you can only set off the amount in the bank at the beginning of the 90 day period. After bankruptcy, the court will let you set off everything in the bank that you are owed. However, if you wait for bankruptcy you take the risk that there will be nothing left in the account at bankruptcy.

5. Executory contractsa. The economic decision

b. The statuteo §365 deals the rules with the contracts surrounding rejections,

assumptions and assignment. Several important rules.o A K may not be assumed if it was terminated prior to bankruptcy

under non-bankruptcy law. IN RE KRYSTAL CADILLAC-OLDSMOBILE-GMC TRUCK, INC. , 1998, p.526

o Because the dealership is still in force, so the franchise K is an asset of the estate. So the trustee is entitled to cure any defects, and to

73

Page 74: Bankruptcy Outline - Pottow 2011

assume and assign the franchise for the benefit of all of the debtor’s creditors.

IN RE JAMESWAY CORP. , 1996, p.529o 365(a) authorizes a DIP to assume or reject, subject to the court’s

approval, any executory K or unexpired lease of the debtor. o The court held that DIP could assign the K, and held the profit sharing

clause void because it limited the debtor’s ability to realize the intrinsic value of the lease.

Notes:o §§ 365(g) and 502(g): rejection damages for any K are

calculated as a pre-bankruptcy unsecured claim, regardless of when rejection occurs.

o Landlords’ damages claims after the rejection are also capped and the landlords are still subject to the duty under state law to mitigate damages by re-leasing the premises as soon as possible.

o §365(c)(1): forbids assumption of a K that could not have been assigned by the debtor under nonbankruptcy law.

o A patent license is unassumable. Debatable.

Problem Set 26, p.52326.1

o We breach the contract and sell the oil for the new higher price. We can reject the contract under §365. Now the counterparty to the contract has allowed damages of a normal breach, but they are treated as an unsecured creditor.

26.2o Jameson can make the choice after the agency reveals the census; and

then Jameson can assume or reject the K.o (d)(2) gives the trustee or DIP enough time to make the choice to

assume or reject until the confirmation of the plan, unless the court specified a period of time.

26.3o §541(c)(1) makes the “no assignment” clause and “bankruptcy

termination” clause unenforceable; the K is till the property of the estate.

o 365(e): Bankruptcy termination clause will be void How about the no assignment clause? Unenforceable; void as well

o §365(c) is the exception to §365(a), the trustee cannot assume or reject the K if

1) Applicable law excuses a party from accepting performance from and rendering performance to an entity other than the debtor or DIP, and such party does not consent to such assumption or assignment.

2) Such K is a K to make a loan, or extend other debt financing or financial accommodations, to or for the benefit of the debtor, or to issue a security of the debtor

74

Page 75: Bankruptcy Outline - Pottow 2011

3) Such lease is of nonresidential real property and has been terminated under applicable non-bankruptcy law prior to the order for relief.

o In conclusion, Farr is fine.26.4

o Assume it and assign it to the third party o If the TIB assume the K and breach it, then the creditor’s claim

gets the administrative expense priority; if the TIB rejects it, the creditor only gets the general unsecured claims against the estate.

o §365(c) protects the creditor under the non-bankruptcy applicable law; it may make the TIB not able to assume or assign a K.

o (f) is a sword for the debtor. (f)(1): Invalidate the anti-assignment clause; Except in (b) and (c), the TIB can assume or reject the K even if the applicable law and K provisions prohibit, restrict or condition the assignment.

o (f)(3): Even if the K or applicable allow parties to terminate or modify the Ks on account of an assignment, the K may not be terminated or modified because of assumption or assignment by the TIB.

o How about the Union K? Is it different from regular Ks? Collective bargaining see §1113 rejection of collective bargaining agreements: TIB can reject and assume the K only with the provisions of this article.

26.5o §365(c)(2). No financial accommodation can be assumed.o Can you assume part of K? A Latin legal term says that you cannot

assume part of K; so possibly the entire K cannot be assumed? o Pottow: from the common sense, only the financial part cannot be

assumed, but you can argue it.

c. Extra-statutory constraints on the trustee’s right to assume or reject

o A debtor’s pre-bankruptcy contract rights are property of the estate under §541(a).

o §365 imposes some constraints on the TIB’s right to assume or reject a pre-bankruptcy K. TIB may not assume a K if its assignment is forbidden under

applicable nonbankruptcy law. (c)(1). TIB must cure or arrange to cure most defaults as a condition of

assumption. §365(b). TIB or its assignee must provide “adequate assurance of future

performance” (b)(1)(C), (f)(2)(B).o Additional extra-statutory constraint: executoriness.

Countryman’s test: material breacho A K under which the obligation of both the bankrupt and the

counterparty to the K are so far unperformed that the failure of either to complete performance would constitute a material breach excusing the performance of the other.

75

Page 76: Bankruptcy Outline - Pottow 2011

o Widely accepted Executoriness

o The K is still alive; if it is dead, you cannot assume it. o I want to assume the K; counter party alleges that the K is dead. o Material breach test is good for 99% Ks. But it may not be true in

option K. o Alternatively, functional approach: is it good for the estate? If it is,

assume it.

IN RE RIDODIZIO , 1997, p.537o The restaurant, Ridodizio, is the debtor; LLC is the shareholders of R;

two guys A and B are shareholders of R. LLC loaned to R money, so it was a creditor; LLC also has the option to buy R’s stocks.

o Three Ks Loan and lease

They can assume the lease; not the loan (waiveable for the creditor)The court said that there was no lease; part of secured financing

Option KThe option K is executory; the consideration to buy R’s stock is only $93R can reject the K under the functional approach

Shareholders agreementsIt is between A and B. How is R a party to the shareholders agreement? They want to reject it. To reject the non-competition KIt is a business decision; no court decision is made

o The functional approach: whether a K is executory is determined by the benefits that assumption or rejection would produce for the estate.

o Material breach test excludes the option K from the executory K; too exclusive

o Three classes of cases are prominent except the option K Covenants against competition The license K, where a debtor is an inventor K for sale of real estate where the seller is the debtor The second and third classes are addressed by the Congress in

§365(h)-(j), (n)o Two conceptual points to wrestle this problem

Specific performance Some cases construe K rejection in a way that seems to create an

avoiding power.

Problem Set 27, p.53327.1

o Filing a bankruptcy petition can stay the landlord’s collection act. After the filing, the TIB should assume the K.

o The landlord may make two arguments.

76

Page 77: Bankruptcy Outline - Pottow 2011

The lease is already terminated under (c)(3); counter: there is no such law or provisions in the K.

The debtor does not cure the defaults or provide adequate assurance; counter: buy the insurance back; a feasible repayment plan.

o Can we keep the lease? If we go into bankruptcy and the trustee assumes the contract by offering to cure (that is giving adequate assurance that cure will be made) then it appears we can keep the lease. §365(b)(1).

o (1)(A) note: excused from impossible cures to nonresidential property leases.

o A different problem about the diagnostic machine? What problem? Is it impossible to cure? No? So no special problem, right?

27.2o (h)(1)(A)(ii) FB can keep the K going even though it is rejected by the

lessor.o What is the remedy if the K is just rejected without regard to (h)? FB

then has only unsecured claims for breach of lease; under (h) FB can retain its rights under such lease

o §502(g)(1) the claim arising from the rejection under §365 should be allowed as the claim that had arisen before the filing.

o Under (h)(1)(B) FB can offset his damages against the rent but nothing more. In this case, he can use the farm free for next five years, but he cannot get $10K more ever year.

o If the landlord rejected the lease and assigned the land, the lease right under (h) cannot be eliminated.

Notes:o Why do we have the special rules for the lease under §365(h)?

The property is unique? Not fungible? Special transaction costs? Property rights under the lease?

o §365(n) if the debtor is a licensor of a right to intellectual property, the licensee may terminate the K retain its rights, the real remedy to keep the license; no similar cap

to the lease.

27.3o File today; the lease won’t terminate today; stayo Assign the lease, §365(e)(1) invalidate clause 1; o How about the clause 2? Assignment approval clause; the clause 1 is

vacated by 365(e); and the class 2 is vacated by §365(f). §365(e) invalidates some clauses conditioned on the bankruptcy filing or insolvency; §365(f) invalidates other provisions that restrict the assignments of the executory K.

o (f)(3): the requirement for the increase is not reasonable.27.6

77

Page 78: Bankruptcy Outline - Pottow 2011

o Under (d)(1), the attorney should not call the TIB and wait for the lapse of 60 days; then the K is regarded as rejected; so Paul may sell it for a higher price?

Summary of §365o §541(c) says these contracts get into the estateo §365(a) you can make a motion to assume or reject these contracts

under BJRo §365(b) you have to do some things to assume the contract, adequate

protection, cure defaultso §365(c) some things you just can't assume no matter what, e.g.

something that federal patent law says is unassignable, or personal service contracts

o (d) 60 dayso (e) invalidate some termination or modified provisions that are

conditioned at insolvencyo (f ) deals with the restrictions on the assignmento (h) more special rules: regarding to a lease

7. Fraudulent Conveyance and other state avoidance laws in bankrutpcy

o §544(b) makes UFTA and state fraudulent conveyance law an important part of the TIB’s tools for reshaping the estate.

o The code has its own FC law §548.o §550 provides the remedies for avoidance actions under §544(b) and

§548.

a. Transfers among related entitieso The REV test: a transfer is made when the debtor is insolvent

and lack of consideration of reasonably equivalent value. o The following cases illustrate that even if there is REV, it is given to

the wrong parties.

IN RE IMAGE WORLDWIDE, INC. , p.554o Steinberg established IM and IW; IM borrowed money from Parkway

Bank; IM also owed FCL. Steinberg borrowed money from Parkway and paid to FCL, secured by IW’s account receivable. When IW went bankrupt, the bank collected the assets of IW’s receivable. TIB alleged the fraudulent conveyance for the guarantee, because IW did not receive REV for the guarantee when it was insolvent at that time.

o Intercorporate guaranteeo Holding: the indirect benefits to a guarantor may be considered when

determining whether a corporation receives reasonably equivalent value for a guarantee. But by paying off IM’s debt, IW kept IM out of bankruptcy by bankrupting itself. This shift of risk is exactly what the fraudulent conveyance law seeks to avoid when applied to guarantees. So there is no clear err that the court held that IW did not receive the REV.

78

Page 79: Bankruptcy Outline - Pottow 2011

o Note: 2005 Amendment extends §548 limitation of statute to 2 years.

o Respect the corporate form; functional analysis

IN RE VIDEO DEPOT, LTD. , p.560o Arlynn was the principal of Video Depot, and also an active gambler.

He “borrowed” some money from VD to buy a cashier’s check to pay his gambling debt to LV Hilton. The TIB sought the fraudulent conveyance.

o Issue: whether LV Hilton was the initial transferee under §550(a)o Reasoning:

Both parties agreed that it was a fraudulent conveyance. For FC, the TIB can recover from the initial transferee absolutely;

it also can recover from subsequent transferees but upon to the exception that the subsequent transferee paid the value in good faith. §550(b).

What is the transferee? The person has dominion over the money or other asset, the right to put the money to one’s own purposes.

Hilton claimed that Arlynn was the initial transferee. The court held no, because since the casher’s check was only payable to LV Hilton, and Arlynn could not use it for other purposes and he also had no control, so LV Hilton was the initial transferee.

Hilton also claimed that the company ledger supported that it was a loan to Arlynn and then he paid to Hilton, so Arlynn was the initial transferee. The court held that so long as the money in VD’s account, Arlynn’s legal right was circumscribed by his duties to the company and its creditors; He was a courier, not a transferee.

o Initial transferee and subsequent transferee Why is so easy to avoid the transaction from the initial

transferee? It is strict liability. Policy concern: cheapest info gather and monitor

Subsequent transferees have the good faith defense In this case, should Hilton ask more?

b. The extraordinary powers of the TIB

IN RE BAKERSFIELD WESTAR, INC. , p.565o Saunders were the shareholders of BW and elected BW as a tax S

company (no tax for the company entity). Then Saunders revocated the S company status and elected it as C company (independent taxpayer). Shifted the tax burden from the shareholders Saunders to the company. TIB claimed that the revocation was FC. IRS was one of defendants.

o Holding: it was a FC.o Reasoning:

Whether it was a property? The ability to not pay taxes has a value to the debtor-company, so it is property

Is it a transfer? Yes, very broad definition Whether the tax code provision limits a TIB’s avoiding powers

under 548? No.

79

Page 80: Bankruptcy Outline - Pottow 2011

Note:o MOORE V. BAY : the TIB could avoid the transaction completely, even

though the resulting liability might be much larger than the claim of the creditor in whose stead the TIB sued.

Difference between UFTA and §548o 548 has a two years limitation of statute and UFTA: four yearso The TIB can use the state law under §544(b); federal law is

§548.o You can step into anyone’s shoes under §548? Broader.

Difference between the preference and fraudulent conveyanceo Preference: only favor some guys; o FC: give money away; o Some creditors may be sued both.

Problem Set 30, p.57230.1

o Who is the debtor? Be Well. o Is it a fraudulent transfer? What is the transfer? §548(a)(1)(B) no

REV + insolvent.o The company gave away $4K and did not get any benefit.o What is the remedy? Can Be Well recover from the cruise company?

Is Burt the courier or the initial transferee? He is the latter. So Be Well cannot recover $4K from the cruise company, because

the cruise company has the good faith defense.o Should we separate Burt and his family’s share of cruise? Can we

treat it as a kind of compensation for the management? So then the cruise company is an aider or abettor?

30.2o To file a petition in Ch 11? o Is it a FC? (a)(1)(B) no REV + insolvent;o Remedy: you reject the K and you cannot get the paid money back;

FCC will not issue a licenseo The debtor can apply for a new license for 1 billion; FCC cannot

discriminate it, see 525(a)30.3

o Is the first transfer between the debtor and Sure Fire a FC? Yes, it is, because of no requirement for good faith for the initial transferee.

o What is the remedy for TIB? He can get the building back or the value. Here the property went to the third party. Under §550, he can only get the value.

o Under §550(b) TIB cannot bring a suit against Eric Wang to recover the property or its value. Wang paid the value in good faith He did not know the voidability He was not the initial transferee..

o If Mr. Wang was in bad faith, the debtor can get the building back.

80

Page 81: Bankruptcy Outline - Pottow 2011

o How much can the debtor get for the value of the building from Sure Fire? The value at transfer or the value today? Not clear. UFTA says the value at transfer

o Improvement: Under §550(e)(1) Sure Fire was also entitled to the lesser of 1) the cost of improvement or 2) increase in value; here the lesser is $10,000 increase in value. Sure Fire can offset $100K, rather than $300K.

30.4. o To file an involuntary bankruptcy against Held.o TIB then avoid the auction for a FC under (a)(1)(B) no REV +

insolvent;o Then CI can be paid pro rata. o BFP says that it is a FC; it is not conclusive because not many people

showed up; open to the public sale o But the buyer can argue that at non-collusive auction (public

foreclosure sale) is not a fraudulent transfer. Sometimes this holding doesn't make much sense. But there it is. This isn't a fraudulent transfer.

8. Equitable Subordinationo §510(c)

If you are a stockholder of a company but you try to act like a creditor, you go to the back of the line

You just do something unfair; you go to the back of the line. More likely to be evil if it is an insider.

o first, the effect of this rule is to postpone or subordinate payment of a particular creditor’s claim until some or all other creditors who have been paid in full

o second, the most simple situation is that the owners of the company capitalize the company by little equity and most loans and the equity is obviously too small for the business Other than the basic situation, it is generally applied where there

is an allegation of wrongdoing and the wrongdoer is in some sense an insider.

The bank lenders have sometime been equitably subordinated when they have begun to manage the operations of their debtors.

o Finally, it is more asserted than applied. More useful in negotiation with insiders and with creditors who have overstepped the creditor role.

o The theory is equity Insider: you put too little money in business. Outsider goes into insider; more control; overstep; shadow

directors.

In re Carolee’s combineo The seed money creditors were given a high return and bore little risk

(guaranteed payment from initial cash flow of the auction business)

81

Page 82: Bankruptcy Outline - Pottow 2011

and so the advance of money was an investment of an equity nature and its return must be postponed until general obligations of the bankrupt are paid.

o Two twists on traditional equitable subordination The ownership or insider element was not control, but rather the

economic position in which the favored creditors put themselves. The second is the remedy to put the avoided creditor behind the

general unsecured creditors.

Contractual subordinationo A creditor typically makes an agreement with another creditor that it

will take payments after the other creditor. If it receives some payments by some bizarre chance, it will give to the other creditor.

o It is not unusual, particular when a debtor and existing creditors want the new infusion of cash.

Equitable subordination and state lawo Indenture trusteeo Negative pledge clause: a promise by the debtor to lender A that it

will not grant a security interest to another creditor unless an equal and pro rata security interest is granted to creditor A.

IN RE EPIC CAPITAL CORP. o Three requirements for equitable subordination:

The claimant engaged in some type of inequitable conduct The misconduct resulted in injury to other creditors and conferred

an unfair advantage on the claimant Equitable subordination is not inconsistent with the provisions of

the Bankruptcy Code.o Holding: the respondent is not an insider or fiduciary of the company,

so no subordination is allowed.Notes:

o TIB or DIP has the powers of avoiding preference (disgorge money received), equitable subordination (get paid last) and tort liability (pay money to the debtor who originally borrowed money).

o The affirmative recovery through a tort action is a powerful tool for negotiation.

John McGrath, The Lawyers Who Rebuilt Eurodisney, p.589o A de facto director (shadow director in UK) is an owner or lender who

exercises such control as to be treated as if it were a director, leading often to piercing the corporate veil or equitable subordination.

Recharacterization and the intersection of state and federal lawo The bankruptcy court may recharacterize a transaction, a property

right, or a K, looking through its legal form to its substance. o In re Vanguard Airlines: the court recharacterized a lender as a co-

venturer.

82

Page 83: Bankruptcy Outline - Pottow 2011

o Intersection of state and federal law; it is hard to distinguish and split; the bankruptcy court may recharacterize a right in state law in the federal bankruptcy law for some specific purposes.

Problem Set 31, p.59331.1

o First, file a bankruptcy petition that will stay the collection behavior of the bank.

o Second, the bank’s payments to some creditors may be avoided as preference.

o Third, the bank may be equitably subordinated, including the new loans

o Fourth, the bank’s behavior is tortuous, and TIB may bring a tort claim.

o The founders’ family’s loan for start-up money may not be paid before the payment to the other creditors.

Special Claimantsa. Introduction

o For environmental claims and the mass torts claims, the problems are when their claims arise and how (or whether) they can be resolved in a business reorganization.

Big liabilities o Distributional problem between the environmental claimants and

mass torts claimants and the trade and business creditorso Timing problem: these claimants often span the pre- and post-filing

period while the heart of the bankruptcy system is the sharp cleavage separating the debtor’s pre-filing past and the post-filing future.

b. Environmental claimso Principally responsible parties = PRPs

IN RE JENSEN , 1991, p.712o Whether the cleanup claim was discharged in bankruptcy; generally it

is dischargeable. But here the DHS asserted that its claim arose post-petition and therefore not discharged. Thus, the issue is when DHS’s claim arose for purposes of bankruptcy?

o Holding: it arose at the time of actual or threatened release of the hazardous waste, or based on the debtor’s conduct. So it incurred prior to filing, so it was discharged.

o Why do we care about when the bankruptcy claim arose? Post-discharge claims are not dischargeable. Between the filing and discharge: Intra-bankruptcy claims get the administrative priority; they are dischargeable.But which one is better for the intra-bankruptcy or post-discharge? It depends on how much will be paid in the distribution and the financial health condition.

83

Page 84: Bankruptcy Outline - Pottow 2011

-----------------------------------------------------Filing discharge

o What are the countervailing reasons? The damages arose post-discharge.

o Class actions

IN RE VIRGINIA BUILDERS , 1993, p.718o Is the environmental claim a general unsecured claim or entitled to a

special priority?o Holding:

Environmental claims have been conferred administrative status based on three grounds: 1) the timing of damages; 2) the provision of 28 USC §959(b) which direct trustees and DIPs to act within bounds of state law; 3) the judicially created exception found in Midlantic.

The courts have consistently construed the language of §503(b)(1)(A) administrative expenses broadly.

Courts generally hold that the entire remediation cost qualifies as an administrative expense in a hybrid of pre-petition and post-petition damages.

c. Mass tortso The bad news for a tort victim was that no distribution could be had

in the bankruptcy case as unsecured creditors; the good news was that the victim’s cause of action was also not discharged. If it is a corporate Ch. 7 case, the news is all bad. If it is a personal bankruptcy or a Ch. 11 corporate reorganization, it generally is good news.

KANE V. JOHNS-MANVILLE CORP. , 1988, p.733o Kane as the asbestos-related disease sufferer has no standing to make

a claim for the future claimants. o Reasoning:

Conflict of interest The bankruptcy court appointed the Legal Representative

specifically for the future claimants.

IN RE FAIRCHILD AIRCRAFT CORP. , 1995, p.738o FAC manufactured a plane in 1985. FAC filed for Ch 11 in 1990, and

was purchased by FAI with a K provision clear any liability on the assets. The bankruptcy court claimed the discharge and the validity of the sale K. In 1993, the plan crashed. The victims’ brought a suit against FAI.

o What is a claim? State law accrual test Conduct test Relationship or conduct plus test

84

Page 85: Bankruptcy Outline - Pottow 2011

o Holding: the order of sale did not insulate FAI, and this court lacked the jurisdiction to enjoin these claimants, because they did not hold bankruptcy claims as defined in this decision. Entered in favor of defendant.

d. Regulatory claimants

Problem Set 38, p.74838.1

o Keep the business going and do nothing; wait the bills and claims come in; then file for bankruptcy and discharge.

o If you are the government counsel, set up a lien on the land. Or force her into bankruptcy to make sure their cleanup is post-petition. Got the administrative expense priority.

38.2o Implication for Congress: conflict with the bankruptcy code?

Preempted by the federal law? To what extent? o Some state law clauses cannot be preempted by the bankruptcy code.

38.3o Buy the insurance policy o Buy some operation but not the entire company; §363 the court’s

approval of the property clear of any right.o Appoint a legal representative for the future claimantso §524(g)(2)(B) exclusive jurisdiction about the validity, application,

construction or modification of such injunction.

2. Confirmation

a. Reorganization structureo payments to the creditors

The lowest creditor class becomes the owner of the company by receiving securities in lieu of payment.

Issue warrants to creditors, the option to buy stock at a fixed price.o Often the best solution in a Ch 11 is sale of the company as a going

concern to some new investor, with the proceeds to be distributed to creditors.

o LBOo The eventual structure is often tax-driven.

b. Best interests of the creditors and feasibilityo Two requirements must be satisfied even if the majority of creditors of

each class approve the plan: the plan must be in the best interests of each individual creditor who does not agree to it and the plan must be found feasible even if every creditor agrees to it. §1129(a)(7) and (11).

o Feasibility goes to the likelihood that the plan will succeed-that the business will survive and prosper at least long enough to make the scheduled payments.

85

Page 86: Bankruptcy Outline - Pottow 2011

What is the policy behind the feasibility requirement? To avoid wasting time and money.

IN RE MALKUS, DEBTOR (2004), p.617o Although success does not have to be guaranteed, the court is

obligated to scrutinize a plan carefully to determine whether it offers a reasonable prospectus of success and is workable.

o A debtor’s past performance is one of the most important measures of whether a debtor’s plan will succeed.

o The motel’s past poor operating results show that the plan is not feasible and unachievable.

IN RE MADE IN DETROIT, DEBTOR (2003), p.620o A plan that is submitted on a conditional basis is not

considered feasible, and thus confirmation of such a plan must be denied.

o In this case, the plan is contingent on exit financing from Kennedy and there is no reasonable assurance that Kennedy will fund the loan.

o What is the problem about the conditions? Substantial review: what the conditions are and whether the plan is crazy. The creditors may have a divergence of interests; someone thinks

it is crazy plan, some not. Who do we worry about to be exploited in the reorganization? At

the expense of creditors to run business.

The best interest doctrineo The best interest rule applies to every individual creditor; the court

has to do a liquidation analysis. o To show at least as well as Ch. 7 what the creditors can get.o The minority creditors can be voted out; dissenting creditors’ private

property rights cannot be eliminated. Special protection in bankruptcy.

SK-PALLADIN PARTNERS V. PLATINUM ENTERTAINMENT , p.2001, p.624o Once there was an objecting creditor, the burden was on the

proponent of the proposed plan (the debtor) to show the liquidation value of the company in order to determine how much the creditor would get in a Ch. 7.

o Why did the creditors lose? Frivolous objecting suit; they did not really have an expert to say the value of property; they never put the testimony.

Problem Set 32, p.627o Phase 1: To negotiate; how about paying 60 cents per dollar? Don’t

forget Phase 1.o Phase 2: To fight

Is it a preference? $7.5 million underwater payment; only $2.5 million lien; other assets is fully encumbered and thus not available.

86

Page 87: Bankruptcy Outline - Pottow 2011

It is a preference.o Violation of the best interest doctrine

Under chapter 7, we avoid the preference and get the $7.5 million back.

Unsecured debts = $4 + $1 + $7.5 = $12.5 million The unsecured creditors can recover 7.5/12.5 = 60% The plan proposed to pay only 50%, so it violated the best interest

doctrine.

c. Classification and Votingo A class votes in favor of a plan only when it is approved by both

a simple majority in number of creditors and a two-thirds majority in amount of debt.

o Voting is by class. The debtor has some flexibility in designating the classes and in some cases debtor may do this with an eye to creating favorable majorities in each class, a kind of financial gerrymandering. Why do we vote by class? Worry about the different economic

incentives; like the fully secured creditors may not want the reorganization.

But fully secured creditors are not impaired and thus cannot vote. You can modify the (fully) secured creditors and thus they may be

impaired; like to extended the repayment period.o Confirmation: §1129(a) – 11 conditions

§1129(a)(10) at least one impaired class agrees with the plan. Why so? For the cramdown.

o Classification §1122 Inside the class, the claims should be substantially similar The Code does not say that substantially similar claims must be put

in the same class o Cram down

At least one class impaired agree with the plan Classification gerrymandering

IN RE US TRUCK CO., INC. , 1986, p.628o Facts: The debtor classified the employees (represented by the

Teamster’s committee) out of the unsecured creditors’ class. o Issue: Whether the Code permits the debtor to keep a creditor out of

a class of impaired claims which are of a similar legal nature and are against the same property as those of the isolated creditor?

o Holding: Yes.o Reasoning:

§1122(a) specifies that only claims which are substantially similar may be placed in the same class. It does not require that similar claims must be grouped together.

The prior cases show that the lowers courts were given broad discretion to determine proper classification according to the factual circumstances of each individual case.

The union employees have a virtually unique interest

87

Page 88: Bankruptcy Outline - Pottow 2011

The Teamsters committee is still protected by the requirement that the plan does not discriminate unfairly and is fair and equitable

o Two means to confirm a plan under §1129 (a) meet all the eleven requirements including (a)(8) which

requires all impaired classes of claims or interest to accept the plan

(b) meet all requirements in (a) except the (a)(8) plus two additional conditions: (b)(1) the plan does not discriminate unfairly and it is fair

and equitable (b)(2) the minimum requirements for a plan to be “fair and

equitable” vary from secured creditors, unsecured creditors and equity holders.

Notes:o Creditors accept the plan by classes not individually. This is highly

important to eliminate the incentive to hold out. o The individual creditor is protected by the best interest and feasibility

tests.o The statute is clear about two elements of classification – permission

to separate the small creditors into a separate class for convenience and prohibition on combining differently situated creditors into a single class. But it offers no guidance on when or if legally similar creditors might be allocated to separate classes.

o NBRC recommendation: §1122 should be amended to provide that a plan proponent may classify legally similar claims separately if upon objection, the proponent can show the classification is supported by “rational business justification.”

IN RE BERNHARD STEINER PIANOS USA, INC. (2002), p.633o The debtor classified the claims of consigned pianos as Class 4 and

other unsecured creditors as Class 6 though the claims of consigned piano were also unsecured. The floor plan lender, one of unsecured creditors, objected to this separate classification.

o Holding: It is fine.o Reasoning: Good business reasons.

The code does not require that similar claims must be grouped together. But also substantially similar claims are not permitted to be separately classified “in order to gerrymander an affirmative vote on a reorganization plan”

Here the court found that where a continuing relationship with an unsecured creditor who had a distinct interest in the debtor’s business was essential to the continued operations of the debtor, separate classification of that creditor was for a good business reason.

Critical venders Is it in good faith?

Notes:I. Classification and the single asset case

88

Page 89: Bankruptcy Outline - Pottow 2011

o A single-asset real estate case, SARE.o In SARE cases, the ability to group together small claimants to create

a separate, accepting class is crucial to the reorganization. But some courts are extremely hostile to such classification.

o 2005 amendment constrict the access to Ch 11 for SAREs. §362(d)(3)-(4).

II. Impairment o An unimpaired class is deemed to have accepted the plan. §1126(f).o Question: whether substitution of collateral of equal dollar value is an

impairment of legal rights.o The choice between the legal or economic impairment

Economic impairment test requires valuation of asset, which is very time-consuming and expensive. The drafters of the Code wanted very much to avoid it whenever possible.

Legal impairment is favorable but it cannot ignore the economic reality at all.

Tightrope: to create a class to vote for the plan but the class must be impaired to be qualified to vote.

IN RE PPI ENTERPRISES (U.S.), INC. , 2003, p.638o PPIE was the lessee of Solow’s property. PPIE filed for Ch 11. The

court held that Solow as an “unimpaired creditor”, so he was deemed to have accepted the plan.

o §1124(1) a claim is impaired unless the plan leaves the legal, equitable and contractual rights to which such claim entitles the holder of such claim.

o Reasoning: the court distinguished the plan impairment and statutory impairment. In this case, Solow’s claim was subject to the cap in §502(b)(6); it was impaired by the Code, not by the plan, though he might recover more without filing for bankruptcy.

d. Claims tradingo Why claim trading?

As an investment To get a seat at the negotiating table A way to buy companies

o Questions: Is vote-buying when only economic interest at stake acceptable? You can buy people's claims and vote them in bankruptcy.

Can you directly buy their votes? After 1965 it became illegal to buy votes in bankruptcy. 18 U.S.C. §152(6).

IN RE FIGTER LTD. , 1997, p.644o Figter had an apartment building and filed for Ch11. Teachers was

the lender and fully secured by the building. Figter proposed the building to be part of rent and part of dominion. Teachers was held fully secured so it could not vote against it. Teachers bought 21 unsecured claims. Fighter alleged that the purchase was in bad faith, and even if it was in good faith, the 21 claims should have only one vote rather than 21 votes.

o Good faith

89

Page 90: Bankruptcy Outline - Pottow 2011

The court is sensitive to situations where a company which was not a preexisting creditor, has purchased a claim for the purpose of blocking an action against it. The court regards this as an indication of bad faith.

Here is not the case. Teachers did not buy a small number of claims to hurt other claims; he offered to buy all claims in Class 3; he was a lender, not a competitor; he had good concern about the use of the building.

o Voting: a creditor with multiple claims has a voting right for each claim it holds.

Notes:o Are all purchases to further one’s own economic interests in good

faith?o Can buy claims to vote to pursue interests in capacity as a

creditor, cannot buy claims to pursue interests other than as a creditor, e.g. a competitor. Based on good faith requirement of §1126(e).

Hypoo I pay you $50 to vote against/for the plan. Is it okay?

Federal criminal law 152: A Buying the voting is illegal. Go to the jail.

o §1126(e) the court may disqualify any entity whose acceptance or rejection of such plan was not in good faith, or was not solicited or procured in good faith.

Problem Set 33, p.65133.1

o Unsecured debts for $5 million; your client has $1 million; the other $600,000; the other rest A and B have 3.4 million. Now we have two classes: class 1 is mortgage creditor, unimpaired; class 2 is unsecured creditors of $5 million including $0.3 million small claimants.

o If the mortgage is unimpaired, he is deemed to have accepted the plan. Cure the default. §1124(2).

o 1.6/5.0 < 1/3 the plan has been acceptedo We can create the class 3 consisting of small claimants; then we take

$0.3 million from accepting unsecured class of $3.4 million. Now we have only $3.1 million that accepted the plan, and the 1.6/4.7 = 37% > 1/3. We can vote against the plan.

o But how can we take small claimants out? §1122(b) allows the debtor’s discretion to take it out. So your client may not do so.

o We can say that the interest are substantial different; the small claimants are personal guaranteed. But the counter argument is that the small claimants are not treated differently under the plan. The personal guarantee is outside the bankruptcy.

o Is the personal guarantee in bad faith? Does it run afoul the bankruptcy code? Can you rely on §105? The prosecutor may argue that it is buying the votes.

o Preference: it is not the debtor’s guarantee; the principal is not the debtor.

90

Page 91: Bankruptcy Outline - Pottow 2011

o Can we use the best interest rule? Does the best interest rule cover the tax consequence to the creditor? Real economic welfare. §1129(a)(7) it does not.

o Feasibility challenge

§1129(c) rival planso The debtor has an exclusive power to propose a plan for the debtor for

a certain period under §1121.o Under §1129(c) the court must confirm only one plan. The court

shall consider the preferences of creditors and equity security holders in determining which plan to confirm.

e. Solicitation and Disclosureo The disclosure requirement can produce heated litigation.

i. Required disclosures o The following case offers a clear list of what info must be included in a

disclosure statement. IN RE MALEK , 1983, p.653

o The list Description of business A history of the debtor prior to filing Financial info Description of the plan How the plan to be executed Liquidation analysis Management to be retained and the compensation of the personnel

retained Projection of operations Litigation Transactions with insiders Tax consequences

o It is common that the plan is amended for many times

ii. The safe harbor ruleo The safe harbor rule exempts companies that make disclosure and

solicit votes from SEC regulations. §1125(e): no person connected with the solicitation of plan

acceptances and rejections is liable for a violation of the securities laws, so long as that person acts in good faith and in compliance with Title 11.

The purpose of §1125 is to avoid the impact of the strict disclosure requirements of the securities laws.

o §1125 provides the safe harbor for fraud liability, and injunctive liability.

o SEC continues to lobby from time to time for a role in bankruptcies to watch out for equity shareholder interests.

iii. Prepackaged bankruptcies o A prepackaged plan straddles a workout and an ordinary Ch. 11.

91

Page 92: Bankruptcy Outline - Pottow 2011

o In a prepackaged plan, the debtor and several key creditors have worked tout a refinancing structure for the debtor, often involving substantial forgiveness of debt, infusion of new capital, and a promise of future credit. To make the deal work, the debtor takes the corporation through Ch. 11, like compelling a dissenting creditor to come along by majority vote of each class, discharging contingent pre-bankruptcy liabilities, or recapturing substantial preferential transfers.

o The essential feature of prepack is the pre-bankruptcy negotiation with an eye toward moving the company into bankruptcy once the details are confirmed among the major players.

o Two questions: first, what law governs solicitations, SEC or bankruptcy law; second, will pre-filing solicitations be adequate for post-filing confirmations? §1126(b) answers both questions: these pre-petition votes are

effective in the bankruptcy proceedings so long as the pre-petition solicitation complied with all applicable disclosure laws and regulation, including SEC rules. If no such SEC or similar laws, satisfaction of §1125(a) solicitation requirements is enough.

If the plan proponent chooses to solicit post-bankruptcy, the ordinary §1125 solicitation slows down the case because the disclosure statement needs the court’s approval.

o Empirical studies show that prepack does not work well.o 2005 amendment facilitates pre-filing disclosure and solicitation,

keeping the fast-moving Ch 11 speeding right along. §1125(g) after the amendment, a debtor that began solicitation

prepetition may continue apace if the solicitation complies with applicable nonbankruptcy law.

o The real effect of prepack is not very clear.

Problem Set 34.1, p.661o Contends that the plan does not provide adequate info as defined in

§1125(a) under §1125(b); it is bad disclosure of info. The debtor did not disclosure the potential preference suit in the plan.

o But what is the remedy? It may ask the court to stop the debtor from bringing a preference suit.

f. CRAMDOWN

i. Absolute priority and the participation of old equityo Cramdown has to meet two additional conditions:

§1129(b)(1) the plan does not discriminate unfairly and it is fair and equitable.

§1129(b)(2) the minimum requirements for a plan to be “fair and equitable” vary from secured creditors, unsecured creditors and equity holders. (b)(2)(A), (B) and (C)

(1) Secured creditors ‒ Must receive the “present value” of their allowed secured claims, i.e. the full value of the collateral plus a market interest rate; the same to Ch. 13.

92

Page 93: Bankruptcy Outline - Pottow 2011

(2) Unsecured creditors and senior interest holders ‒ the absolute priority rule; different from Ch. 13 the disposable income test.

o New value “defense”Ch. 13

o Is there a cram down for Ch. 13 plan? Yes, see §1325(b): Cram down and the disposable income test.

o What do the secured creditors get in the Ch. 13 plan? Reasonable percentage; allowed to demand the present value; interest payment. See §1325(a)(5)(B)(ii). The secured creditors could be crammed down in Ch. 13. But the creditors still keep their lien on the collateral or proceeds.

o What is the protection for the unsecured creditor under Ch. 13 cram down? The creditors should pay the net income over the reasonable expenses.

o Is there a best interest doctrine in Ch 13? Yes, see §1325(a)(4).Ch 11 gets extra protection:

o For secured creditors, you get the full payment of your claims, not just the value of collateral, at least the present value of collateral. In Ch 13, the secured creditor can only get the value of collateral,

not the full claim. o §363(k): the lien holder may bid the collateral in the auction; use of

collateral; post-petition financing.o Hypo: Suppose the claim is $1000, and the value of the collateral is

$700. §1129(b)(2)(A): Secured creditors should get in aggregate the

payment for the full claim, but at least present value of the collateral.

In Ch 13, the creditors can only get $700. Questions about Ch. 13, see §1325(a)(5)(B)(iii)(II) “adequate

protection during the period of the plan.”

BANK OF AMERICA NATIONAL TRUST & SAVINGS V. 203 NORTH LASALLE STREET PARTNERSHIP , 1999, p.657

o Facts: the reorganization plan proposed that the former partners would contribute $6 million in exchange for the partnership’s entire ownership of the reorganized debtor, though unsecured creditors were not paid fully. There was also an exclusivity condition: only the old equity holders could contribute new capital.

o Issue: whether a debtor’s pre-bankruptcy equity holders may, over the objection of a senior class of impaired creditors, contribute new capital and receive ownership interests in the reorganized entity, when that opportunity is given exclusively to the old equity holders under a plan without consideration of alternatives.

o Holding: No. Sweat equity is not allowed; not worth of cash. Exclusivity is not allowed.

o Reasoning:

93

Page 94: Bankruptcy Outline - Pottow 2011

Douglas’ dictum for the new value exception in CASE V. LA LUMBER PRODUCTS .

Interpretation of “on account of” in §1129(b)(2)(B)(ii). The opportunity to buy the equity is an item of also property in its

own right. The old equity holders cannot get it at no cost. Two substantial reasons:

The plan itself: it vested complete ownership in the former partners immediately upon confirmation.

The confirmation procedure: no other party had an opportunity to propose a competing plan.

Dissenting:o Requirements of §1129(a) and (b) are identical for plans submitted

by shareholders and by others, except the requirement in §1129(b)(2)(B)(ii) that no holder of a junior claim or interest may receive or retain any property “on account of such junior claim or interest.” In every reorganization case, serious questions concerning the

value of the debtor’s assets must be resolved. We assumed this question had been correctly solved.

o Two substantial reasons for majority opinion cannot stand: If the partners paid a fair price for the exclusive opportunity, the

retention of 100% interest is not any more “on account of” their former position than retaining a lesser percentage might have been.

If the valuation question has been solved, both the former partners and the creditors are in the same position to buy the old interest.

NBRC recommendationo The first is to amend §1129(b)(2)(B)(ii) to make it clear that a junior

class could purchase a new interest in the reorganized debtor.o The second is to amend §1121 to provide that if the debtor moves to

cram down a plan that provides for the sale of an interest in the business to old equity then exclusivity should be terminated so that any party in interest can propose a competing plan.

SARE caseso A huge proportion of the classification and cramdown cases reported

at any level are SARE cases.o The SARE creditor has a special position: bifurcated claims.

Summary:o Negotiated consent is the essence of the Ch. 11 scheme, particularly

outside the SARE context.o The exclusive right to propose a plan and the possibility of cramdown

represent the leverage that the debtor brings to the negotiating table. o In most case, the key to retaining ownership is §1126, which controls

the voting process by which creditors approve a plan. The absolute priority rule comes into play only if the creditors are crammed down, that is, if the majority does not vote approval of a plan.

ii. Cramdown against the secured creditor

94

Page 95: Bankruptcy Outline - Pottow 2011

o Each secured creditor can demand the present value of its allowed secured claim as an absolute condition of confirmation. Only one additional element need to be discussed: the §1111(b) election.

o §1111(b) election is important as a method of blocking certain kinds of debtor plans even if it is not often formally invoked. Its primary use to date has been in SARE cases.

o §1111(b) election often applies in two circumstances: The first is the usual undersecured creditor that has both an

allowed secured claim and an allowed unsecured claim under §506(a).

The second is the nonrecourse undersecured creditor. o The first, undersecured creditor, See §1129(b)(2)(A)(i)(II).

The undersecured creditor can elect to waive any deficiency or unsecured claim and his participation in the voting; in return, the debtor is forced to pay the creditor full dollars that the creditor is owed.

The debtor is not required to pay the present value of the entire claim, only the present value of the collateral.

The debtor can stretch out an electing creditor’s payments over a long period and thus pay that creditor much less on a present value basis than the full claim.

o The second, nonrecourse creditor §1111(b)(1)(A) gives the nonrecourse creditor a full recourse

claim in bankruptcy. Such creditor can then assert the usual secured-unsecured claims or make the §1111(b) election.

It is like a preference, but it is a deliberate Congressional choice. Also gives the creditor a say under the absolute priority rule; undervaluation of the secured claim.

o Restrictions on §1111(b) election If the property is sold under §363, the creditor cannot use the

election. Sometimes the participation as unsecured creditor is more

valuable than the payment over time without an allowance for present value.

Hypos: $1000 claims with $700 collateralo Non-recourse loan

Under §1111(b)(2), if the election is made, the secured claim extends to the full claim of $1000.

How is it different from §1129(b)(2)(A)(i)(II)? Only if you make the election, you got the $1000 full secured claim, full payment of $1000 and at least $700 present value.

Without the election, under §1129(b)(2)(A)(i)(II), we bifurcate the claim as secured and unsecured portion. You got a secured claim of $700 and unsecured claim of $300. You will be paid at least $700 and as least the $700 present value. “the value, as of the effective date of the plan, of at least the value of such holder’s claim in the estate’s interest in such property”

New value exception

95

Page 96: Bankruptcy Outline - Pottow 2011

o New stocks; new capital contribution to buy the new stocks, so why not? But we are nervous about something – valuation.

o The judges will decide the value of the companies. How about that the valuation is wrong?

o Solution: It should be sold to the public; auctions

Problem Set 35, p.681o Are you afraid the cram down? Can they be crammed down? If they

are crammed down, what will happen? Your client was an unsecured creditor, and would get the equity of the company under the absolute priority rule.

o Assume you client got 20% shares of the debtor, and the old equity holders have to contribute new value to buy the company.

o How about the compromise? They can reach a comprise: the debtor pays more than 70% and the unsecured creditor can vote yes.

o Even if your client is crammed down, it cans still make the §1111(b) election. Then the debtor has to pay the full payment of the claims and at least the present value of the collateral.

o Question: If the bank agreed to less than full payment, how does it change the analysis? Two implications: the creditor may lower his claim, so the bank may be an unimpaired class; or the bank can be one class that is impaired but agree to the plan, so the plan satisfies the condition of the cramdown: at least one impaired class agrees to the plan.

g. Small business reorganization, p.696

One chapter or three? o Merged into only one chapter: Ch 11 in 1978.o 2005 amendment makes it more difficulty for the small business.

More paperwork and greater supervision from UST will drive up the cost of operating Ch 11.

Shorter deadlines and decreased discretion for judges will increase the leverage of the creditors.

These provisions are motivated by the concern to do it faster.

Who is a small business debtor?o A small business debtor is one that has aggregate noncontingent

liquidated secured and unsecured debts as of the data of the petition in an amount not more than $2 million. Debts to insiders or affiliates are excluded.

o Under 2005 amendment, no attorney can tell the debtor to run up the debts.

o The size of the debts is not the only criteria to be a small business. Only cases in which no creditors’ committee has been appointed or in which the committee is not sufficiently active and representative to provide effective oversight of the creditor will be tagged for treatment as a small business.

96

Page 97: Bankruptcy Outline - Pottow 2011

A well-advised debtor may have friends and family in the creditor body to keep the business out of the small business land.

The new rules for the little guyso More paperworko More reports like periodic financial infoo More monitoring from UST: an initial interview after filing as soon as

practicableo The biggest change is the exclusivity rules.

A small business has the exclusive right to propose a plan of reorganization for 180 days and the plan, whenever proposed, must be approved within 300 days of the bankruptcy filing or the case will be dismissed. §1121(e). Courts may not extend those periods unless the debtor demonstrates that it is “more likely than not” that the court will confirm a plan within a reasonable time.

The above limit draws real fire. It increases the creditors’ leverage.

o Ch 22: a small business returns to the Ch 11 again after the first filing is dismissed or the plan is not confirmed. At the second bite at Ch 11 within two years of the first filing it must prove to the court that its bankruptcy resulted from circumstances beyond the debtor’s control that were not foreseeable at the time the case was filed and that it is more likely than not that the court will confirm a plan. Otherwise, there is no automatic stay available to protect the debtor.

o More disclosure, limits on refiling are all good policies. Why don’t they apply to all business filing for bankruptcy? The lobby. For the big business, you have collective problems, so automatic

stay is important

Who will own the reorganized small business?o Sweat equity is not acceptable. o No value defense is not acceptable. o But for the small business, creditors may want to retain the managers

who are often the equity holders. So they can make a deal. o 203 NORTH LASALLE decision

IN RE MJ METAL PRODUCTS o The court held that the court had an independent duty to determine

whether a plan complies with §1129, including the absolute priority rule.

o The court also said that the Supreme Court has not specifically stated that a new value contribution by equity holders is an exception to the absolute priority rule.

An interesting question about classification

97

Page 98: Bankruptcy Outline - Pottow 2011

o If you classify the groups into two classes: one is trade creditors, paid fully; the other class is litigation creditor, paid $0. Is that in bad faith? Fair and equitable?

Problem Set 36, p.69636.1

o Incur debt? We can advise this. Agency exception only relevant to consumer creditors. How do we get more debt? Settle the lawsuit to increase our liquidated liability.

36.4o He wants a reorganization where he keeps his equity. But we can't do

that in cramdown. We can't offer sweat labor. So we have to buy the equity post-confirmation. LaSalle says there has to be an open auction.

o We have an offer for financing, but we have no unencumbered assets. Will the bank still lend to us?

o §363(f) lets us sell property of the estate free and clear of liens. We sell everything, including the X-ray machines. We can liquidate for $500,000. Happy smiles can sell all these products to THE DENTIST. The bank's loan will attach to the assets after the sale. The creditors will get the same amount anyway. They get the cash proceeds of the sold items rather than the assets themselves. This is better for the creditors.

3. The reorganization of public companieso The central distinguishing characteristic of public companies is that

management is separated from ownership.

Controlo The separation makes it possible that any stakeholders of the debtor

might exercise the DIP’s control through their influence on management.

Three categories of controlo First, some creditors may exercise control through obtaining security

interest or through making DIP loans conditional upon strict covenants in a first day order. SPIP, secured party in possession.

o Second is that management has interests of its own that creditors may be better able to satisfy.

o The third is the role of the valuation of the company’s worth in determining who will be in control of the wealth that emerges from a Ch 11 under a confirmed plan. The lower the valuation, the more power flows to those at the top

of the priority tree.

IN RE CHANNEL MASTER HOLDINGS, INC. , 2004, p.700o The lenders, secured creditors, objected the excessive payments to

the debtor’s and the committee’s counsels. They claimed that the lawyers performed many unnecessary services. The lawyers asserted that the service they rendered were in furtherance of their fiduciary

98

Page 99: Bankruptcy Outline - Pottow 2011

duty and were designed to enhance the possibilities of a recovery for unsecured creditors. The fact that they were not successful should not mandate denial of their fees. The court agreed with lawyers but it was a little bit excessive.

o The lenders also complained that the committee spent too much time on KERP. The court did not agree and held that it was reasonable.

o The lenders also complained that the committee’s counsels spent too much time on sale of asset. The court did not agree and held that it was part of their fiduciary duty.

How long will the debtor have to reorganize? o §1121(b) gives the debtor an exclusive right to propose its plan

within 120 days and to solicit votes to confirm the plan with 180 days. Both times are measured from the date of filing and overlap each other.

o But 120 days are not enough for complex business. Prior to 2005, the court can extend it indefinitely. But after 2005, §1121(d)(2) imposes a cap on judicial discretion of 18 months to propose a plan and 20 months to solicit votes.

o 2005 amendment also has a big impact on the exclusivity right. o The dynamic for the negotiation totally changed and it increased the

creditors’ leverage.

Liquidating plans and asset saleso The use of Ch 11 rather than Ch 7 for liquidation has grown greatly in

recent years. Sometimes the liquidating plan provides for the mechanism by which assets will be sold over time.

o Use §363(f) to sell the asset of a clean title, free and clear of all liens. o Chapter 11 - 1123(b)(4) liquidation; Chapter 7 – liquidation;

and 363 - sell the assetso What is the difference in selling under 363 rather than selling under

Chapter 11 plan? 363 only requires court approval. No class vote.

Management changeIN RE GENEVA STEEL , 1999, p.700

o The facts: the debtor proposed a plan to retain six executives and key employers with a big severance payment and emergence bonus. UST and Unions sued. But question is, was Union a party to this plan? It’s not, right? So

why can it sue? o Holding:

The court denied that the retention plan was a sound business plan without first having discussed with Union.

The court did not approve the severance plan because it demoralized other in the administration of the case in Ch 7. The severance plan was entitled to administrative priority but the plan must provide that in the event that the case is converted to a case under Ch 7, severance payments shall be treated as Ch 11 administrative expense claims, and shall be paid by the Ch 7

99

Page 100: Bankruptcy Outline - Pottow 2011

trustee as promptly as practically on a pro rata basis with all other Ch 11 administrative expense claims. The plan should also have a mitigation provision.

For emergence bonus, the court held the entire emergency bonus in stock was the better approach.

Treatment of equityo Traditionally, Ch. 11 in a number of publicly held cases has become a

mechanism for sale of the company as a going concern or a turnover of its ownership to existing creditors.

o One area in which equity owners have fought back is valuation.

IN RE LORAL SPACE & COMMUNICATIONS, LTD. , 2004, p.718o Facts: The Ch. 11 plan did not provide any recovery for equity

owners. So the equity owners sought to appoint an examiner. They claimed that the debtors were undervaluing many of their most valuable assets and the debtors and creditors’ committee were improperly colluding to depress the valuations of Loral.

o Why do the equity holders want an examiner? The evaluation of the company. To save portion of distribution. The creditors may cancel all old stocks.

o Holding: The bankruptcy court had no discretion to deny appointment of an examiner where, as here, the $5 million debt threshold is met and shareholders of a public company have moved for appointment of an examiner. §1104.

o Notes: The valuation problem is also a question of control.

Class notes:o Cram down is a threat for the equity holder: they may lose their

equity.o Can you make an argument of the best interest? To get what they can

get in liquidation; a valuable K. Can you assume a K in Ch 7 liquidation? Is the K the part of the estate? Yes, §365. Can you first assume the K and then converse into Ch 7 under

§1112? The creditors may not confirm the plan.o Absolute priorityo Hypo: dentist

The debtor wants to keep his business Under the absolute priority rule, what does he have to do to keep

his business under Ch 11? He has to buy the company by new capital.

How much should he pay? Valuation problem. Put it on the auction in the market, NORTH LASALLE .

Pottow: I don’t know how to buy it by $500. What can he do by the $500? The debtor sold the equipment, and trademark to the highest

bidder free and clear from any lien.

100

Page 101: Bankruptcy Outline - Pottow 2011

The debtor’s shareholder will buy them. Does this screw the secured creditors? No. they retain their lien on the proceeds.

Then cram down. o Summary:

Ch. 11 plan: Liquidate all business §1123(b)(4). Ch. 11 you got more time to do it.

Ch. 7 plan: Just liquidate Ch. 7 or Ch. 11: Sale under §363 only requires the disclosure. No

need to get approval of creditors; much better choice for debtors

Problem 37, p.708o Should he sell the company or not? (Should the board even pass this

decision off to management?) His fiduciary duty is to maximize duty for shareholders. If he sells now they get nothing. The bank wants to liquidate now so they get paid off in full before they lose more money.

o Who does he owe his duty to? Are we in the “zone of insolvency”? Delaware court held that in the field of insolvency, you can consider the interest of creditors. What should he do? Solicit other bids? We are not under Revlon

duty as of now.o What of his own personal gains? His job as now seems better,

although he has an offer to keep his job at the new company. Will he lose his signing bonus if they went into Ch. 11? §503(c). Is this only for payments inducing him to stay (i.e., post-petition)? Would his signing bonus before somehow fall under this? Does his pay count as an inducement? Does this function as an injunction on accepting or rejecting executory (employment) contracts?

o The intra-bankruptcy employment contract: seems to be part of the estate, but is this covered by §365? Can we invalidate it?  When the estate assumes the employment contract, is it incurring an obligation under the meaning of §503(c)?

o  Is there a §548 issue? Is this REV? Would be a transfer within 2 years under 548 in general and also §548(a)(1)(B)(IV). If not REV may be a fraudulent transfer.

o Conclusion: You can make the best interest challenge For §548(a)(1), maybe For §503(c), questionable

o Reggie: Please abide by your fiduciary duty: to maximize the shareholders’ interest

5. Post-confirmation: life after Ch 11o More problems than before confirmation o Claims by the post-confirmation debtor against other entities: res

judicatao Timing problem: §1141 discharges almost all debts up to the

moment of confirmation but the problem is when an obligation comes into existence.

101

Page 102: Bankruptcy Outline - Pottow 2011

o Notice problem: Due process requirement; a second occasion to determine whether certain creditors’ claims against the debtor have or have not been discharged.

o The claim by a third party, often an insider of the debtor, that the plan has discharged the liabilities of the third party as well as those of the debtor.

o Post-confirmation concerns also implicate the continuing jurisdiction of the bankruptcy courts after confirmation. If a tort action is going on, how does the bankruptcy proceeding

influence the action? See next section/chaptero What is the consequence of discharge injunction?

Does the bankruptcy extinguish the creditors’ rights? Yes. It is a discharge injunction

Claims are precluded by the bankruptcy . How to bring an action after the bankruptcy?

§1141 Effect of confirmationo §1141(b) confirmation vests all of the property in the debtor.o (c) except (d)(2) and (3), after confirmation, the property dealt

with by the plan is free and clear of all claims and interests of creditors, equity holders, and of general partners in the debtor.

o (d)(1) – the confirmation of a plan (A) discharges any debt that arose before the

confirmation(B) Terminates all rights and interests of equity security

holders and general partners provided for by the plan. (2) A discharge does not discharge a debtor who is an individual fro any debt excepted from discharge under §523. (3) The confirmation does not discharge a debtor if

(A) the plan provides for the liquidation of all or substantially all of the property of the estate;

(B) the debtor does not engage in business after consummation of the plan; and

(C) the debtor would be denied a discharge under §727(a) if the case were a case under Ch 7.

(4) …

a. Effect of confirmation: claims against the debtoro Whether a claim against the debtor survives confirmation and

discharge turns on timing and notice.

IN RE U.S.H. CORP. , 1998, p.752o The plaintiffs asserted that they are not bound by the US Home

bankruptcy discharge because they were known contingent creditors who were not given formal notice of the Ch. 11 proceeding. The notice of bar date.

o Holding: No, they are not known creditors.o Reasoning:

102

Page 103: Bankruptcy Outline - Pottow 2011

That discharge binds all parties presumes that all creditors bound by the plan have been given notice sufficient to satisfy due process. Whether a creditor received adequate notice depends on the facts and circumstances. Due process is met if notice is “reasonably calculated to reach all interested parties, reasonably conveys all the required info, and permits a reasonable amount of time for response.”

Reasonable notice: when a creditor is unknown to the debtor, publication notice of the claims bar date may satisfy the requirements of due process. If a creditor is known to the debtor, notice by publication is not constitutionally reasonable and actual notice of the relevant bar dates must be afforded to the creditor.

A known creditor is one whose identity is either known or reasonably ascertainable by the debtor. A creditor’s identity is reasonably ascertainable if that creditor can be identified through reasonable diligent efforts.

b. Claims by debtorsLiquidating trusts

o With a litigation trust, the estate contributes causes of action that the debtor had against other parties. The estate might be paid immediately for this property and the money distributed to creditors.

o The trust may get this money by selling shares to third parties or to some subset of creditors. The trust then litigates or settles the suits and whatever it recovers is distributed to the trust shareholders. Sometimes the estate receives a contingent percentage of whatever the trust ultimately recovers, with the trust responsible for funding the litigation expenses.

Res Judicatao While claims by the debtor are not discharged under §1141, other

legal doctrines may bar them.IN RE HOWE , 1990, p.757

o A plan is binding upon all parties once it is confirmed and all questions that could have been raised pertaining to such plan are res judicata. The test for res judicata requires that 1) the parties be identical in both suits 2) a court of competent jurisdiction rendered the prior judgment 3) there was a final judgment on the merits in the previous

decision, and 4) The plaintiff raises the same cause of action or claim in both

suits. o Issue in this case: Whether two suits involve the same claim for res

judicata purposes.o Holding: We cannot escape the conclusion that the Howes’ present

allegations merely asset new theories based on the same nucleus of operative facts that informed their earlier bankruptcy proceedings.

c. Discharge of non-debtors

103

Page 104: Bankruptcy Outline - Pottow 2011

IN RE ARROWMILL DEVELOPMENT CORP. , 1997, p.753o Facts: One creditor challenged the debtor’s reorganization plan that

discharged the equity holder from all claims against him. o Holding: The equity holder is not discharged. The court did not have

the authority to enter a discharge of nondebtor as prohibited by §524(e); the creditor did not consent to the release of the nondebtor.

o Reasoning: The circuit courts are divided over the issue of nondebtor

discharge. Three lines: First, the plan can discharge nondebtors even over the

objection of creditors Second, the bankruptcy courts may never discharge or release

a nondebtor. Third and majority view is that bankruptcy courts may

“discharge” or release nondebtors from their debtors only if the affected creditors consent.

§524(e) specifically limits the scope of the discharge. This court adopted the majority view.

IN RE BERNHARD STEINER PIANOS USA, INC. , 2002, p.763o Temporary injunction, conditions 1-4, p.o Channeling injunction: set up a trust for mass claimants; they recover

the damages from the trust first; an injunction remains in place to protect both the debtor and the insurance company.

d. Ch. 22o The Ch. 22 cases lead to a concern that the courts are finding

feasibility where it does not exist.

Problem Set 39, p.76639.1

o This case is like US HOME . The “plaintiffs” are unknown to the debtor. So the publication notice is enough. Their rights are extinguished by the confirmation of the plan.

39.2o Can he be released under Ch. 11? Can the corporation bankruptcy

discharge the principal? How about the plan itself extinguishing the obligations of the principal, instead §1141 exemption of the claims? What do we worry about?

o §524(e) discharge limits to the debtor. But is he released partially who voted for the plan?

o Is the release a fraudulent conveyance?o The deeper question is that whether the bankruptcy code can release

others than the debtor? Is it constitutional? o The courts hold that we have no authority to extinguish the principal,

but we can impose an injunction order until some years later or upon some conditions. See the piano case

o Three different approaches from ARROWMILL .

104

Page 105: Bankruptcy Outline - Pottow 2011

39.3o Is the debtor the agent of the charity for the $35K? Who owned the

money? Pottow: It is the charity’s money; so charity was a creditor.o Is the payment to the foundation a preference? It seems yes. Does it

have any exception available? Is it ordinary course of business? Maybe not. So there is a risk that the payment could be revoked as a preference. But the charity can challenge the deal as follows:

o The sale of all the claims. It could be illegal to sell some cause of actions? Some state laws illegalize the sale of cause of action for torts or crimes

o The sale could be a fraudulent conveyance under UFTA Sec.5 or 4. It is okay to sell the claims for preference.

o Here the remedy is to challenge the trustee’s sale to the insider based on the conflict of interest; it is an abuse of the trustee’s discretion to sell it to insiders, because otherwise the estate can recover more and the creditors can be paid more. We can sue the trustee for acting not in the best interests of the creditors by selling the preference claims.

o Hypo: If the trustee sells all claims to an insider, can a creditor bring a suit for preference? Under federal law, no. because the trustee has the exclusive right to dispose the property of the estate, claims being part of the estate. So the creditor has no standing to sue. But can the creditor sue under state fraudulent conveyance law? The state proceeding may be stayed by the federal bankruptcy proceeding and it may be vacated by the federal discharge.

o $10K is not equivalent value for all the claims. The charity as a creditor can challenge this transfer. The daughter may have defense? The trustee can say that the sale of claims are a sound business decision, because the daughter paid the fair value of the property, there is no preference, so the sale is not an abuse.

o Conclusion: it is a fraud for him to buy the claims to shield the claims against his daughter; it may be criminal; threaten to complain to prosecutors; so the charity can settle with him.

F. Ethical issues1. Compensation and disclosure

o The court has much more active role in supervising the attorney-client relationship than it does in ordinary litigation. The attorney represents DIP. Counsel can serve only with court approval. §327(a). Counsel’s fees should be approved by the court. §328(a), §329(b)

and §330(a). Only disinterested persons may serve as counsel. §327(a). The representation and fee arrangements must be disclosed

to the court and creditors. §329(a)o §330 excludes any compensation from the estate for the debtor’s

attorney. For attorneys representing individuals in small

105

Page 106: Bankruptcy Outline - Pottow 2011

bankruptcies, this provision forces them to demand payment up front. Counsels to DIP or TIB are not bound by §330.

IN RE LEE , (1989), p.768o LSY represented Lees and their company Seoul Corporation at the

same time. o Holding: The court has the power to disqualify counsel in both of

these cases, on each of the following grounds: 1) failure to disclose the retainer; 2) failure to disclose the parallel employment application; 3) the attempt to represent debtors with interests in actual conflict The court exercised its discretion and only disqualified one

representation of Lees. o Reasoning:

Failure to disclose the retainer is failure to disclose the compensation arrangement.

Failure to disclose the parallel employment application is alone a sufficient basis for disqualifying counsel in both cases.

Conflict of interest: adverse interest Counsel’s argument that the Lee would give up any claims he has

against the corporation. But the court held that Lee did not have such powers. Lee does not have the power unilaterally to waive a claim of his

estate against the corporation. The power belongs to the estate and Lee has a fiduciary duty to assert any claim.

It is too early to determine whether the corporation has any claim against Lees, its shareholders and president.

Inter-debtor claims are not the only grounds for conflicts of interest. Substantive consolidation.

Lack of financial resources is often a reason for waiving a conflict of interest. But an attorney must obtain a written waiver from the debtor, all creditors and the UST. No such waiver has been offered in this case.

IN RE FILENE’S BASEMENT , 1999, p.766o HD acted as the debtors’ counsel. TAC filed a motion for

reconsideration for the appointment of HD as debtor’s counsel. TAC alleged that HD was not a disinterested person. TAC is plaintiff and Filene is the defendant. TAC, as F’s competitor, also sues the counsel of the debtor. TAC sued F for conspiring to violate the non-competition agreement and sue the counsel for violation of non-competition agreement.

o Holding: the court held that there was an actual conflict of interest; vacate HD’s representation of the debtor; HD may file an application for reimbursement of actual and necessary expenses which has incurred.

o Reasoning: The purpose of §327 is to “ensure that all professionals appointed

pursuant to §327(a) tender undivided loyalty and provide

106

Page 107: Bankruptcy Outline - Pottow 2011

untainted advice and assistance in furtherance of their fiduciary responsibilities.”

The purpose of disclosure is to provide the court with info necessary to determine whether the professional’s employment meets the broad test of being in the best interests of the estate.

HD represented TAC in connection with the termination of McGowan’s employment; and HD also represented the debtor in connection with the hiring of McGowan. The debtor and TAC were competitors and their business plans were strikingly similar. Can F get over the non-competing agreement between MG with TAC?

Adverse interest test… Adverse interest pertains to “any interest, however slight, that

would even faintly color the independence and impartial attitude required by the Code and Bankruptcy rules.”

Small client argument: Though it is a small potato client to HD, it does not matter to the conflict of interest; HD does 90% of legal work for TAC and TAC relied HD as general counsel; HD has the duty of loyalty. It is not a good argument to say that TAC is not an important client. The judge will be against this argument.

o Notes: the attorney has to disgorge the fees paid by the debtor.

The problem of retaining a debtor’s long term general counsel as its bankruptcy counsel is complex:

o If the attorney sits on the debtor’s board, he may be sued.o If the debtor owed a lot of money to the counsel, counsel is a creditor.o In above two situations, the court may well find the firm disqualified

as bankruptcy counsel. In this case, the debtor has to educate the new attorneys.

IN RE MARTIN (1987), p.781o This case raises a conflict that is so central to the attorney-client

relationship: The attorney wants payment while the debtor does not want to pay. Can he take a security interest?

o VD was the counsel for Martins to reorganize their restaurant business. But they cannot provide the retainer of $5K. They bargained and arrived at a K that VD took the security interest in debtor’s house in Maine.

o Holding: We don’t find the grant of the mortgage to the law firm for its fees was impermissible per se. we hold that the court must assess the appropriateness of the mortgage against the backdrop of the litigation. Vacated and remanded.

o Reasoning: The lower court concluded that VD was not disinterested and

should not have been employed as attorney for DIP. §327(a) will not support, either by its terms or by its objectives, a

bright-line rule precluding an attorney at all times and under all circumstances from taking a security interest to safeguard the payment of his fees.

This inquiry is of necessity case-specific.Notes:

107

Page 108: Bankruptcy Outline - Pottow 2011

o Bankruptcy courts generally recognize the validity of retainer agreements. It must be disclosure as fee arrangement.

o The attorneys must seek court approval before they draw against the retainer as fees.

o How to get paid? Up-front payment in Ch 7. Retainer in Ch 11: Who owns the property in retainer account?

The debtor/client. But the counsel has the right to offset the retainer account. But the lawyer cannot draw the money from the retainer account.

You have to watch the retainer account money and your outstanding claims. If you outstanding claims exceed the retainer, you turn into a creditor; and then you may lose the position of counsel.

Some circuit courts are divided whether the counsel can be kept when he becomes a creditor.

Retainer: Sell the investment property and put cash in the retainer. How about the first lien instead of the cash? Can the attorney withdraw if the retainer is dry? It needs approval of the court.

2. Privilege and conflict of interestCOMMODITY FUTURES TRADING COMMISSION V. WEINTRAUB (1985), p.786

o Facts: The court appointed a trustee for CDCB. The trustee asked the former counsel questions about misappropriation of customer funds and counsel refused to answer some questions based on the attorney-client privilege.

o The question is whether the trustee of a corporation in bankruptcy has the power to waive the debtor company’s attorney-client privilege with respect to communications that took place before the filing of the petition in bankruptcy.

o Procedural history: 7th Circuit held that the trustee did not have such power to waive the attorney-client privilege before the filing.

o Holding: This court held that the trustee could waive this privilege.o Reasoning:

When control of a corporation passes to new management, the authority to assert and waive the corporation’s attorney-client privilege passes as well.

The legislative history also makes clear that Congress did not intend to give the debtor’s directors the right to assert the corporation’s attorney-client privilege against the trustee.

§542(e) was not intended to limit the trustee’s ability to obtain corporate info.

Code gives the trustee wide-ranging management authority over the debtor. In contrast, the powers of the debtor’s directors are severely limited.

It is often extremely difficult to uncover the inside fraud if we keep the attorney-client privilege.

Notes:

108

Page 109: Bankruptcy Outline - Pottow 2011

o It is divided over Justice Marshall’s analysis that the trustee is “successor management” as with any corporate takeover, while the other argues that the trustee only inherits the property of the corporation and the right to control that property, not the right to control its governance or to control any rights that it has in the bankruptcy independent of its property.

o Pottow’s questions: How is the privilege for the corporation different from its brain the

executives? Can you waive the privilege beforehand in individual bankruptcy?

Ch. 9 Domestic Jurisdiction

Jurisdiction o Subject-Matter Jurisdiction, which cannot be waived.o Personal jurisdiction, subject to waiver (called attorn).o Venue, Internal allocation rule subject to S-M and Personal

jurisdiction.

Here below are all the subject-matter jurisdictions:o JRX –Title 28 §1334o “jurisdiction” – abstention – §1334(c) – discretionary and mandatory

The federal courts should/may abstain the jurisdiction.o Allocation of federal power. Art. III v. Art. I. Found the power under

§157. Non-Art. III courts: Bankruptcy courts, immigration courts,

territory courts and courts of marshals. There are specialized Art. III courts: Court of international trade,

subject to the Court of Appeals. Art. I (i) territory; (ii) military, plenary jurisdiction; (iii) public

rights o In some situations, the Art. I courts make judgment, subject to appeal

to the Art. III courts; in some situations, Art. I courts are special masters and can only propose the fact finding and conclusions of law to the Art. III courts. §157 allows the district courts to have the discretionary power to refer to the bankruptcy judges.

o Jury trial, the 7th Amendment.

Proceedings (Only similar, not exactly right)o Arising Under – coreo Arising in – coreo Related to – non core

US bankruptcy code §1334 o Subject matter jurisdiction o 5 requirements for mandatory abstention under §1334(c)(2).o Abstention is rare. 90% cases alleging the abstention fail.

109

Page 110: Bankruptcy Outline - Pottow 2011

o If it is core proceeding in bankruptcy law, the federal courts cannot abstain; if it is non-core proceeding, we should first examine the mandatory abstention and then the discretionary abstention.

o §1334(a) - exclusive jurisdiction to district courto §1334(b) - concurrent jurisdictiono §1334(c) - (1) discretionary abstention; (2) mandatory

abstention.o §1334(d) - no appellate review for some abstentions

§157 o The bankruptcy court cannot enter the judgment for non-core

proceeding matters; only recommendation o (a) – District court may refer everything to the bankruptcy judgeso (b) – Bankruptcy judges may hear all cases under Title 11 and all core

proceedings arising under/ino (b)(2) – Examples of core proceedingso (b)(3) – Bankruptcy judge can determine whether something is

core. “Toehold jurisdiction,” court can always determine whether it has jurisdiction.

o (b)(4) – Non-core proceedings under §157(b)(2)(B) should not be subject to mandatory abstention.

o (b)(5) – Wrongful death and personal injury shall be tried in the district court where the bankruptcy court is.

o (c)(1) – Bankruptcy court can hear non-core proceedings but that is otherwise related to title 11. B judge submits findings of fact to district judge, and D judge makes ruling.

o (c)(2) – B judge can enter his own judgment for non-core if he has consent of parties

o (d) – District judge can withdraw a case or proceeding from B judge for cause, if there is enough non-chapter 11 law.

o (e) – B judge can conduct a jury trial subject to district court rules and parties' consent

A. The 1978 Codeo This chapter is an introductory exploration of the functions of

bankruptcy law and its proper place within the larger scheme of the law.

o Summary jurisdiction and plenary jurisdictiono Two questions intertwined

Is this dispute within the federal bankruptcy jurisdiction, as opposed to the general jurisdiction of a state court or the non-bankruptcy (federal question and diversity) jurisdiction of a federal court?

If the dispute is within bankruptcy jurisdiction, is the district judge the only one who can hear the matter or may the bankruptcy judges decide it?

o NORTHERN PIPELINE CONSTRUCTION V. MARATHON PIPE LINE , 1982

B. The 1984 amendment

110

Page 111: Bankruptcy Outline - Pottow 2011

o The political battle over bankruptcy jurisdiction and the role of the bankruptcy judges was hard fought.

o The senate approach prevailed. Bankruptcy judges remain Art I judges and their jurisdiction would be curtailed.

o 1984 Amendment requires abstention by the federal courts in matters involving state law claims and limited the matters that bankruptcy judges could decide on a “clearly erroneous” basis.

GRANFINANCIERA S.A. V. NORDBERG (US 1989), p.803o It is limited to the question of entitlement to jury trial under the 7th

amendment, but it raises serious questions about the constitutionality of the entire jurisdictional edifice concocted by the 1984 Act.

o Issue: Does a person who has not submitted a claim against a bankruptcy estate have a right to a jury trial when sued by the TIB to recover an allegedly fraudulent monetary transfer?

o Holding: 7th Amendment entitles such a person to a trial by jury, notwithstanding Congress’ designation of fraudulent conveyance actions as “core proceeding” in the Code.

o Reasoning: §548(a)(2) seems to us more accurately characterized as a private

rather than a public right as we have used those terms in our Art. III decision.

Under the 7th amendment, a creditor’s right to a jury trial on a bankruptcy trustee’s reference claim depends upon whether the creditor has submitted a claim against the estate, not upon Congress’ precise definition of the “bankruptcy estate.”

We don’t decide whether the jury trial provision permits bankruptcy courts to conduct jury trials in fraudulent conveyance actions like the one presented.

o Dissent (White, J.): The action to recover fraudulently conveyanced property is

integrally related to the essence of bankruptcy proceeding; “core proceeding.”

Notes: the current legal schemeo The jury-trial rights protected in GRANFINANCIERA would be deemed

waived by a creditor who had voluntarily come into bankruptcy court to assert its remaining claims.

o In 1994 Amendment Congress weighed in with a provision authorizing jury trials in bankruptcy court, but only if a, b and c, p.804, see §157(e).

o 2005 Amendment permits direct appeal from the bankruptcy court to the court of appeals in limited circumstances. §158(d).

o The statute vests a very broad “related to” bankruptcy jurisdiction in the district court. §1334(b). It then limits the actual operation of that jurisdiction by provisions for discretionary or mandatory abstention in favor of state courts. §1334(c)(1), (2).

o §157(a) gives the district judges in each district the collective right to refer some or all of the court’s bankruptcy jurisdiction to the bankruptcy judges.

111

Page 112: Bankruptcy Outline - Pottow 2011

o §157 divides bankruptcy matters into three categories: (a) proceedings that arise under Title 11, (b) proceedings that arise in Title 11 cases, and (c) proceedings that are related to Title 11 cases. Bankruptcy courts may be able to hear (a) and (b) on a “clearly

erroneous” basis. Whether a particular “under” and “in” proceeding can be so heard by the bankruptcy judge depends on its further categorization as a “core proceeding.” Core proceeding will be heard by the bankruptcy judges, but non-core proceedings will go to the district court.

What are core proceedings? §157(b)(2). It is open-ended, see §157(b)(2)(O).

Cases that are merely “related to” may also be heard by the bankruptcy judge, but only as a master making recommendations to the district judge, unless the parties consent to bankruptcy judge jurisdiction. §157(c).

Personal injury and wrongful death claims will be tried by the district judge, unless the parties consent to bankruptcy jurisdiction, §157(c)(2), or the district court abstains on a discretionary basis and permits a state court to try the case. §1334(c)(1). Other related articles: §§157(b)(2)(B), 157(b)(4), 1334(c)(2).

o §157(d) the district judges have the unlimited discretion to withdraw any matter from the bankruptcy court at any time. Furthermore, the district judge must take over any matter involving federal statutes “regulating organizations or activities affecting interstate commerce”.

IN RE DOW CORNING CORP. (6th Cir. 1996), p.812o Facts: This is an appeal to determine the subject matter jurisdiction

of federal district courts, sitting as bankruptcy courts, over proceedings “related to” a case filed under Ch. 11 and the ability of federal district courts to transfer such proceedings to the district court in which the bankruptcy case is pending. These are not the plaintiffs suing the defendant, but is other plaintiffs suing other defendants.

o Issue: Whether the district court has subject matter jurisdiction over breast implant claims pending not only against the debtor, Dow Corning, but also over certain claims pending against the nondebtor defendants.

o Holding: Remanded to the district court.o Reasoning:

Congress intent was to grant comprehensive jurisdiction to the bankruptcy courts so that they might deal efficiently and expeditiously with all matters connected with the bankruptcy estate.

Whether a civil proceeding is related to bankruptcy is whether the outcome of that proceeding could conceivably have any effect on the estate being administered in bankruptcy.

Abstention analysis: it is for the district court to determine in each individual case whether hearing it would promote or impair

112

Page 113: Bankruptcy Outline - Pottow 2011

efficient and fair adjudication of bankruptcy cases: discretionary abstention and mandatory abstention, see p.819 bottom

§157(b)(4) rendered exempt from the mandatory abstention requirement all personal injury tort claims pending against Dow Corning, and decided not to abstain discretionarily with regard to those claims at this time.

The district court is in a better position to make the necessary abstention determinations.

o Pottow: It is far away, but the court still held that it was related to the bankruptcy. The jurisdiction is yes.

IN RE DOW CORNING CORP. (6th Cir. 1997), p.820o This court issued a writ of mandamus ordering the district court to

transfer the claims against the shareholders to the Eastern District of Michigan and to evaluate each claim individually to determine whether mandatory abstention applies.

o Reasoning: Two reasons, p.821.

IN RE US LINES, INC. (2d Cir. 1999), p.822o The bankruptcy code divides claims in bankruptcy proceedings into

core and non-core. With respect to non-core claims, unless the parties otherwise agree, the bankruptcy court can only recommend findings of fact and conclusions of law to the district court.

o Proceedings can be core by virtue of their nature if either (1) the type of proceeding is unique to or uniquely affected by the bankruptcy proceedings, or (2) the proceedings directly affect a core bankruptcy function.

o Issue: Whether the underlying insurance contract claims are core proceedings.

o Holding: Yes.o Reasoning:

The K claims are not rendered core simply because they involve property of the estate

The critical question in determining whether a K dispute is core by virtue of timing is not whether the cause of action accrued post-petition, but whether the K was formed post-petition. The bankruptcy court has core jurisdiction over claims arising from a K formed post-petition. But a dispute arising from pre-petition K will usually not be rendered core simply because the cause of action could only arise post-petition.

Whether a lawsuit alleging a post-petition breach of a pre-petition K is a core proceeding depends on the impact the K has on core bankruptcy function.

Concurring, Newmano Whether a suit for a post-petition breach of a pre-petition K is core

remains openo The efficient functioning of the bankruptcy system will be better

served by a bright-line rule that treats as core proceedings all suits alleging post-petition breaches of pre-petition Ks.

113

Page 114: Bankruptcy Outline - Pottow 2011

Problem Set 41, p.83341.1

o Claims: Want to file bankruptcy, also have defective goods claim against creditor. Who takes the case? Would the D judge or the bankruptcy judge take this?

o Seems like B judge takes it because this counterclaim against a creditor it is a core proceeding. BUT NO, Atlantic has not filed a claim so this does not meant (b)(2)(C).

o What about (b)(2)(O)? the catchall provision about the core proceedings

o Do we have an abstention issue here? Not mandatory. Not already in state court.

(2) The second parto If this was a Chapter 11, still no mandatory abstention. It cannot be

adjudicated timely in a state court (18 months).o Independent federal jurisdiction? Yes, if there is the antitrust claim

too. And then supplementary jurisdiction for the defective goods claim.

o Does it depend when the breach occurs? Seems like any post-petition breach would be a core proceeding. Remember Calabresi opinion.

o If you want to solve it quickly, you can settle or drop it; sell the claim?41.2

o Title VII is federal jurisdiction. D judge case?o Related to bankruptcy (broad interpretation from DOW CORNING ), but

core or non-core proceeding. Is this a personal injury claim excepted from core proceedings in (b)(2)(B), (O)? What about the assault claim?

o Mandatory abstention for any of this? None of these have already been commenced in state action. Could still have discretionary abstention.

41.3o §544(b). It is avoidableo Is it a state cause or a federal cause of action? It is a state law. But is

it also a federal cause of action? The policy reasons. §157(b)(3).o The federal court can hear the case.o Before a district judge or district judge? Core proceeding under

§157(b)(2)(H).o Is it subject to mandatory abstention? No. It cannot be timely

adjudicated; get to trial for at least two years; not commenced in state court (and is arising under Title 11); §1334(c)(2).

o How about the discretionary abstention under §1334(c)(1)? Comity with state courts. The judge should not abstain. It is a core proceeding.

o If the opponent seeks delay, she can bring a motion for abstention. She might get the Supreme Court’s review by challenging unconstitutionality of the jurisdiction rules of the Bankruptcy Code.

41.4o §523(a)(6): Willful and malicious injury is not dischargeable, but it is

still a claim of the bankruptcy.

114

Page 115: Bankruptcy Outline - Pottow 2011

o Ethel's lawsuit can't go forward because of the automatic stay.o Is it a core proceeding? §157(b)(2)(B). The personal injury is not a

core proceeding.o Is it subject to the mandatory jurisdiction under §1334(c)(2)?

Five requirements: (1) it is a state law cause of action-torts; (2) only related to the Title 11; (3) lack of a federal JRX basis absent the bankruptcy; (4) the action is commenced; and (5) it can be timely adjudicated.

All conditions are met.o But §157(b)(4) clearly states that it is not subject to the mandatory

abstention; but can be discretionary abstention.o Can you appeal the order about the abstention? No. See §1334(d).

By the way, the decision can only be appealed when it becomes a judgment or order.

o If we conclude no abstention, is it subject the district judge or bankruptcy judges? At least in a capacity where he passes findings of fact to the district judge? (c)(1). No. It should be adjudicated in the district court under §157(b)(5).

Business Bankruptcy Theory 873-889

A. Introduction

B. Exclusions from bankruptcy o Financial companies and Securities Investors Protection Act (SIPC):

Reasons to be excluded: they hold large amounts of value deposited (as opposed to invested) by the public and they are heavily regulated at all times, not just when they become financially distressed.

o Railroad: history backgroundso Hedge funds

Technically, they are eligible to file for bankruptcy. Yet they are effectively excluded, because a bankruptcy filing

for them would be pointless. The financial instruments like “swaps” and “repos” are exempt from the automatic stay and the avoiding powers. This means that virtually all of their assets would be effectively seized by the “counterparties” to these instruments at the moment of bankruptcy or shortly thereafter, leaving nothing for a bankruptcy court to administer. Thus, the general default of many hedge funds is effectively excluded from bankruptcy

o Long Term Capital Management LTCM was the mother of all hedge funds.

o There is a strong movement to increase financial exemptions from bankruptcy protection, like “asset securitization.”

C. Alternative approaches to bankruptcy o The traditional approaches to bankruptcy law were to ensure equality

of distribution and to prevent fraud and unravel its effects.

115

Page 116: Bankruptcy Outline - Pottow 2011

o Two most popular alternatives: automated bankruptcy and contractualists.

Automated bankruptcy theoryo Automated bankruptcy rests on the idea that it would be useful to

separate the purely economic function of bankruptcy from the litigation aspects, so that a business could be rapidly and inexpensively liquidated, sold, or placed under new management, without the cost and delay associated with litigation over issuers such as fraud, mismanagement, and lender liability.

o Professor Bebchuko It would not require much court involvement. The process would be

completely controlled by creditors.Contractualist approach

o The contractualists propose eliminating or narrowing bankruptcy law so that it contains a minimum of mandatory rues and permits economic actors to construct their own rules to manage general default. The range of proposals runs from virtually complete contractualization of bankruptcy to a broad tolerance of waivers of bankruptcy rules by debtors.

o Robert Rasmussen: bankruptcy menu from which a firm chooses the bankruptcy provisions that would be best for it and its creditors. It would be made at the inception of the firm and would be included in its corporate charter.

o Alan Schwartz: Tecognized a problem in menu approach that some situations will change in the development, so he proposes a rolling readjustment in the bankruptcy regime to reflect these changes. He thinks that the principal obstacle to the most efficient choice of

bankruptcy solutions is the “private” benefit that those controlling the firm may gain from the choice (e.g., reorganization and another paycheck). These benefits are like a bribe. It will make the controller do the suboptimal choice.

Therefore, the renegotiation-proof contract often would be best. o Some contractualists have a more limited agenda. They propose that

bankruptcy law remain as it is but that to some extent the rules become default rules rather than mandatory ones. That is, the parties would be free to waive the default rules. Marshall Tracht Steven Schwarcz also favors some bankruptcy waivers and

procedure contracts, but is more detailed and selective in analyzing which sorts of waivers are consistent with bankruptcy policy and which are not. His central argument is that pre-bankruptcy waivers will permit debtors to obtain the liquidity necessary to avoid bankruptcy.

o These theories have been criticized on various grounds. Professor Lynn LoPucki: argues that the basic model proposed by

Alan Schwartz lacks both internal logic and a mathematical basis. Most creditor’s interests are too small to warrant their active, knowledgeable participation.

Susan Block-Lieb: A central obstacle to all the contractualist proposals: The collective nature of the bankruptcy process.

116

Page 117: Bankruptcy Outline - Pottow 2011

o There is a second problem with all of the theorists discussed to this point-automated, contractualist and creditors’ bargain-which is the missing-debtor problem.

o A growing school of thought that control of the general default process is the central concept in bankruptcy law, whatever system of priorities for various interests might be adopted.

o From a policy perspective, a number of bankruptcy scholars have rejected the contractual or automatic bankruptcy theories, believing that bankruptcy laws serve interests beyond those of maximizing value to creditors or producing the lowest cost of borrowing. Elizabeth Warren

Problem 43.2, p.899o Their contractual law?o Secured transaction law?

409-131. Control

o Control is a central concept in all bankruptcy laws. The immediate reponses to the filing- the automatic stay, the automatic transfer of ownership of the debtor’s property to a new legal entity, and the appointment of a trustee in bankruptcy-put the courts in overall charge of the debtor and its business and therefore empower these agents of gov to sort out the effects of a debtor’s general default.

o DIP concept was to put the existing pre-bankrutpcy management in charge of Ch 11 debtor but with no congressional definition of management’s role. They can act on behalf of several possible constituencies, like shareholders, creditors, employees and communities, etc. finally, management will give consideral attention to protecting its own interests.

o Management turnover in Ch 11 cases: in public companies, more than 90% CEOs resign within 2 years after the filing date. Some scholars have claimed that public company Ch 11 cases are controlled by creditors.

o Control of smaller businesses in Ch 11 is also allegedly abused.o The question of control: who exercises it for whose benefit?

2. Large public companies and nontraditional Ch 11 caseso New approaches to the Ch 11 cases have been emerging. The two

most outstanding: auctions and prepackaged plans.

a. Auctionso It is increasingly common for a sale of the entire business for a

financially distress company.o The sale is quite often through an auction process, although nothing

like the old-fashioned, piecemeal liquidation with a fast-talking auctioneer. It usually involves lawyers and bankers, like an M&A.

o Two points:

117

Page 118: Bankruptcy Outline - Pottow 2011

First, it almost always means no meaningful recoveries for the shareholders.

Second, the auction may be held pursuant to a confirmed plan, but often is not. In the latter case, the sale is completed under §363 prior to any creditor vote and the plan is simply a mechanism for allocating and distributing the proceeds of the sale.

b. Prepackaged planso The debtor has negotiated the deal with its creditors before the

petition is ever filed, but needs the help of the bankruptcy law to close the deal.

3. Single asset real estate cases (SARE)o SARE is defined in §101(51B)o It is often a two-party struggle.

4. Liquidation in Ch 11o 20% or more of the confirmed plans in Ch 11 are liquidating plans.o Liquidating plans are often with auctions and with prepackaged plans.o In fact, nowadays very few companies of substantial size liquidate in

Ch 7.

5. Costs of Ch 11

LBO, 97-98o The use of fraudulent conveyance statutes to attack LBOs has drawn

sharp academic criticism. The critics have asserted that current fraudulent conveyance law is too all-encompassing in its effects.

Ch 10 Transnational Bankruptcies

A. Introductiono Traditionally, the grab rule: seizing local assets and distributing them

to the claimants in a local proceeding, with little concern for the overall result for the company or for claimholders outside the domestic jurisdiction.

o International bankruptcy: territorialism and universalism.o In reality, difference between the modified territorialism and modified

universalism.o Choice of forum and choice of law.

CH 15o §1501 purpose of this chapter

To preserve employment; is this good? Is it within the purview of Congress? How about “to preserve Medicare” in bankruptcy code? Is it

within the purview of Congress? Should the employment be put here?

Fair and efficient administration

118

Page 119: Bankruptcy Outline - Pottow 2011

…o Liquidation vs. Reorganization

Reorganization is preferred; is it because of the employment? If not, what are the reasons?

B. Choice of forumo Choice of forum arises only in a system of modified universalism.o Choice of forum means choice of a primary forum, the lead court in

administering the case. The country where the lead court sits is the home country, typically the situs of the “center of its main interests”

o Protocols emerge: protocols are agreements among major stakeholder groups as to how a multinational bankruptcy will be managed.

1. Home court: USo §541 the estate reaches the property wherever located.

IN RE MCLEAN INDUSTRIES (1986), p.844o The broad reach of US jurisdiction claims with respect to the debtor’s

property and the serious practical difficulties of enforcement.o Facts: US Lines, the debtor, seeks from this court a preliminary

injunction restraining defendant GAC Marine from taking any action to arrest or interfere with vessels and other property of this estate. It further seeks an order holding GAC in civil attempt for violating automatic stay. GAC declared that it was incorporated in UK and its principal place of business was in London, so it is not subject to the personam jurisdiction. GAC arrested the ships in HK and Singapore.

o Issue: Whether the defendant is subject to in personam jurisdiction.o Holding: Yes.o Reasoning:

Two grounds: transactional jurisdiction and general doing business jurisdiction in the Restatement of Conflicts of Laws.

The Supreme Court has taken the formula in terms of claims that “arising out of or relate to” forum business activities or activities.

Here both a strict relevance test and a historical connection test are satisfied.

Doing business test, p.84x fine print. GAC has an office presence in New Jersey.

IN RE MCLEAN INDUSTRIES (1987), p.848o GAC asserted that it was impossible for it to comply because it

assigned its pre-petition claims against the debtor to another company.

o Holding: The GAC’s argument cannot stand; enter for the debtor.o Reasoning:

No evidence was offered to show that GAC Marine cannot buy back the claim from FAL; FAL did not testify that it would not retransfer the claim.

Instead of taking steps to discontinue those arrest proceedings as the contempt decree required, GAC took the active steps of ensuring their continuation.

119

Page 120: Bankruptcy Outline - Pottow 2011

Thus, GAC violated §362(a)(1), (4) and (6) renew. o Notes: in this case, the recalcitrant creditor had enough contacts with

the US to permit enforcement of the stay; conversely, the extension of the stay to overseas assets to the extent the US courts have personal jurisdiction over creditors creates a serious risk of conflict with other countries.

2. Home court: elsewhere

X V. SCHENKIUS (RECEIVER FOR Y) , (1995) from Netherlands, p.852o The Netherlands court decided that the overseas property that

belongs to the debtor is also part of the estate. The receiver may attempt to liquidate, for the benefits of the estate, any debtor’s property that is located abroad.

o Notes: some countries, like Japan, take a traditional highly territorial view that they make no claim whatsoever to overseas property; some countries take a more cosmopolitan view like Netherlands.

o The movement to a less territorial and more cooperative rule.

C. Cooperating with the home country court1. Cooperating court: US

o §304 is designed to permit and encourage US bankruptcy courts to cooperate with home countries courts abroad.

o UN 1997 Model Law of Cross-border Insolvencyo Part of Model Law was adopted by 2005 amendment and became Ch

15.

IN RE BOARD OF DIRECTORS OF MULTICANAL S.A. , 2004, p.860o Argentina’s proceeding may be recognized and enforced in the US.

The involuntary parallel proceeding was dismissed under §305 in favor of ancillary cooperation with the main proceeding in Argentina.

o Conflicting goals: international cooperation and protection of US creditors-Congress simply incorporated them both and left it to the courts to sort out.

o NY courts and the 2nd Cir refuse to enforce Bahamian proceeding, because Bahamian was too different from US law to satisfy 304(c).

o Ch 15 has eliminated the §304(c) requirements except in one unusual respect. See 1507.

o The most far-reaching change in the Model law is to encourage direct communication between courts in multinational bankruptcy cases.

2. Cooperating court: elsewhereo In response to §304 initiative, more cooperation trend.

ROBERTS V. PICTURE BUTTE MUNICIPAL HOSPITAL , 1999, p.857o Facts: DCC the breast implant manufacturer was sued by Canadian

customers. DCC applied to the Canadian court to enforce the US court’s stay order.

120

Page 121: Bankruptcy Outline - Pottow 2011

o Whether there has been an attornment or not, I conclude it is appropriate for me to exercise my discretion and apply the principles of comity and grant the defendant’s stay application.

D. International law reformo International bankruptcy law reforms sponsored by IMF, World Bank,

etc.,. have not addressed choice of law, but have focused on cooperation among courts in managing international cases.

o EU has adopted a bankruptcy regulation since May 2002 that is binding on all member countries and enforceable in their courts.

E. Choice of lawo Three aspects of applicable law are most important: avoiding powers,

distribution (priority) rules and discharge.

BARCLAYS BANK P.L.C. V. HOMAN AND OTHERS , 1992, p.862o US could be the natural forum, since the presence in the US of

substantial creditors, of assets and of moneys derived from asset, coupled with the absence of any challenge to the Ch 11 proceeding. Leave to appeal to the House of Lords refused.

IN RE MAXWELL COMMUNICATION CORP. PLC (1996), p.874o The administrators proceeded to file their preference actions in the

US.o Issue: Whether Maxwell Communication, as a debtor estate in Ch 11,

may recover under American law millions of dollars it transferred to three foreign banks shortly before declaring bankruptcy?

o Holding: The doctrine of international comity precludes application of the American avoidance law to transfers in which England’s interest has primacy.

o Reasoning: International comity, p.868 bottom Two reasons for international comity , p.869 middle Primacy of English law. Relative interests of forum and foreign states. Cooperation and harmonization: systemic interest.

Lessons from MAXWELL o Every international bankruptcy case requires both a choice of forum

and a choice of law analysis.o Protocols of cooperation in the administration of an international case

are crucial to the achievement of a satisfactory result in the absence of effective legal rules.

o Politeness matters.

§304 Cases ancillary to foreign proceedings

121

Page 122: Bankruptcy Outline - Pottow 2011

(a) A case ancillary to a foreign proceeding is commenced by the filing with the bankruptcy court of a petition under this section by a foreign representative.(b) Subject to the provisions of subsection (c) of this section, if a party in interest does not timely controvert the petition, or after trial, the court may—

(1) Enjoin the commencement or continuation of—(A) Any action against—

(i) A debtor with respect to property involved in such foreign proceeding; or (ii) Such property; or

(B) The enforcement of any judgment against the debtor with respect to such property, or any act or the commencement or continuation of any judicial proceeding to create or enforce a lien against the property of such estate;

(2) Order turnover of the property of such estate, or the proceeds of such property, to such foreign representative; or (3) Order other appropriate relief.

(c) In determining whether to grant relief under subsection (b) of this section, the court shall be guided by what will best assure an economical and expeditious administration of such estate, consistent with—

(1) Just treatment of all holders of claims against or interests in such estate;

(2) Protection of claim holders in the United States against prejudice and inconvenience in the processing of claims in such foreign proceeding; (3) Prevention of preferential or fraudulent dispositions of property of

such estate; (4) Distribution of proceeds of such estate substantially in accordance with the order prescribed by this title; (5) Comity; and (6) If appropriate, the provision of an opportunity for a fresh start for the individual that such foreign proceeding concerns.

Problem 42, p.880o We should know Canadian law; how to get it? Foreign experts.

The scope of jurisdiction of Canadian law. Does the Canadian court have a personal jurisdiction over

CompuTech?o We can ignore the stay, right? Pottow: we can ignore it.o Will US courts recognize the Canadian stay? It can recognize.

§1515 application for recognition? Ancillary proceeding. Comity analysis: Discretionary, not compulsory.

§304 proceedings: §304(c) whether to extend the comity proceedings

Public policyo Can Canadian guy file for Ch 7 in U.S. courts?

They have assets.

122

Page 123: Bankruptcy Outline - Pottow 2011

What is difference between Ch 7 and §304 (now Chapter 15) proceedings? Timing, use of automatic stay. Wouldn't the U.S. filing cover the whole estate no matter where

the things are located? Yes. A chapter 7 is a plenary proceeding, not an ancillary

proceeding. If we have two separate, plenary, proceedings going on simultaneously, we still want cooperation. See MAXWELL . There they drafted a protocol, a treaty, to reconcile the two judge's decision.

An ancillary proceeding is jurisdictionally inferior, and it recognizes the territorial scope of the assets. The U.S. court only exercises jurisdiction of the assets in the territorial U.S. in its ancillary capacity.

o If I were Maple Leaf, I will not file for Ch 15; just open the CH 7 or 11 proceedings because I HAVE assets in the U.S.

Class notes: o §1520 effects of recognition of a foreign main proceeding.

(a) limited to U.S. territory (c) subsection (a) does not affect the right of a foreign entity to file

a petition commencing a case in the U.S. Does (c) influence the foreign main proceeding? No?

o §1506 public policy; the court can reject a case based on public policy of U.S.

o §1517 tries to allocate the jurisdiction to one country. Foreign proceeding shall be recognized as a foreign main

proceeding if it is pending in the country where the debtor has the center of its main interests.

o If Country A has more important jurisdiction than B country. Main proceeding: the debtor’s center of main interest. Non-main proceeding: the foreign country can also ask for

assistance of the U.S. courts, it is discretionary.o §1528 – commencement of a case under this title after recognition of

a foreign main proceeding. after recognition of a foreign main proceeding, a case under

another chapter of this title may be commenced only if the debtor has assets in the U.S.

The effects of such case shall be restricted to the assets in the U.S.o §1529 – coordination of a case under this title and a foreign

proceeding. If a foreign proceedings and a case under another chapter are

pending concurrently regarding the same debtor, the court shall seek cooperation and coordination under §§ 1525/6/7, and §1520 does not apply even if such foreign proceeding is recognized as a foreign main proceeding.

Optional universalism.

123